OBM por assunto: Os primeiros passos olímpicos · tenha muito a acrescentar na sua vida pois o...

169

Transcript of OBM por assunto: Os primeiros passos olímpicos · tenha muito a acrescentar na sua vida pois o...

Page 1: OBM por assunto: Os primeiros passos olímpicos · tenha muito a acrescentar na sua vida pois o foco do treinamento é justamente para alcançar esse nível de habilidade. ... Esse

OBM por assunto: Os primeiros passos

olímpicos

Alan Anderson

25 de fevereiro de 2018

Page 2: OBM por assunto: Os primeiros passos olímpicos · tenha muito a acrescentar na sua vida pois o foco do treinamento é justamente para alcançar esse nível de habilidade. ... Esse

2

Page 3: OBM por assunto: Os primeiros passos olímpicos · tenha muito a acrescentar na sua vida pois o foco do treinamento é justamente para alcançar esse nível de habilidade. ... Esse

Sumário

0.1 Para quem é esse texto? . . . . . . . . . . . . . . . . . . . . . . . 60.2 Estrutura do Livro . . . . . . . . . . . . . . . . . . . . . . . . . . 60.3 Agradecimentos . . . . . . . . . . . . . . . . . . . . . . . . . . . . 8

1 Problemas básicos 91.1 Conjuntos . . . . . . . . . . . . . . . . . . . . . . . . . . . . . . . 91.2 Extremos . . . . . . . . . . . . . . . . . . . . . . . . . . . . . . . 101.3 Lógica . . . . . . . . . . . . . . . . . . . . . . . . . . . . . . . . . 10

1.3.1 Calculo direto . . . . . . . . . . . . . . . . . . . . . . . . . 20

2 Indução 232.1 O Princípio da Indução . . . . . . . . . . . . . . . . . . . . . . . 23

2.1.1 Exemplos . . . . . . . . . . . . . . . . . . . . . . . . . . . 242.1.2 Exercícios . . . . . . . . . . . . . . . . . . . . . . . . . . . 26

2.2 Possíveis equívocos . . . . . . . . . . . . . . . . . . . . . . . . . . 262.2.1 Cavalos: Indução que deu errado . . . . . . . . . . . . . . 272.2.2 "Pela lógica" não é prova . . . . . . . . . . . . . . . . . . 27

2.3 Indução Forte . . . . . . . . . . . . . . . . . . . . . . . . . . . . . 282.3.1 Exercícios . . . . . . . . . . . . . . . . . . . . . . . . . . . 29

3 Combinatória e Probabilidade 313.1 Exercícios de aquecimento . . . . . . . . . . . . . . . . . . . . . . 313.2 Princípios aditivo e multiplicativo . . . . . . . . . . . . . . . . . . 323.3 Números binomiais . . . . . . . . . . . . . . . . . . . . . . . . . . 38

3.3.1 Fatorial . . . . . . . . . . . . . . . . . . . . . . . . . . . . 383.3.2 Número Binomial . . . . . . . . . . . . . . . . . . . . . . . 393.3.3 Justi�cando o nome . . . . . . . . . . . . . . . . . . . . . 40

3.4 Princípio das casas dos pombos . . . . . . . . . . . . . . . . . . . 413.5 Invariantes . . . . . . . . . . . . . . . . . . . . . . . . . . . . . . 453.6 Probabilidade . . . . . . . . . . . . . . . . . . . . . . . . . . . . . 47

3.6.1 Probabilidade Discreta . . . . . . . . . . . . . . . . . . . . 483.6.2 Probabilidade Geométrica . . . . . . . . . . . . . . . . . . 49

4 Álgebra 534.1 Exercícios de Equações e sistemas . . . . . . . . . . . . . . . . . . 53

4.1.1 Equações . . . . . . . . . . . . . . . . . . . . . . . . . . . 534.1.2 Sistemas . . . . . . . . . . . . . . . . . . . . . . . . . . . . 55

4.2 Produtos Notáveis e Fatorações Notáveis . . . . . . . . . . . . . . 584.2.1 Lista de produtos notáveis . . . . . . . . . . . . . . . . . . 58

3

Page 4: OBM por assunto: Os primeiros passos olímpicos · tenha muito a acrescentar na sua vida pois o foco do treinamento é justamente para alcançar esse nível de habilidade. ... Esse

4 SUMÁRIO

4.2.2 Completando Quadrados . . . . . . . . . . . . . . . . . . 614.3 Equação do Segundo Grau . . . . . . . . . . . . . . . . . . . . . . 614.4 Binômio de Newton . . . . . . . . . . . . . . . . . . . . . . . . . 654.5 Polinômios . . . . . . . . . . . . . . . . . . . . . . . . . . . . . . 66

4.5.1 Conceitos Básicos . . . . . . . . . . . . . . . . . . . . . . 664.5.2 Teorema Fundamental da Álgebra . . . . . . . . . . . . . 68

5 Análise 715.1 Desigualdes . . . . . . . . . . . . . . . . . . . . . . . . . . . . . . 71

5.1.1 Exercícios de aquecimento . . . . . . . . . . . . . . . . . . 715.1.2 Desigualdade entre as médias . . . . . . . . . . . . . . . . 765.1.3 Desigualdade de Cauchy-Schwarz . . . . . . . . . . . . . . 77

5.2 Função Parte Inteira . . . . . . . . . . . . . . . . . . . . . . . . . 785.3 Sequências . . . . . . . . . . . . . . . . . . . . . . . . . . . . . . . 80

5.3.1 OBM . . . . . . . . . . . . . . . . . . . . . . . . . . . . . 805.3.2 Progressões Aritmética e Geométrica . . . . . . . . . . . . 81

5.4 Séries . . . . . . . . . . . . . . . . . . . . . . . . . . . . . . . . . 835.4.1 Somas telescópicas . . . . . . . . . . . . . . . . . . . . . . 835.4.2 Somas com Números Binomiais . . . . . . . . . . . . . . . 86

5.5 Funções . . . . . . . . . . . . . . . . . . . . . . . . . . . . . . . . 875.5.1 Introdução . . . . . . . . . . . . . . . . . . . . . . . . . . 875.5.2 Substituição . . . . . . . . . . . . . . . . . . . . . . . . . . 885.5.3 Mais problemas sobre funções . . . . . . . . . . . . . . . . 91

6 Teoria dos Números 936.1 Divisibilidade . . . . . . . . . . . . . . . . . . . . . . . . . . . . . 93

6.1.1 Máximo divisor comum . . . . . . . . . . . . . . . . . . . 966.1.2 Critérios de divisibilidade . . . . . . . . . . . . . . . . . . 996.1.3 Teorema Fundamental da Aritmética . . . . . . . . . . . . 102

6.2 Congruências . . . . . . . . . . . . . . . . . . . . . . . . . . . . . 1046.2.1 De�nição e exemplos . . . . . . . . . . . . . . . . . . . . . 1046.2.2 Propriedades . . . . . . . . . . . . . . . . . . . . . . . . . 105

6.3 Pequeno Fermat e Grande Euler . . . . . . . . . . . . . . . . . . 1066.3.1 Pequeno Teorema de Fermat . . . . . . . . . . . . . . . . 1076.3.2 Função φ de Euler . . . . . . . . . . . . . . . . . . . . . . 107

6.4 Teorema Chinês dos Restos . . . . . . . . . . . . . . . . . . . . . 1096.5 Problemas variados . . . . . . . . . . . . . . . . . . . . . . . . . . 111

6.5.1 Contagem com TN . . . . . . . . . . . . . . . . . . . . . . 1116.5.2 Equações com inteiros . . . . . . . . . . . . . . . . . . . . 1136.5.3 Outros Problemas . . . . . . . . . . . . . . . . . . . . . . 113

7 Geometria 1157.1 Fatos básicos . . . . . . . . . . . . . . . . . . . . . . . . . . . . . 115

7.1.1 Desigualdade triangular . . . . . . . . . . . . . . . . . . . 1157.1.2 Figuras planas básicas . . . . . . . . . . . . . . . . . . . . 1167.1.3 A soma dos ângulos internos . . . . . . . . . . . . . . . . 1177.1.4 Diagonais . . . . . . . . . . . . . . . . . . . . . . . . . . . 121

7.2 Pitágoras e Triângulo Retângulo . . . . . . . . . . . . . . . . . . 1217.2.1 Semelhança de triângulos . . . . . . . . . . . . . . . . . . 1217.2.2 Teorema de Pitágoras . . . . . . . . . . . . . . . . . . . . 122

Page 5: OBM por assunto: Os primeiros passos olímpicos · tenha muito a acrescentar na sua vida pois o foco do treinamento é justamente para alcançar esse nível de habilidade. ... Esse

SUMÁRIO 5

7.2.3 Potência de Ponto . . . . . . . . . . . . . . . . . . . . . . 1257.3 Informações básicas de trigonometria . . . . . . . . . . . . . . . . 126

7.3.1 Seno e Cosseno como funções reais . . . . . . . . . . . . . 1277.3.2 Ângulos notáveis . . . . . . . . . . . . . . . . . . . . . . . 1277.3.3 Fórmulas úteis . . . . . . . . . . . . . . . . . . . . . . . . 1277.3.4 Substituição Trigonométrica . . . . . . . . . . . . . . . . . 128

7.4 Lei dos Cossenos . . . . . . . . . . . . . . . . . . . . . . . . . . . 1307.5 Pontos, retas e segmentos notáveis . . . . . . . . . . . . . . . . . 131

7.5.1 Bissetrizes . . . . . . . . . . . . . . . . . . . . . . . . . . . 1317.5.2 Alturas . . . . . . . . . . . . . . . . . . . . . . . . . . . . 1337.5.3 Medianas . . . . . . . . . . . . . . . . . . . . . . . . . . . 1337.5.4 Mediatrizes . . . . . . . . . . . . . . . . . . . . . . . . . . 133

7.6 Ângulos Inscritos e Ângulos Tangentes . . . . . . . . . . . . . . . 1347.6.1 Ângulos tangentes . . . . . . . . . . . . . . . . . . . . . . 1347.6.2 Ângulos Inscritos . . . . . . . . . . . . . . . . . . . . . . . 136

7.7 Áreas de �guras planas . . . . . . . . . . . . . . . . . . . . . . . . 1387.7.1 Área do triângulo . . . . . . . . . . . . . . . . . . . . . . . 1437.7.2 Lei dos Senos . . . . . . . . . . . . . . . . . . . . . . . . . 144

7.8 Quadriláteros Inscritíveis . . . . . . . . . . . . . . . . . . . . . . 1457.9 Geometria espacial . . . . . . . . . . . . . . . . . . . . . . . . . . 147

8 Enunciados dos Problemas 1498.1 Combinatória e Probabilidade . . . . . . . . . . . . . . . . . . . . 149

8.1.1 Contagem . . . . . . . . . . . . . . . . . . . . . . . . . . . 1498.1.2 Extremo . . . . . . . . . . . . . . . . . . . . . . . . . . . . 1528.1.3 Jogos e Invariantes . . . . . . . . . . . . . . . . . . . . . . 1528.1.4 Probabilidade . . . . . . . . . . . . . . . . . . . . . . . . . 154

8.2 Álgebra . . . . . . . . . . . . . . . . . . . . . . . . . . . . . . . . 1548.2.1 Funcões e Recorrências . . . . . . . . . . . . . . . . . . . 1548.2.2 Somatórios e Produtórios . . . . . . . . . . . . . . . . . . 1568.2.3 Equações do segundo grau e Polinômios . . . . . . . . . . 1568.2.4 Parte Inteira e Aproximação . . . . . . . . . . . . . . . . 1578.2.5 Cotas e desigualdades . . . . . . . . . . . . . . . . . . . . 1578.2.6 Várias variáveis . . . . . . . . . . . . . . . . . . . . . . . . 158

8.3 Teoria dos Números . . . . . . . . . . . . . . . . . . . . . . . . . 1588.3.1 Equações e divisibilidade . . . . . . . . . . . . . . . . . . 1588.3.2 Teorema Fundamental da Aritimética . . . . . . . . . . . 1608.3.3 Teorema Chinês dos Restos e Sistemas de Congruências . 1608.3.4 Fermat e Euler . . . . . . . . . . . . . . . . . . . . . . . . 161

8.4 Geometria . . . . . . . . . . . . . . . . . . . . . . . . . . . . . . . 1628.4.1 Básico . . . . . . . . . . . . . . . . . . . . . . . . . . . . . 1628.4.2 Semelhança de Triângulos e Triângulos Retângulos . . . . 1638.4.3 Lei dos Senos e dos Cossenos . . . . . . . . . . . . . . . . 1648.4.4 Áreas . . . . . . . . . . . . . . . . . . . . . . . . . . . . . 1658.4.5 Pontos notáveis e Ângulos na circunferência . . . . . . . . 1668.4.6 Potência de Ponto . . . . . . . . . . . . . . . . . . . . . . 1678.4.7 Geometria Espacial . . . . . . . . . . . . . . . . . . . . . . 167

Page 6: OBM por assunto: Os primeiros passos olímpicos · tenha muito a acrescentar na sua vida pois o foco do treinamento é justamente para alcançar esse nível de habilidade. ... Esse

6 SUMÁRIO

0.1 Para quem é esse texto?

Comecei a fazer esse texto para ajudar alguns amigos, que na época estudavamno Instituto Federal de Educação, Ciência e Tecnologia de Alagoas (IFAL). Mastrabalhos geram ideias que dão trabalho. Inicialmente seriam somente os pro-blemas da antiga segunda fase separados por área, mas agora o livro visa ajudarestudantes que não sabem exatamente como estudar para as olimpíadas. Es-crevi um texto que eu gostaria de ter lido quando fazia ensino médio e estudavapara as olimpíadas, espero que seja útil.

Um grupo que talvez não faça proveito máximo desse livro são as pessoasque acham que a prova da segunda fase da OBM é fácil, esse texto talvez nãotenha muito a acrescentar na sua vida pois o foco do treinamento é justamentepara alcançar esse nível de habilidade. Eu classi�caria como um texto de nívelbásico que te leva a um nível intermediário.

A principal singularidade do texto é o fato de que ele está cheio de resul-tados sem demonstrações com convites e os passos para que o próprio leitordesenvolva a demonstração, às vezes os exercícios que estão antes dos resultadossão informações úteis na demonstração do mesmo. As proposições em que oleitor é convidado a demonstrar, em geral são simples, e por simples quero dizerque é algo que não necessariamente você ache fácil de fazer ao tentar, mas quequando você �zer provavelmente pensará "ah, é só isso". Em geral, serão deixa-das nos esboços as idéias principais das provas, �cando para o leitor o trabalhode fazer as contas, de desenvolver ideias simples e aplicar ideias anteriormenteapresentadas no próprio texto. A �loso�a do livro é "Matemática se aprendefazendo".

Um fato relevante é que esse livro não é um desenvolvedor de criatividade.Esse livro é um livro feito para treinar a técnica, deixaremos para o próprio leitora responsabilidade de desenvolver a criatividade, e uma sugestão de como fazerisso é: pegue problemas da terceira fase da OBM e problemas das olimpíadasinternacionais (IMO, IberoAmerican, ConeSul, etc). Aqui você aprenderar apintar quadros, mas �cará em suas mãos a jornada para se tornar um artista.

0.2 Estrutura do Livro

O primeiro capítulo do livro trata de problemas básicos, isto é, problemas quepoderiam ser feitos com bem pouco conhecimento. Isso não necessariamenteimplica que o problema seja fácil, apenas que seja simples. Não há conteúdonesse capítulo, ele funciona como um capítulo de aquecimento. Sugiro que emcada seção de estudo o leitor escolha um problema arbitrário desse capítulo,tente resolvê-lo o máximo possível, e quando concluir essa missão comece aestudar o conteúdo planejado. Não pare um dia pra resolver todos os exercíciosnem deixe para seguir em frente só quando terminá-lo.

O capítulo 2 é sobre o Princípio da Indução. Esse capítulo é importantepra você como estudante de matemática, mas não é essencial pra você comoleitor desse livro. A principal razão pela qual ele está no livro é que diversasdemonstrações nos capítulos seguintes dependem de indução, aí eu deixo umtrabalho de cunho psicológico para o leitor: decidir se quer ou não ler logo ocapítulo. Se decidir não ler o capítulo, o leitor pode ir direto para o foco dolivro, no entanto irá ser "necessário" pular algumas demonstrações (que �caram

Page 7: OBM por assunto: Os primeiros passos olímpicos · tenha muito a acrescentar na sua vida pois o foco do treinamento é justamente para alcançar esse nível de habilidade. ... Esse

0.2. ESTRUTURA DO LIVRO 7

como exercício) pois dependem de indução (sempre que a demonstração requeririndução isso será deixado explícito). Se decidir ler o capítulo terá que adiar umpouquinho a leitura principal, mas terá aprendido uma ferramenta de extremaimportância para o futuro.

O capítulo 3, sobre combinatória, ensina os conhecimentos mais básicos dessaárea, entre os quais, um que é especialmente importante: o princípio das casasdos pombos. Esse princípio é muito relevante porque ele é muito útil em áreasdiversas, não é uma coisa que vai se fechar em combinatória, sem contar que éteorema com o nome mais legal desse livro. Seus passeios pelo centro da cidadenunca mais serão os mesmos...

O capítulo 4, sobre álgebra, talvez seja aquele com o qual o leitor terá maiorfamiliaridade, a�nal é um dos que mais intersecta o conteúdo apresentado naescola. Produtos notáveis, equações do segundo grau, polinômios. Mas há oque aprender aqui também, coisas as quais não se é dado muito foco na escola,como a demonstração da fórmula da equação do segundo grau e alguns produtosnotáveis menos comuns, por exemplo. Além disso o foco nos polinômios temuma natureza um pouco diferente daquela dada nas escolas.

O capítulo 5, trata dos primeiros passos na área conhecida como análise. Láo leitor terá contato com desigualdades básicas, além de algumas sequências eséries. No �m do capítulo o leitor poderá encontrar uma breve passagem pelasequações funcionais, um tema muito incomum na vida do estudante. Essa seçãojuntamente com o próximo capítulo talvez sejam as mais estranhas para umestudante não iniciado.

O capítulo 6, teoria do números, é talvez o que �que mais fora da caixinha.Para um leitor que venha de uma formação tradicional, as descobertas serãoin�nitas. Geralmente é aqui que as pessoas se deslumbram com as olimpíadas.Neste capítulo o leitor poderá se aprofundar um pouco mais em divisibilidade,aprender a usar com mais precisão omdc (confesso que na escola eu não entendiapor que raios se estuda mdc, entendi graças as olimpíadas). Além disso se iniciaaqui o contato com congruência, e a partir daí o capítulo irá girar em tornodessa idéia, apresentando alguns teoremas famosos tais como Euler-Fermat eChinês dos Restos.

O capítulo 7 é sobre geometria. Esse capítulo começa suave, mas aos poucosvai se tornando incomum (em relação a escola normal). Aqui algumas idéiasconhecidas são exploradas de modos diferentes e algumas ideias menos comunssurgirão. Um dilema que tive enquanto escrevia foi "incluir ou não semelhançade triângulos?". Cheguei a conclusão de que como esse assunto é bem saturadonão será difícil o leitor encontrar outras fontes por onde estudar e decidi focarem outras partes desse conteúdo, isso se o leitor já não souber mais do que eusobre esse assunto. Mesmo deixando de lado vários aspectos básicos esse é ocapítulo mais robusto do livro.

Todos os problemas do livro que tem um ano antes do enunciado são daOBM nível 3, de 1998 a 2016. Os que se encontram nos capítulos 2 a 6são da primeira fase e os que se encontram no capítulo 7 são da segundafase. As soluções desses problemas podem ser encontradas na página da pró-pria OBM (http://www.obm.org.br/como-se-preparar/provas-e-gabaritos/), in-clusive foi de lá que extraí os problemas. Os outros problemas tem outrasnaturezas, podem ter sidos extraídos de livros, sites, outras provas, são os pas-sos necessários para se provar algum teorema ou exercício mais difícil, ou sãoversões simpli�cadas de outros problemas.

Page 8: OBM por assunto: Os primeiros passos olímpicos · tenha muito a acrescentar na sua vida pois o foco do treinamento é justamente para alcançar esse nível de habilidade. ... Esse

8 SUMÁRIO

Alguns "spin-o�s"desse livro seram publicado sem uma regularidade espe-cí�ca no blog https://matematizandoal.wordpress.com/. Coisas como listas deexercícios, sugestão de leitores e erratas. Quero fazer desse livro um livro vivo,que se transforma via contribuição dos leitores. Todas as sugestões serão devi-damente creditadas. Este livro é só o começo da jornada!

0.3 Agradecimentos

Começo agradecendo a meus mentores de olimpíadas por ordem de surgimentona minha vida acadêmica. Primeiro a Elivaldo Bezerra, o meu grande professore primeiro incentivador que me apresentou as olimpíadas e que me mostrou oprincípio das gavetas e me mostrou que matemática era incrivelmente diver-tida. A Iedo Bezerra, que me ensinou a equação fundamental da Trigonometria.Krerley Oliveria, que me mostrou o que era congruência. A Davi Lima que meensinou o indução. Carlos Argolo que me ensinou a derivar. A Diogo Santosque me ensinou o que era uma série de Taylor. A José Luís Vicente, por terme ensinado técnicas de resolver recorrência. A Hiago Rocha e a Bruno Berlepor compartilharem comigo seus aprendizados de resolver problemas olímpicos.Sem essas pessoas minha relação com as olimpíadas, se existisse, certamente nãoteria sido tão grati�cante.

Agradeço a todos os que foram meus alunos em aulas de olimpíadas, aprenderpara ensinar foi tão grati�cante que descobri ali que meu desejo é ser professor.Entre os quais os estudantes Gabriel Oliveira, Kevyn Lacerda, Leonardo Mari-nho e Rafael Nascimento, que além de ótimos alunos revisaram o livro e deramdiversas contribuições na sua construção.

Gostaria de agradecer ao professor da UFAL Isnaldo Barbosa pela revisão�nal do livro e suas incontáveis sugestões de como melhorar o texto e torná-lomais agradável para o leitor, além de contribuir com diversas introduções deseções.

Finalmente agradeço a você leitor, pois uma obra não é nada sem seu público.E aproveito para convidar o leitor a me enviar sugestões de correção aos possíveiserros que por descuido inteiramente meu possa ter �cado ainda no livro. Aosdispostos o email é [email protected].

Pessoal, sou profundamente agradecido pelos incentivos e pelos ensinamen-tos, que obviamente são muito maiores do que os dos nossos primeiros contatos,que escolhi para vos agradecer neste texto. Esse livro é um profundo agradeci-mento pela vossa boa vontade.

Page 9: OBM por assunto: Os primeiros passos olímpicos · tenha muito a acrescentar na sua vida pois o foco do treinamento é justamente para alcançar esse nível de habilidade. ... Esse

Capítulo 1

Problemas básicos

Talvez você esteja ansioso para começar a resolver problemas olímpicos masnão sabe quais problemas atacar primeiro! Se esse era um dos seus problemasentão ele acabou! Neste primeiro capítulo apresentamos três seções nas quaisos conhecimentos exigidos são mínimos e podem ser discutidos sem exigir muitatécnica (não estou a�rmando que eles são triviais!). Esperamos que neste capí-tulo o leitor perceba o quão importante é saber interpretar bem um texto paraquem deseja aventurar-se em problemas de olimpíadas de Matemática.

"Estou pronto!"- Bob Esponja.

1.1 Conjuntos

2005. Um professor de Inglês dá aula particular para uma classe de 9 alunos,dos quais pelo menos um é brasileiro. Se o professor escolher 4 alunos para fazeruma apresentação, terá no grupo pelo menos dois alunos de mesma nacionali-dade; se escolher 5 alunos, terá no máximo três alunos de mesma nacionalidade.Quantos brasileiros existem na classe?

2008. Uma grande empresa possui 84 funcionários e sabe-se que cada funci-onário fala pelo menos uma das línguas entre Português e Inglês. Além disso,20% dos que falam Português também falam Inglês e 80% dos que falam Inglêstambém falam Português. Quantos funcionários falam as duas línguas?

2015. Sejam A e B dois conjuntos disjuntos tais que n(A) = 5 e n(B) = 7,em que n(X) é a quantidade de elementos do conjunto X. Quantos subconjun-tos não-vazios C de A ∪B são tais que n(A ∩ C) = n(B ∩ C)?

2016. O conjunto X está contido em {1, 2, 3, · · · , 2016} e tem a seguintepropriedade: para todos x, y ∈ X com x < y, y + 1 é múltiplo de x. Qual é aquantidade máxima de elementos que X pode ter?

9

Page 10: OBM por assunto: Os primeiros passos olímpicos · tenha muito a acrescentar na sua vida pois o foco do treinamento é justamente para alcançar esse nível de habilidade. ... Esse

10 CAPÍTULO 1. PROBLEMAS BÁSICOS

1.2 Extremos

1999. Em um hotel há 100 pessoas: 30 comem porco, 60 comem galinha e 80comem alface. Qual é o maior número possível de pessoas que não comem ne-nhum desses dois tipos de carne?

2005. Em um ano, no máximo quantos meses têm cinco domingos?

2013. Considere cinco pontos no plano. Qual é a quantidade máxima detriângulos equiláteros com vértices em três desses cinco pontos?

1.3 Lógica

1998. Um crime é cometido por uma pessoa e há quatro suspeitos: André,Eduardo, Rafael e João. Interrogados, eles fazem as seguintes declarações:

- André: Eduardo é o culpado.- Eduardo: João é o culpado.- Rafael: Eu não sou culpado.- João: Eduardo mente quando diz que eu sou culpado.Sabendo que apenas um dos quatro disse a verdade, quem é o culpado?

1998. Se x homens fazem x embrulhos em x segundos, em quantos segundosy homens farão y embrulhos?

1998. A respeito da resposta de um problema, Maurício, Paulo, Eduardo eCarlos �zeram as seguintes a�rmações:

- Maurício: É maior que 5.- Paulo: É menor que 10.- Eduardo: É um número primo.- Carlos: É maior que 12.Entre as a�rmações acima, quantas, no máximo, podem ser verdadeiras?

2000. Há três cartas viradas sobre uma mesa. Sabe-se que em cada umadelas está escrito um número inteiro positivo. São dadas a Carlos, Samuel eTomás as seguintes informações:

i) todos os números escritos nas cartas são diferentes;ii) a soma dos números é 13;iii) os números estão em ordem crescente, da esquerda para a direita.Primeiro, Carlos olha o número na carta da esquerda e diz: Não tenho infor-

mações su�cientes para determinar os outros dois números. Em seguida, Tomásolha o número na carta da direita e diz: Não tenho informações su�cientes paradeterminar os outros dois números. Por �m, Samuel olha o número na carta domeio e diz: Não tenho informações su�cientes para determinar os outros doisnúmeros. Sabendo que cada um deles sabe que os outros dois são inteligentes eescuta os comentários dos outros, qual é o número da carta do meio?

2000. Juliano colou uma bandeirinha cinza em cada engrenagem, comomostra a �gura abaixo:

Page 11: OBM por assunto: Os primeiros passos olímpicos · tenha muito a acrescentar na sua vida pois o foco do treinamento é justamente para alcançar esse nível de habilidade. ... Esse

1.3. LÓGICA 11

As engrenagens são iguais e quando a engrenagem da esquerda girou umpouco, a sua bandeirinha �cou na posição indicada com a bandeirinha brancapontilhada. Nesta condição, podemos a�rmar que a posição da bandeirinha naengrenagem da direita é:

2001. Cinco animais A,B,C,D, e E, são cães ou são lobos. Cães semprecontam a verdade e lobos sempre mentem. A diz que B é um cão. B diz que Cé um lobo. C diz que D é um lobo. D diz que B e E são animais de espéciesdiferentes. E diz que A é um cão. Quantos lobos há entre os cinco animais?

2001. Somente uma das �guras a seguir representa a plani�cação de umcubo na qual está destacada a sua interseção com um plano. Qual?

2002. Qual é a quantidade total de letras de todas as respostas incorretasdesta questão?

A) Quarenta e oito.B) Quarenta e nove.C) Cinqüenta.D) Cinqüenta e um.E) Cinqüenta e quatro.

2003. A sequência "22" descreve a si mesma, pois ela é formada por exa-tamente dois 2. Analogamente, a seqüência "31123315" descreve a si mesma,pois é formada por exatamente três 1, um 2, três 3 e um 5. Qual das seguintesseqüências não descreve a si mesma?

A) 21322316

Page 12: OBM por assunto: Os primeiros passos olímpicos · tenha muito a acrescentar na sua vida pois o foco do treinamento é justamente para alcançar esse nível de habilidade. ... Esse

12 CAPÍTULO 1. PROBLEMAS BÁSICOS

B) 31123318C) 3122331719D) 2132332415E) 41322324151618

2003. Você está em um país estrangeiro, a LUCIÂNIA, e não conhece oidioma, o LUCIANÊS, mas sabe que as palavras "BAK"e "KAB" signi�camsim e não, porém não sabe qual é qual. Você encontra uma pessoa que entendeportuguês e pergunta "KAB signi�ca sim", a pessoa responde "KAB". Pode-sededuzir que:

A) KAB signi�ca sim.B) KAB signi�ca não.C) A pessoa que respondeu mentiu.D) A pessoa que respondeu disse a verdade.E) Não é possível determinar sem um dicionário LUCIANÊS-PORTUGUÊS

2003. Carlinhos pensa num número ímpar positivo menor do que 100. Pe-drinho se dispõe a descobrir que número é esse fazendo a seguinte pergunta,quantas vezes forem necessárias: O número que você pensou é maior, menor ouigual a x? Note que x é um número que Pedrinho escolhe. Quantas perguntasdesse tipo Pedrinho poderá ter que fazer até descobrir o número pensado porCarlinhos?

2004. O produto dos números que aparecem nas alternativas incorretas dessaquestão é um cubo perfeito. Assinale a alternativa correta.

A) 4 B) 8 C) 18 D) 54 E) 192

2004. O dono de uma loja empilhou vários blocos medindo 0, 8m× 0, 8m×0, 8m no canto da loja e encostados numa parede de vidro que dá para a rua,conforme mostra a �gura abaixo. Quantos blocos no máximo, uma pessoa de1, 80m de altura que está do lado de fora da loja pode enxergar?

Obs. Consideramos que uma pessoa pode enxergar uma caixa se conseguever uma pequena região de área positiva de sua superfície.

2004. Sobre uma mesa estão três caixas e três objetos, cada um em umacaixa diferente: uma moeda, um grampo e uma borracha. Sabe-se que

A caixa verde está à esquerda da caixa azul;A moeda está à esquerda da borracha;A caixa vermelha está à direita do grampo;A borracha está à direita da caixa vermelha.

Page 13: OBM por assunto: Os primeiros passos olímpicos · tenha muito a acrescentar na sua vida pois o foco do treinamento é justamente para alcançar esse nível de habilidade. ... Esse

1.3. LÓGICA 13

Em que caixa está a moeda?A) Na caixa vermelha .B) Na caixa verde.C) Na caixa azul.D) As informações fornecidas são insu�cientes para se dar uma resposta.E) As informações fornecidas são contraditórias.

2004. Um feirante vende batatas e, para pesar, utiliza uma balança de doispratos, um peso de 1kg, um peso de 3kg e um peso de 10kg. Considere a seguintea�rmação: "Este feirante consegue pesar (com uma pesagem) n quilogramas debatatas". Quantos valores positivos de n tornam essa a�rmação verdadeira,supondo que ele pode colocar pesos nos dois pratos?

A) 7 B) 10 C) 12 D)13 E)14

2005. Um bloco de dimensões 1 × 2 × 3 é colocado sobre um tabuleiro8 × 8, como mostra a �gura, com a face X, de dimensões 1 × 2, virada parabaixo. Giramos o bloco em torno de uma de suas arestas de modo que a faceY �que virada para baixo. Em seguida, giramos novamente o bloco, mas destavez de modo que a face Z �que virada para baixo. Giramos o bloco mais trêsvezes, fazendo com que as faces X,Y e Z �quem viradas para baixo, nessaordem. Quantos quadradinhos diferentes do tabuleiro estiveram em contatocom o bloco?

2005. Esmeralda adora os números triangulares (ou seja, os números 1, 3, 6,10, 15, 21, 28 · · · ), tanto que mudou de lugar os números 1, 2, 3, · · · , 11 do relógiode parede do seu quarto de modo que a soma de cada par de números vizinhosé um número triangular. Ela deixou o 12 no seu lugar original. Que númeroocupa o lugar que era do 6 no relógio original?

2005. Entre treze números reais não nulos há mais números positivos do que

negativos. Dentre os13× 12

2= 78 produtos de dois dos treze números, 22 são

negativos. Quantos números dentre os treze números dados são negativos?

2007. O desenho abaixo mostra um dado comum cujas somas das pontuaçõesem faces opostas é sempre igual a 7. Ele é colocado em uma mesa horizontalcom a face "1" voltada para Leste. O dado é, então, movido quatro vezes. Ummovimento consiste em uma rotação de 90◦ em relação a uma aresta. Depoisdo primeiro movimento a face em contato com a mesa passa a ser a "1", depoisa "2", então a "3" e, �nalmente, a face "5". Para que sentido está voltada aface "1" após esta seqüência de movimentos?

Page 14: OBM por assunto: Os primeiros passos olímpicos · tenha muito a acrescentar na sua vida pois o foco do treinamento é justamente para alcançar esse nível de habilidade. ... Esse

14 CAPÍTULO 1. PROBLEMAS BÁSICOS

2007. Os números 72, 8, 24, 10, 5, 45, 36, 15 são agrupados em duplas de modoque o produto de cada dupla é o mesmo. Qual número �ca com o 10?

2007. Uma avenida possui 100 prédios numerados de 1 a 100, onde prédioscom numeração par se situam do lado direito da rua e prédios com numeraçãoímpar se situam no lado esquerdo. A quantidade de andares de cada prédioé igual à soma dos algarismos do número correspondente ao prédio. Assim,podemos a�rmar que:

A) A quantidade de prédios com mais de 10 andares é maior do lado direitoda rua.

B) A quantidade de prédios com menos de 5 andares é maior do lado direitoda rua.

C) Pelo menos metade dos prédios possui 10 ou mais andares.D) Em ambos os lados da rua há a mesma quantidade de prédios com exatos

8 andares.E) Pelo menos 25% dos prédios possui menos de 5 andares.

2008. A primeira fase da OBM se realiza no dia 14 de junho, um sábado doano bissexto 2008. Daqui a quantos anos o dia 14 de junho será novamente nosábado?

2008. Tenho um cubo de madeira, com três faces vermelhas e três facesazuis. O cubo é cortado em 3 × 3 × 3 = 27 cubos menores. Quantos destescubos menores têm, pelo menos, uma face vermelha e outra azul?

2008. Abaixo temos um quadrado mágico multiplicativo, onde o produtodos números em cada linha, coluna e diagonal é o mesmo e igual ao número dequatro dígitos ABCD, onde cada letra representa um dígito e cada casa contémum número inteiro. Se AC representa o número de dois dígitos no centro doquadrado, quanto vale a soma A+B + C +D?

2008. Rafael tem 10 cartões. Cada um tem escrito um dos números

3, 8, 13, 18, 23, 28, 33, 48, 53, 68,

Page 15: OBM por assunto: Os primeiros passos olímpicos · tenha muito a acrescentar na sua vida pois o foco do treinamento é justamente para alcançar esse nível de habilidade. ... Esse

1.3. LÓGICA 15

e todos os dez números aparecem. Qual o menor número de cartões que Rafaelpode escolher de modo que a soma dos números nos cartões escolhidos seja exa-tamente 100?

2009. Sempre que Agilulfo volta para casa depois da escola com uma adver-tência, se sua mãe está em casa, ela o coloca de castigo. Sabendo-se que ontemà tarde Agilulfo não foi colocado de castigo, qual das seguintes a�rmações écertamente verdadeira?

A) Agilulfo recebeu advertência ontem.B) Agilulfo não recebeu advertência ontem.C) Ontem à tarde a sua mãe estava em casa.D) Ontem à tarde a sua mãe não estava em casa.E) Nenhuma das a�rmações acima é certamente verdadeira.

2009. O professor Piraldo aplicou uma prova de 6 questões para 18 estudan-tes. Cada questão vale 0 ou 1 ponto; não há pontuações parciais. Após a prova,Piraldo elaborou uma tabela para organizar as notas, em que cada linha repre-senta um estudante e cada coluna representa uma questão. Piraldo constatouque cada estudante acertou exatamente 4 questões e que cada questão teve amesma quantidade m de acertos. Qual é o valor de m?

2009. Alguns cubos foram empilhados formando um bloco. As �guras aolado representam a vista da esquerda e da frente desse bloco. Olhando o blocode cima, qual das �guras a seguir não pode ser vista?

2010. Quatro amigos, Arnaldo, Bernaldo, Cernaldo e Dernaldo estão jogandocartas. São 20 cartas diferentes, cada carta tem uma entre 4 cores (azul, amarelo,verde, vermelho) e um número de 1 a 5. Cada amigo recebe cinco cartas, demodo que todas as cartas são distribuídas. Eles fazem as seguintes a�rmações:

Arnaldo: "Eu tenho quatro cartas com o mesmo número."Bernaldo: "Eu tenho as cinco cartas vermelhas."Cernaldo: "As minhas cinco cartas são de cores que começam com a letra

V."Dernaldo: "Eu tenho três cartas de um número e duas cartas de outro

número."

Page 16: OBM por assunto: Os primeiros passos olímpicos · tenha muito a acrescentar na sua vida pois o foco do treinamento é justamente para alcançar esse nível de habilidade. ... Esse

16 CAPÍTULO 1. PROBLEMAS BÁSICOS

Sabe-se que somente uma das a�rmações é falsa. Quem fez essa a�rmação?

2010. A �gura a seguir foi recortada em cartolina e depois dobrada paraformar um icosaedro. As faces em branco foram numeradas de modo que aoredor de cada vértice (pontas do sólido) apareçam os números de 1 a 5. Qualnúmero está na face com a interrogação?

2010. As quatro faces de um dado são triângulos equiláteros, numerados de1 a 4, como no desenho. Colando-se dois dados iguais, fazemos coincidir duasfaces, com o mesmo número ou não. Qual dos números a seguir não pode ser asoma dos números das faces visíveis?

A) 12 B) 14 C) 17 D) 18 E)19

2011. A �gura abaixo representa um mapa de estradas. Os números escritosnas setas indicam quanto de pedágio um viajante deve pagar ao passar pelaestrada. Todas as estradas são de mão única, como indicam as setas. Qual ovalor mínimo de pedágio pago por um viajante que sai da cidade A e chega nacidade B?

2011. Um cubo de madeira, pintado de vermelho, foi serrado em 27 cubosmenores iguais e as faces desses cubos ainda não pintadas o foram de branco.Qual é a razão entre a área da superfície total pintada em vermelho e a área da

Page 17: OBM por assunto: Os primeiros passos olímpicos · tenha muito a acrescentar na sua vida pois o foco do treinamento é justamente para alcançar esse nível de habilidade. ... Esse

1.3. LÓGICA 17

superfície total pintada de branco?

2011. Numa classe de 36 alunos, todos têm alturas diferentes. O mais baixodos meninos é mais alto do que cinco meninas, o segundo menino mais baixo émais alto do que seis meninas, o terceiro menino mais baixo é mais alto do quesete meninas e assim por diante, observando-se que o mais alto dos meninos émais alto do que todas as meninas. Quantas meninas há nessa classe?

2011. Qual é o produto da quantidade de vogais pela quantidade de con-soantes na alternativa correta? (Não considere as letras A, B, C, D, E dasalternativas na contagem.)

A) Vinte e quatro.B) Trinta e seis.C) Quarenta e dois.D) Quarenta e oito.E) Cinquenta e seis.

2011. Esmeralda tem 11 notas de dois reais, Rosa tem 7 notas de cinco reaise Nelly tem 3 notas de dez reais. Qual é o menor número possível do total denotas que devem mudar de mãos de forma que todas as moças �quem com amesma quantia?

2011. No Planeta Nérdia, existem três espécies de nerds: ET-nerds, UFO-nerds e OVNI-nerds. A primeira mente quando chove e diz a verdade quandonão chove; a segunda sempre mente; a terceira sempre diz a verdade. Certodia Bruberson, um nerd muito camarada, se encontra com quatro nerds. E elesfalam:

X: "Hoje está chovendo."Y: "O nerd que acabou de falar está mentindo."Z: "Hoje não está chovendo."W: "O primeiro nerd mentiu ou eu sou um ET-nerds."Com quantos ET-nerds Bruberson falou no máximo?

2012. Quantas vogais têm a resposta correta desse problema? Não conte aletra A ou E das alternativas A e E.

A) Seis B) Cinco C) Quatro D) Três E) Duas

2012. Anos bissextos têm um dia a mais, 29 de fevereiro, que os demaisanos e ocorrem a cada 4 anos. Esmeralda nasceu no dia 29 de fevereiro, em umdomingo. Sabendo que 29 de fevereiro de 2012 caiu em uma quarta-feira, emqual ano Esmeralda pode ter nascido?

A) 1972 B) 1976 C) 1980 D) 1984 E) 1988

2013. Os algarismos desse ano, 2013 são 0, 1, 2 e 3, obviamente não nessaordem. Daqui a quantos anos ocorrerá o próximo ano cujos algarismos serão0, 1, 2 e 3 novamente?

2013. Joana preenche completamente um quadriculado retangular escre-vendo os números de 1 a 2013, sendo um número para cada quadrado. Ela co-meça no canto superior esquerdo e preenche a primeira coluna, depois preenche

Page 18: OBM por assunto: Os primeiros passos olímpicos · tenha muito a acrescentar na sua vida pois o foco do treinamento é justamente para alcançar esse nível de habilidade. ... Esse

18 CAPÍTULO 1. PROBLEMAS BÁSICOS

a segunda coluna, de cima para baixo e continua, da mesma forma, preenchendoa terceira coluna, a quarta, etc., até chegar à última coluna e terminar no cantoinferior direito. Se o número 50 está na segunda coluna e o número 100 naquarta coluna, em qual coluna estará escrito o número 1000?

2014. Roraima Jones, um arqueólogo aventureiro, ao fugir de uma cavernase depara com quatro portas, numeradas de 1 até 4, e quatro mensagens. Asmensagens dizem:

Mensagem 1: "As portas 1 e 2 são seguras."Mensagem 2: "Exatamente duas entre as portas 1, 2 e 3 são seguras."Mensagem 3: "A porta 1 é segura."Mensagem 4: "A porta 3 é segura."Roraima Jones é um estudioso e, por isso, sabe que exatamente uma das

mensagens é mentira e exatamente uma das portas não é segura (ativaria umaarmadilha). Qual porta Roraima Jonas pode garantir que é segura?

2014. Quantas alternativas contêm uma palavra com mais letras que a pa-lavra na alternativa correta?

A) Duas B) Três C) Quatro D) Cinco E) Seis

2014. Em uma calculadora muito simples não é possível apertar dois dígitossem apertar algumas das operações +,−,× ou ÷ entre as apertadas dos dígitos.Ao apertar o dígito a calculadora faz a operação imediatamente. A calculadoracomeça com o 0 no visor e a primeira apertada tem que ser uma operação. Ouseja, primeiro se aperta uma operação, depois um dígito, depois uma operação,e assim por diante. Por exemplo, um jeito para aparecer 29 no visor é apertar +e depois 7, fazendo 0 + 7 aparecer no visor; em seguida, apertar × e 5, passandoa ter 7×5 no visor, e concluir apertando − e 6 tendo como resultado 35−6 = 29.Assim, é possível obter 29 com 6 apertadas de botão. Pedro quer que apareça onúmero 100 no visor. Qual o número mínimo de apertadas, contando operaçõese dígitos, que Pedro tem que fazer na calculadora?

2014. Um quadrado mágico multiplicativo é um quadrado n × n com nú-meros inteiros positivos distintos cujos produtos de números na mesma linha,coluna ou diagonal são iguais. Por exemplo, temos o seguinte quadrado mágicomultiplicativo:

Qual é menor valor possível do número no centro de um quadrado mágicomultiplicativo 3× 3?

2015. Qual é a soma dos quadrados das quantidades de vogais e consoantesda resposta correta? Não conte as letras A, B, C, D, E das alternativas.

Page 19: OBM por assunto: Os primeiros passos olímpicos · tenha muito a acrescentar na sua vida pois o foco do treinamento é justamente para alcançar esse nível de habilidade. ... Esse

1.3. LÓGICA 19

A) Vinte e seis.B) Setenta e três.C) Oitenta e cinco.D) Noventa e sete.E) Cento e dezesseis.

2015. Dizemos que dois anos coincidem se têm a mesma quantidade de diase os dias da semana de todos os seus dias coincidem. O ano de 2015 coincidecom 2009; qual é o próximo ano que coincide com 2015? Lembre-se de que osanos múltiplos de 4 no século XXI (com exceção de 2100) são bissextos e têm366 dias; os demais anos têm 365 dias.

2015. Violeta quer numerar de 1 a 9 os quadrados do tabuleiro abaixo, demodo que a soma de dois números em quadrados vizinhos (quadrados com ladoscomuns) seja um número ímpar. Além disso, ela quer que a soma dos númerosescritos nos quadrados cinza seja a maior soma possível. Qual é a soma dosnúmeros escritos nos quadrados brancos?

2016. Lena quer completar as casas do tabuleiro 3×3 ao lado, usando as mes-mas letras já escritas, de modo que casas vizinhas (casas com um lado comum)não tenham a mesma letra. Que letra poderá ser escrita na casa cinzenta?

2016. Num país imaginário vivem somente duas espécies de pessoas: oshonestos, que sempre dizem a verdade e os mentirosos, que só dizem mentira.Numa �la de 2016 pessoas da ilha, o primeiro da �la diz que todos atrás dele sãomentirosos e todas as demais pessoas da �la dizem que a pessoa imediatamenteà sua frente é mentirosa. Quantas pessoas mentirosas estão nessa �la?

2016. O ano de 2016 é sabadoso, pois há cinco meses com cinco sábados.Qual será o próximo ano sabadoso?

Page 20: OBM por assunto: Os primeiros passos olímpicos · tenha muito a acrescentar na sua vida pois o foco do treinamento é justamente para alcançar esse nível de habilidade. ... Esse

20 CAPÍTULO 1. PROBLEMAS BÁSICOS

1.3.1 Calculo direto

2001. Papa-Léguas participou de uma corrida (junto com o Ligeirinho e oFlash), que consistia em dar 100 voltas em um circuito. Como sempre, o Coiotequeria pegar o Papa-Léguas e colocou um monte de alpiste no meio da pista.É claro que o Coiote não conseguiu pegar o Papa-Léguas, mas ele fez com quea velocidade média dele na primeira volta fosse de apenas 200km/h. Sabendodisso, a velocidade média do Papa-Léguas na corrida:

A) Não ultrapassa 200km/h.B) Não ultrapassa 250km/h, mas pode ultrapassar 200km/h.C) Não ultrapassa 2000km/h, mas pode ultrapassar 250km/h.D) Não ultrapassa 20000km/h, mas pode ultrapassar os 2000km/h.E) Pode ultrapassar 20000km/h.

2002. A linha poligonal AB é desenhada mantendo-se sempre o mesmopadrão mostrado na �gura. Quanto vale o seu comprimento total ?

2005. Uma loja de sabonetes realiza uma promoção com o anúncio "Compreum e leve outro pela metade do preço". Outra promoção que a loja poderiafazer oferecendo o mesmo desconto percentual é

A) "Leve dois e pague um".B) "Leve três e pague um".C) "Leve três e pague dois".D) "Leve quatro e pague três".E) "Leve cinco e pague quatro".

2005. Um piloto percorreu três trechos de um rali, de extensões 240km, 300kme 400km, respectivamente. As velocidades médias nos três trechos foram 40km/h,75km/h e 80km/h, mas não necessariamente nessa ordem. Podemos garantirque o tempo total em horas gasto pelo piloto nos três trechos é:

A) menor ou igual a 13 horas.B) maior ou igual a 13 horas e menor ou igual a 16 horas.C) maior ou igual a 14 horas e menor ou igual a 17 horas.D) maior ou igual a 15 horas e menor ou igual a 18 horas.E) maior ou igual a 18 horas.

2005. O número (2 +√

2)3(3−√

2)4 + (2−√

2)3(3 +√

2)4 éA) inteiro ímparB) inteiro parC) racional não inteiroD) irracional positivoE) irracional negativo

Page 21: OBM por assunto: Os primeiros passos olímpicos · tenha muito a acrescentar na sua vida pois o foco do treinamento é justamente para alcançar esse nível de habilidade. ... Esse

1.3. LÓGICA 21

2005. Um relógio, com ponteiros de horas, minutos e segundos, faz plimtoda vez que um ponteiro ultrapassa outro no mostrador. Qual é o número deplins registrados em um certo dia no período entre as 12 horas e 1 segundo e as23 horas, 59 minutos e 59 segundos?

2005. Platina é um metal muito raro, mais raro até do que ouro. Suadensidade é 21, 45g/cm3. Suponha que a produção mundial de platina foi decerca de 110 toneladas em cada um dos últimos 50 anos e desprezível antes disso.Assinale a alternativa com o objeto cujo volume é mais próximo do volume deplatina produzido no mundo em toda a história.

A) Uma caixa de sapatos .B) Uma piscina.C) Um edifício de dez andares .D) O monte Pascoal.E) A Lua.

2010. Qual o número que obtemos dividindo o número 4(42) por 44?

2011. Por conta de uma erupção de um vulcão, 10% dos voos de um aero-porto foram cancelados. Dos voos restantes, 20% foram cancelados pela chuva.Que porcentagem do total de voos deste aeroporto foram cancelados?

2013. Se Joana comprar hoje um computador cujo preço é 2000 reais, elaconseguirá um desconto de 5%. Se ela deixar para amanhã, irá conseguir omesmo desconto de 5%, mas o preço do computador irá aumentar 5%. Se elaesperar, o que acontecerá?

A) Nada, pois pagará a mesma quantia.B) Ela perderá 100 reais.C) Ela ganhará 105 reais.D) Ela perderá 95 reais.E) Ela perderá 105 reais.

2014. Em Portugal, o dia 4 de outubro de 1582 foi o último dia do calendá-rio juliano, que foi substituído pelo calendário adotado atualmente, o calendáriogregoriano. O dia seguinte foi de�nido como 15 de outubro de 1582, ou seja, nãohouve os dias 5 a 14 de outubro de 1582. A única diferença entre os calendáriosé que, no calendário juliano, todos os anos múltiplos de 4 eram bissextos; nocalendário gregoriano, os anos que são múltiplos de 100, mas não de 400, nãosão bissextos. Assim, 1900 seria um ano bissexto no calendário juliano, masnão no calendário gregoriano. Que dia seria hoje, 3 de junho de 2014, se nãotivéssemos mudado de calendário?

Page 22: OBM por assunto: Os primeiros passos olímpicos · tenha muito a acrescentar na sua vida pois o foco do treinamento é justamente para alcançar esse nível de habilidade. ... Esse

22 CAPÍTULO 1. PROBLEMAS BÁSICOS

Page 23: OBM por assunto: Os primeiros passos olímpicos · tenha muito a acrescentar na sua vida pois o foco do treinamento é justamente para alcançar esse nível de habilidade. ... Esse

Capítulo 2

Indução

Esse capítulo é um capítulo curinga. Para entender algumas demonstrações doscapítulos seguintes é necessário saber indução, mas as demonstrações não sãoessenciais para o aprendizado das idéias principais do texto, portando, se vocêpreferir, pode deixar esse capítulo para o futuro. Particularmente eu acho queindução é um dos primeiros divisores de águas na matemática (apesar de muitosdiscordarem), aprender indução vai te transformar num estudante novo, é comoo morfador dos Power Rangers. Com respeito aos exercícios apresentados nesselivro, a não ser pelos teoremas deixados como exercício, não é necessário saberindução.

Nas seções a sguir explicaremos do que se trata e quais são as utilidade doPrincípio da Indução (ou Princípio da Indução Finita) e Princípio da InduçãoForte e é claro, deixaremos uns exercícios pra que você se divirta bastante.

"É hora de morphar!"- Ranger Vermelho.

2.1 O Princípio da Indução

Seja P (n) uma propriedade do número natural n, por exemplo:

n pode ser fatorado em números primos;

a soma dos naturais menores ou iguais a n é n(n+1)2 ;

se você elevar n ao cubo e subtrair n o resultado será um múltiplo de 6.

Uma maneira de provar que P (n) é verdadeira para todo natural n ≥ n0, éutilizar o chamado Princípio da Indução Finita (PIF), que é um dos axiomasque caracterizam os números naturais.

Princípio da Indução: Dada uma propriedade dependendo do natural n,se

1. (Base de indução) P (n0) é verdadeira para algum natural n0,

23

Page 24: OBM por assunto: Os primeiros passos olímpicos · tenha muito a acrescentar na sua vida pois o foco do treinamento é justamente para alcançar esse nível de habilidade. ... Esse

24 CAPÍTULO 2. INDUÇÃO

2. (Passo indutivo) P (k) ser verdadeira para algum número natural k ≥ n0

implica P (k + 1) também ser verdadeira,

então P (n) é verdadeira para todo n natural.

Na base da indução veri�camos que a propriedade é válida para um valorinicial n = n0. O passo indutivo consiste em mostrar como utlizar a validadeda propriedade para um caso n (chamado hipótese de indução) para provar avalidade da mesma propriedade para o inteiro seguinte n+1. Uma vez veri�cadosa base e o passso indutivo, temos uma cadeia de implicaçoes

P (n0) é verdadeira⇒ P (n0 + 1) é verdadeira

⇒ P (n0 + 2) é verdadeira

⇒ P (n0 + 3) é verdadeira

· · ·

de modo que P (n) é verdadeira para todo n ≥ n0.

2.1.1 Exemplos

1. Mostre que para todo natural n, vale 1 + 2 + 3 + · · ·+ n =n(n+ 1)

2.

Solução:

Base da indução (k0 = 1): 1 =1 · 2

2

Hipótese de indução: 1 + 2 + 3 + · · ·+ k =k(k + 1)

2.

Passo de indução:

1 + 2 + 3 + · · ·+ k + (k + 1) =k(k + 1)

2+ k + 1

= (k + 1)(k

2+ 1)

=(k + 1)(k + 2)

2.

2. Mostre a Desigualdade de Bernoulli, ou seja, que se x ≥ −1, então paratodo natural n, vale

(1 + x)n ≥ 1 + nx.

Solução:

Base da indução (k0 = 1): (1 + x)1 = 1 + x ≥ 1 + 1 · x;Hipótese de indução: (1 + x)k ≥ 1 + kx;

Passo de indução:

(1 + x)k+1 = (1 + x)k(1 + x)

≥ (1 + kx)(1 + x)

= 1 + (k + 1)x+ kx2

≥ 1 + (k + 1)x [pois kx2 ≥ 0].

Page 25: OBM por assunto: Os primeiros passos olímpicos · tenha muito a acrescentar na sua vida pois o foco do treinamento é justamente para alcançar esse nível de habilidade. ... Esse

2.1. O PRINCÍPIO DA INDUÇÃO 25

3. Mostre que n3 − n é múltiplo de 6, para todo natural n.

Solução:

Base da indução (k0 = 1): 13 − 1 = 0 e 6|0;Hipótese de indução: k3 − k é múltiplo de 6;

Passo de indução:

(k + 1)3 − (k + 1) = k3 + 3k2 + 3k + 1− k − 1

= k3 − k + 3k(k + 1)

Como k3−k é múltiplo de 6, para concluir precisamos mostrar que 3k(k+1) também é múltiplo de 6, que é mesmo que mostrar que k(k + 1) émúltiplo de 2. Você pode fazer isso por indução, ou então pode observarque dados dois números inteiros consecutivos, um é necessariamente par,logo o produto é par, o que conclui a demonstração.

Pausa para notação:Quando se deseja somar uma sequência de números x1, · · · , xn para os quais

se conhece um padrão usa-se o símbolo de somatório:

xk + xk+1 + · · ·+ xn =

n∑j=k

xj .

O "j = k" que �ca abaixo no símbolo signi�ca que o primeiro termo da sequênciaé o xk, os índices vão variando até chegar no índice que se encontra acima nosomatório, neste caso "n". Por exemplo

7∑k=3

xk = x3 + x4 + x5 + x6 + x7.

Os termos xk podem ser dados por uma fórmula. Por exemplo, se xj = j2 ey` = 3` então podemos escrever

6∑j=3

xj =

6∑j=3

j2 = 32 + 42 + 52 + 62

e

13∑`=10

y` =

13∑`=10

3` = 3 · 10 + 3 · 11 + 3 · 12 + 3 · 13.

Por último você pode por uma regra qualquer na parte inferior do somatório,por exemplo ∑

124<j<126

j = 124 + 125 + 125

ou ainda ∑s<10:s é primo

s = 2 + 3 + 5 + 7.

Exercício: Calcule10∑i=1

4.

Dica: Perceba que xi é constante igual a 4, nesse caso.

Page 26: OBM por assunto: Os primeiros passos olímpicos · tenha muito a acrescentar na sua vida pois o foco do treinamento é justamente para alcançar esse nível de habilidade. ... Esse

26 CAPÍTULO 2. INDUÇÃO

2.1.2 Exercícios

1. Mostre que:

a)n∑j=1

j2 = 12 + 22 + 32 + · · ·+ n2 =n(n+ 1)(2n+ 1)

6.

b)n∑j=1

j3 = 13 + 23 + 33 + · · ·+ n3 =

[n(n+ 1)

2

]2

.

c) Identidade de Lagrange:

∑1≤j<k≤n

(ajbk − akbj)2 =

(n∑k=1

a2k

)(n∑k=1

bk

)−

(n∑k=1

akbk

)2

.

2. A sequência de Fibonacci é de�nida por F1 = F2 = 1 e Fn+1 = Fn+Fn−1.Mostre que

a) F1 + F2 + · · ·+ Fn = Fn+2 − 1;

b) Fn+1 · Fn−1 − F 2n = (−1)2.

3. Demonstre que

1− 1

2+

1

3− 1

4+ · · ·+ 1

199− 1

200=

1

101+

1

102+ · · ·+ 1

200.

4. Todos os números da forma 1007, 10017, 100117, · · · são múltiplos de 53.

5. Mostre que 2n < n!, para todo n ≥ 4.

6. Prove que 1 +1√2

+1√3

+ · · ·+ 1√n< 2√n.

7. Iremos construir a torre√

2√

2√

2...

, a0 = 1 e an+1 = (√

2)an , mostre quepara todo n, an+1 > an e an < 2.

8. Mostre que se α é tal que α+1

α∈ Z, então αn+

1

αn∈ Z para todo n ∈ N.

9. (A Torre de Hanói.) São dados três suportes A,B e C. No suporte A estãoencaixados n discos cujos diâmetros, de baixo para cima, estão em ordemestritamente decrescente. Mostre que é possível, com 2n − 1 movimentos,transferir todos os discos para o suporte B, usando o suporte C comoauxiliar, de modo que jamais, durante a operação, um disco maior �quesobre um disco menor.

2.2 Possíveis equívocos

Algumas confusões podem surgir quando você se depara com problemas deindução, por isso é necessário certa cautela. Nem sempre o que parece ser umpadrão num problema é de fato um padrão. Algumas vezes cometemos um erroao tomarmos uma premissa errada e algumas vezes ao seguirmos uma estratégiaerrada. A seguir veremos dois casos aos quais o leitor deve �car atento.

Page 27: OBM por assunto: Os primeiros passos olímpicos · tenha muito a acrescentar na sua vida pois o foco do treinamento é justamente para alcançar esse nível de habilidade. ... Esse

2.2. POSSÍVEIS EQUÍVOCOS 27

2.2.1 Cavalos: Indução que deu errado

Provaremos que todos os cavalos são rosas. De fato, considere a sentença aberta:P (n): Num conjunto com n cavalos, todos têm a mesma cor.Note que P (1) é obviamente verdadeira. Agora, suponha o resultado válido

para conjuntos contendo n cavalos. Considere um conjunto

C = {C1, C2, ..., Cn, Cn+1}

com n+ 1 cavalos. Decompomos o conjunto C numa união de dois conjuntos:

C = C′ ∪ C′′ = {C1, · · · , Cn} ∪ {C2, · · · , Cn+1}.

Pela hipótese indutiva, segue-se que os cavalos em C′ têm mesma cor, ocorrendoo mesmo para os cavalos em C′′. Como C2 ∈ C′ ∩ C′′ segue-se que os cavalos deC′ têm a mesma cor dos cavalos de C′′, permitindo assim concluir que todos oscavalos em C têm a mesma cor.

Assim, a nossa "demonstração" por indução está terminada, provando queP (n) é verdadeira para todo n ∈ N.

No carnaval eu vi um cavalo rosa, logo todo cavalo é rosa.Esse problema foi inventado pelo matemático húngaro George Pólya (1887-

1985). Claro que há um erro na demonstração, a�nal existem cavalos que nãosão rosas no mundo. Qual é o erro?

Reza a lenda, que esse problema foi posto numa prova e um aluno respon-deu "Quem disse que a matemática se aplica aos cavalos?". Não sei, mas apesarde eu gostar de lembrar dessa resposta, te deixarei com outra possibilidade noexercício abaixo.

Exercício: Veri�que se P (1) ser verdadeira implica em P (2) ser verdadeira,ou seja, veri�que se o passo de indução vale de 1 para 2.

2.2.2 "Pela lógica" não é prova

É comum encontrarmos em provas de concursos ou em "testes de inteligência"questões que dão alguns termos de uma sequência e pedem o seguinte. No en-tanto, esse tipo de questão não tem formalidade matemática, como veremos noexemplo a seguir.

Exemplo: Considere a sequência a1 = 1, a2 = 4, a3 = 9, e a4 = 16. Quantovale a5?

É natural �car tentado a responder a5 = 25 mas não há uma razão formalpara que essa seja a resposta. Se a sequência fosse an = n2, teríamos a5 = 25, ede fato a sequência an = n2 tem os termos inicias iguais aos dados no enunciado,no entanto a sequência an = (n− 1)(n− 2)(n− 3)(n− 4) + n2 também tem, enesse caso a5 = 49. Na verdade, para qualquer valor m é possível encontrar umpolinômio p(x) tal que p(1) = 1, · · · , p(4) = 4 e p(5) = m.

Caso esse tópico tenha despertado o seu interesse investigue sobre o polinô-mio interpolador de Lagrange.

Page 28: OBM por assunto: Os primeiros passos olímpicos · tenha muito a acrescentar na sua vida pois o foco do treinamento é justamente para alcançar esse nível de habilidade. ... Esse

28 CAPÍTULO 2. INDUÇÃO

Essa é uma das razões que torna indução importante, sempre que você en-contrar um padrão, não basta perceber que aquele padrão acontece, é necessárioprovar que vale para todo n, e aí, geralmente você vai usar indução. Apesar detudo isso, não subestime sua percepção de padrão, só tenha em mente que tera intuição de uma resposta não é su�ciente (mesmo podendo ser muito útil).

Exercício: Encontre polinômios que façam a5 = 1 e a5 = 73, no exercícioanterior.

Exercício: Considere a sequência a1 = 1, a2 = 2, a3 = 3, e a4 = 4. Quantovale a5?

- Pela Lógica é 5, não é?- Quem é essa tal de Lógica?

2.3 Indução Forte

Princípio da Indução (versão forte) Dada P (n) uma propriedade que de-pende do natural n se

1. (Base de indução) P (n0), P (n0 + 1), · · · , P (n0 + m) é verdadeira paraalgum natural n0 e um natural m,

2. (Passo indutivo) para todo k > n0 + m, P (j) ser verdadeira para todo jcom n0 ≤ j < k implica que P (k) é verdadeira,

então P (n) é verdadeira para todo n ≥ n0.

A diferença entre os enunciados de indução e indução forte é que no casode indução olhamos para o número imediatamente antes de n e tiramos umaconclusão sobre n, enquanto que na indução forte não olhamos somente para ocara imediatamente antes de n, mas para todos os números indo do caso inicialaté n− 1.

Exemplo: Mostre que todo inteiro n ≥ 2 pode ser escrito como produto denúmeros primos.

Solução: Observe que n0 = 2 é o produto de um único primo, que é elemesmo. Agora suponha que tenhamos provado que o resultado valha para todonúmero menor do que k. Se k for primo, ele é, assim como o 2 o produto deum único número primo, se k for composto, então existem a, b < k tais quek = a ·b e pela hipótese de indução a e b podem ser decompostos como produtosde números primos, donde segue que k é o produto dos primos que compõe apelos primos que compõem b.

Observe que aqui, não seria su�ciente usar que k − 1 é decomposto porprimos, porque não daria para tirar nenhuma conclusão sobre k a partir dek − 1, assim há uma substância em considerar a indução forte.

Page 29: OBM por assunto: Os primeiros passos olímpicos · tenha muito a acrescentar na sua vida pois o foco do treinamento é justamente para alcançar esse nível de habilidade. ... Esse

2.3. INDUÇÃO FORTE 29

2.3.1 Exercícios

1. O postulado de Bertrand é um teorema (é sério, o nome é por questãohistórica) que diz que dado um natural n, entre n e 2n existe um númeroprimo. Use o postulado de Bertrand para mostrar que todo número podeser escrito como soma de dois números primos.

2. Mostre que todo número pode ser escrito como soma de dois númerosdistintos da sequência de Fibonacci.

Page 30: OBM por assunto: Os primeiros passos olímpicos · tenha muito a acrescentar na sua vida pois o foco do treinamento é justamente para alcançar esse nível de habilidade. ... Esse

30 CAPÍTULO 2. INDUÇÃO

Page 31: OBM por assunto: Os primeiros passos olímpicos · tenha muito a acrescentar na sua vida pois o foco do treinamento é justamente para alcançar esse nível de habilidade. ... Esse

Capítulo 3

Combinatória e Probabilidade

Grosseiramente podemos dizer que Combinatória é o estudo dos conjuntos �nitosequanto que probabilidade estuda a relação entre um conjunto especí�co como todo. Neste capítulo, iremos ver algumas técnicas básicas que permitirão aoleitor ter um gostinho da área. Em geral, as técnicas básicas de combinatória sãoum tanto rasas por si só e elas encontram profundidade nas variadas aplicações;são técnicas simples de entender mas que requerem a criatividade na hora deaplicar.

"O queijo suíço grande tem mais buracos,então quanto mais queijo menos queijo!"

- Autor desconhecido.

3.1 Exercícios de aquecimento

Para garantir seu bom desempenho neste "jogo"sugerimos um leve aquecimento!A ideia é oferecer um "ritmo"para minimizar a entrada neste campo de proble-mas não tão explorados no ensino tradicional. Pronto? Então vamos lá!

1999. Uma bola de futebol é feita com 32 peças de couro. 12 delas são pen-tágonos regulares e as outras 20 são hexágonos também regulares. Os lados dospentágonos são iguais aos dos hexágonos de forma que possam ser costurados.Cada costura une dois lados de duas dessas peças. Quantas são as costurasfeitas na fabricação de uma bola de futebol?

2000. Quantos são os retângulos quetêm os pontos A e B como vértices, ecujos vértices estão entre os pontos deinterseção das 9 retas horizontais comas 9 retas verticais da �gura abaixo?

31

Page 32: OBM por assunto: Os primeiros passos olímpicos · tenha muito a acrescentar na sua vida pois o foco do treinamento é justamente para alcançar esse nível de habilidade. ... Esse

32 CAPÍTULO 3. COMBINATÓRIA E PROBABILIDADE

2004. O arranjo a seguir, composto por32 hexágonos, foi montado com varetas,todas com comprimento igual ao lado dohexágono. Quantas varetas, no mínimo,são necessárias para montar o arranjo?

2006. Quantos resultados diferentes podemos obter somando pares de nú-meros distintos do conjunto {1, 2, · · · , 2006}?

2016. Na igualdade D×O×Z×ED×O×I×S = S × E × I × S, letras iguais representam

algarismos iguais e letras diferentes representam algarismos diferentes. Se osalgarismos são todos diferentes de zero, quantos valores diferentes o produtoS × E × I × S pode ter?

3.2 Princípios aditivo e multiplicativo

O princípio aditivo consiste em dizer que a união de dois conjuntos disjuntos(isto é, sem elemento comum) possui uma quantidade de elementos igual a somados números de elementos em cada conjunto. O princípio multiplicativo con-siste em dizer que a quantidade de elementos no produto cartesiano de doisconjuntos é igual ao produto do número de elementos do primeiro pelo númerode elementos do segundo. Em símbolos, temos respectivamente

Princípio aditivo

A ∩B = ∅ ⇒ |A ∪B| = |A|+ |B|

Princípio multiplicativo

|A×B| = |A| · |B|.Esses princípios são extremamente simples de serem enunciados, e como a

maioria das ideias em combinatória, o grande truque é saber onde e como usá-los. A seguir serão exibidos alguns exemplos para que se torne mais intuitivo ojeito de usar essas ideias.

Exemplo: De quantas maneiras podemos ordenar 4 galinhas em uma �la?

Solução: Existem 4 lugares relativos na �la. Vamos escolher uma das ga-linhas para pôr no primeiro lugar. Como temos 4 galinhas, temos 4 opções.No momento em que escolhemos uma das galinhas, �camos com uma galinha amenos, para pôr nos outros lugares, assim para o próximo lugar, temos apenas3 galinhas. Repetindo o argumento temos no próximo lugar 2 opções e para oúltimo lugar apenas 1 opção. Assim o número possível de �las é 4×3×2×1 = 24possíveis maneiras. Difícil mesmo é as galinhas �carem na �la!

Exemplo: Quantos são os números de três dígitos que não possuem dígitosadjacentes iguais?

Page 33: OBM por assunto: Os primeiros passos olímpicos · tenha muito a acrescentar na sua vida pois o foco do treinamento é justamente para alcançar esse nível de habilidade. ... Esse

3.2. PRINCÍPIOS ADITIVO E MULTIPLICATIVO 33

Solução: Queremos construir um número de três dígitos, digamos abc. Paraa escolha de a temos 9 opções, que não pode ser o 0, em seguida temos 9 opçõespara b que não pode ser igual a mas pode ser o 0, e temos 9 opções para c, quenão pode ser igual a b. Portanto temos 9× 9× 9 = 729 tais números.

Exemplo: De quantos modos podemos formar uma roda de ciranda com 5camundongos?

Solução: Temos 5 opções de camundongos para colocarmos inicialmente.Ao colocarmos o primeiro, temos 4 opções para ser aquele que �cará a es-querda do primeiro, 3 para �car a esquerda do segundo, dois a esquerda doterceiro, e 1 a esquerda do quarto (e a direita do primeiro). Mas observe quese �zermos uma roda pondo os camundongos C1C2C3C4C5 nessa ordem, po-demos obter a mesma roda pondo C5C1C2C3C4 , ou C4C5C1C2C3, por exem-plo. Vamos analizar o caso geral, isto é, dada qualquer roda com n camun-dongos, podemos obtê-la escolhendo qualquer camundongo Cj pra ser o pri-meiro, e depois seguir a con�guração até o �m Cj+1Cj+2 · · ·Cn, e colacarmosos camundongos do começo em seguida, ou seja, C1C2 · · ·Cj−1. Em outraspalavras, para cada j a roda CjCj+1Cj+2 · · ·CnC1C2 · · ·Cj−1 é igual a rodaC1C2 · · ·Cj−1CjCj+1 · · ·Cn. Assim, cada roda é contada uma vez para cadacamundongo que pode ser posto no início, ou seja, cada roda é contada n vezes.No caso de n = 5, cada roda é contada 5 vezes, e devemos então dividir por5 o número obtido pela nossa contagem. Assim, temos 5×4×3×2×1

5 = 24 rodasde ciranda formadas por 5 camundongos, ou (n − 1) · (n − 2) · · · 3 · 2 · 1 rodasformadas com n camundongos. Esse exercício �ca muito mais difícil trocandocamundos por tiranossauros, porque os tiranossauros têm os braços curtos e nãoconseguem brincar de ciranda.

Exemplo: Para pintar a bandeira abaixo estão disponíveis as seis coresdadas, sendo que regiões adjacentes devem ser pintadas de cores diferentes.

(a) Qual é o número mínimo de cores a serem usadas?(b) De quantos modos a bandeira pode ser pintada?

Solução:a) Claramente não podemos pintar toda a bandeira com uma única cor se-

guindo a regra da adjacência. Se tivéssemos apenas duas cores, como a faixalateral, a faixa de cima e a faixa do meio são todas adjacentes entre si, nenhumpar formado por essas faixas podem ter a mesma cor, logo, precisamos de pelomenos três cores. Com três cores, digamos verde, amarelo e azul, podemos pin-tar a faixa do meio de azul, a lateral de verde e a superior e inferior de amarelo.Logo a resposta é 3.

Page 34: OBM por assunto: Os primeiros passos olímpicos · tenha muito a acrescentar na sua vida pois o foco do treinamento é justamente para alcançar esse nível de habilidade. ... Esse

34 CAPÍTULO 3. COMBINATÓRIA E PROBABILIDADE

b) O que esse problema tem a ensinar é que procurar um jeito simples decontar é importante (às vezes não existe um jeito muito simples, mas nesse casoexiste). Vamos começar dando uma solução mais trabalhosa e em seguida da-remos uma solução mais simples.

Solução trabalhosa: Se começarmos pintando a faixa de cima, temos 6opções, para a faixa lateral, temos 5 opções, pois não podemos usar a cor decima, para a faixa de baixo, temos 5 opções, pois não podemos usar a cor dafaixa lateral, e para o meio temos 3 opções, pois não podemos usar nenhumadas três cores já utilizadas, certo? Errado, para a do meio, depende, pois seusarmos em baixo a mesma cor que usamos em cima, temos 4 opções, e se usar-mos em baixo uma cor diferente temos 3 opções. Assim temos que considerardois casos: usamos em baixo a cor igual a cor de cima, nessa caso temos (cima,lateral, baixo, meio) 6×5×1×4 = 120 possibilidades, e se usarmos cores diferen-tes temos 6×5×4,×3 = 360, agora usando o princípio aditivo temos 480 opções.

Solução simples: Vamos começar pintando a faixa do meio, temos 6 pos-sibilidades. Agora pinte a faixa lateral, temos 5 possibilidades, agora as faixasde cima e de baixo podem ter qualquer cor diferente das duas faixas pintadas,ou seja, temos 4 possibilidades para cada uma. Logo o número de formas depintar é 6× 5× 4,×4 = 480.

OBM

1998. Quantos números inteiros entre 10 e 1000 possuem seus dígitos em ordemestritamente crescente? (Por exemplo, 47 e 126 são números deste tipo; 52 e566 não).

1999. Um gafanhoto pula exatamente 1 metro. Ele está em um ponto Ade uma reta, só pula sobre ela, e deseja atingir um ponto B dessa mesma retaque está a 5 metros de distância de A com exatamente 9 pulos. De quantasmaneiras ele pode fazer isso?

2000. Quantos números inteiros e positivos menores do que 1.000.000 exis-tem cujos cubos terminam em 1?

2000. Quantos números de três algarismos (que não começam com 0) pos-suem um algarismo que é a média aritmética dos outros dois?

2001. Os números inteiros positivos de 1 a 1000 são escritos lado a lado,em ordem crescente, formando a seqüência 123456789101112131415 · · · 9991000.Nesta seqüência, quantas vezes aparece o grupo "89"?

2001. Um círculo é dividido, por 2n+1 raios, em 2n+1 setores congruentes.Qual é o número máximo de regiões do círculo determinadas por estes raios epor uma reta?

2001. Quantos números de dois algarismos não são primos nem múltiplos de2, 3 ou 5?

Page 35: OBM por assunto: Os primeiros passos olímpicos · tenha muito a acrescentar na sua vida pois o foco do treinamento é justamente para alcançar esse nível de habilidade. ... Esse

3.2. PRINCÍPIOS ADITIVO E MULTIPLICATIVO 35

2004. Esmeralda escreveu (corretamente!) todos os números de 1 a 999, umatrás do outro:

12345678910111213 · · · 997998999.

Quantas vezes aparece o agrupamento "21", nesta ordem?

2004. Esmeralda, a digitadora, tentou digitar um número de seis algarismos,mas os dois algarismos 1 não apareceram (a tecla devia estar com defeito). Oque apareceu foi 2004. Quantos são os números de seis algarismos que ela podeter tentado digitar?

2005. Quantos números entre 10 e 13000, quando lidos da esquerda para adireita, são formados por dígitos consecutivos e em ordem crescente? Exempli-�cando, 456 é um desses números, mas 7890 não.

2006. De quantas maneiras podemos colocar, em cada espaço abaixo, umentre os algarismos 4, 5, 6, 7, 8, 9 de modo que todos os seis algarismos apareçame formem, em cada membro, números de dois algarismos que satisfazem a dupladesigualdade?

__ > __ > __

2006. O piso de um quarto tem forma de um quadrado de lado 4m. Dequantas maneiras podemos cobrir totalmente o quarto com oito tapetes iguaisde dimensões 1m e 2m?

2007. Um número de quatro dígitos é dito peroba se possui pelo menos doisdígitos vizinhos com a mesma paridade. Quantos números perobas existem?

2007. Dizemos que uma palavra Q é quase-anagrama de outra palavra Pquando Q pode ser obtida retirando-se uma letra de P e trocando a ordem dasletras restantes, resultando em uma palavra com uma letra a menos do queP . Um quase-anagrama pode ter sentido em algum idioma ou não. Por exem-plo, RARO, RACR e ARCO são quase-anagramas de CARRO. Quantos são osquase-anagramas da palavra BACANA que começam com A?

2008. Um número de quatro dígitos é dito paladino se é múltiplo de 9 enenhum de seus dígitos é nulo. Quantos números paladinos existem?

2008. Considere 10 pessoas, todas de alturas diferentes, as quais devem �carem �la de tal modo que, a partir da pessoa mais alta, as alturas devem decrescerpara ambos os lados da �la (se a pessoa mais alta for a primeira ou a última da�la, todas as pessoas a partir dela devem estar em ordem decrescente de altura).Obedecendo essas condições, de quantos modos essas pessoas podem �car em�la?

2008. Soninha tem muitos cartões, todos com o mesmo desenho em umadas faces. Ela vai usar cinco cores diferentes (verde, amarelo, azul, vermelho elaranja) para pintar cada uma das cinco partes do desenho, cada parte com umacor diferente, de modo que não haja dois cartões pintados da mesma forma. Na�gura abaixo, por exemplo, os cartões são iguais, pois um deles pode ser girado

Page 36: OBM por assunto: Os primeiros passos olímpicos · tenha muito a acrescentar na sua vida pois o foco do treinamento é justamente para alcançar esse nível de habilidade. ... Esse

36 CAPÍTULO 3. COMBINATÓRIA E PROBABILIDADE

para se obter o outro. Quantos cartões diferentes Soninha conseguirá produzir?

2009. Um número natural A de três algarismos detona um número naturalB de três algarismos se cada algarismo de A é maior do que o algarismo corres-pondente de B. Por exemplo, 876 detona 345; porém, 651 não detona 542 pois1 < 2. Quantos números de três algarismos detonam 314?

2009. A �gura ao lado é o mapa de um bairro: ospontos A,B,C e D são as casas e os segmentossão as ruas. De quantas casas é possível fazerum caminho que passa exatamente uma vez porcada uma das ruas? É permitido passar mais deuma vez por uma mesma casa.

2009. Esmeralda tem cinco livros sobre heráldica em uma estante. No �nalde semana, ela limpou a estante e, ao recolocar os livros, colocou dois deles nolugar onde estavam antes e os demais em lugares diferentes de onde estavam.De quantas maneiras ela pode ter feito isso?

2010. De quantas maneiras é possível desenhar a �gura a seguir sem tiraro lápis do papel (ou qualquer outro utensílio, se você preferir!) começando deP e sem passar sobre o mesmo ponto mais de uma vez, com exceção do pontocomum aos três triângulos?

2012. De quantas maneiras podemos cobrir um tabuleiro 4 × 4 com umquadrado de lado 2 e quatro peças idênticas no formato de L que ocupam três

Page 37: OBM por assunto: Os primeiros passos olímpicos · tenha muito a acrescentar na sua vida pois o foco do treinamento é justamente para alcançar esse nível de habilidade. ... Esse

3.2. PRINCÍPIOS ADITIVO E MULTIPLICATIVO 37

casinhas do tabuleiro? O tabuleiro não pode ser rotacionado, ou seja, as duaspossibilidades a seguir devem ser consideradas distintas.

2012. Um painel luminoso é formado por 10 círculos grandes. Dentro decada círculo há quatro lâmpadas: uma amarela, uma verde, uma vermelha euma azul. De quantos modos podemos acender o painel de modo que pelo me-nos uma lâmpada de cada cor �que acesa? Cada círculo pode ter de zero aquatro lâmpadas acesas, ou seja, é permitido duas lâmpadas acesas no mesmocírculo.

2012. Numa festa de criança, o palhaço Macaxeira irá distribuir 21 balaspara 5 crianças que participam de uma brincadeira. Macaxeira quer fazer adistribuição satisfazendo às seguintes condições:

1) Cada criança deve receber pelo menos uma bala;2) Cada criança recebe um número diferente de balas;3) O número de balas é feito em ordem decrescente, de acordo com sua altura

(a menor criança recebe mais balas e a maior recebe menos balas).Supondo que todas as crianças tem alturas diferentes, de quantos modos ele

pode fazer essa distribuição?

2013. De quantos modos podemos distribuir 10 bolas brancas e 8 bolas ver-melhas em cinco caixas iguais, de modo que em cada caixa haja pelo menos umabola e que em cada caixa haja um número diferente de bolas brancas?

2013. Quantos números de quatro algarismos distintos não têm 1 nas uni-dades, nem 2 nas dezenas, nem 3 nas centenas e nem 4 nos milhares?

A) Menos de 1000.B) Mais de 1000 e menos de 2000.C) Mais de 2000 e menos de 3000.D) Mais de 3000 e menos de 4000.E) Mais de 4000.

2014. O jogo de triminó simpli�cado é composto por peças na forma detriângulo em que cada um dos vértices possui um número de 0 a 5. Sabe-seque para qualquer peça do triminó simpli�cado quando se coloca o menor dosnúmeros no vértice superior os números estão em ordem crescente no sentidohorário, ou seja, a peça faz parte do triminó simpli�cado quando X ≤ Y ≤ Z.Existem quantas peças em um jogo de triminó simpli�cado?

Page 38: OBM por assunto: Os primeiros passos olímpicos · tenha muito a acrescentar na sua vida pois o foco do treinamento é justamente para alcançar esse nível de habilidade. ... Esse

38 CAPÍTULO 3. COMBINATÓRIA E PROBABILIDADE

2014. Bitonho está jogando em seu celular o Super Paciência, cujo objetivoé preencher um tabuleiro com zeros e uns de modo que dois números vizinhosiguais em uma mesma linha impedem que se preencha também com númerosiguais as casas correspondentes da outra linha. De quantas maneiras Bitonhopode preencher um tabuleiro de Super Paciência?

2015. Jonas gosta de observar os relógios digitais espalhados por sua cidadeque informam a hora e a data. Por coincidência ele viu que hoje é dia 12/06 enaquele momento marcava 12:06, ou seja, data e hora são formados com os mes-mos números! Ele �cou encucado com a coincidência e chamou o momento (datae hora) de encucado. Ele pensou que também seria interessante se a hora fosseformada com os mesmos números mas na ordem trocada, por exemplo, no dia21/06 às 06:21, então chamou esse momento de encucado reverso. Considerandoque 2015 não é um ano bissexto, desde 01/01/2015 às 00:00 até 31/12/2015 às23:59 quantos momentos são encucados ou encucados reversos?

2016. De quantas maneiras podemos escolher n casas de um tabuleiro n×n,n > 3, sem escolher duas casas na mesma linha ou na mesma coluna, sabendoque os quatro cantos do tabuleiro não podem ser escolhidos?

3.3 Números binomiais

Os números binomiais são números de extrema importância em combinatória,de�ní-los torna a vida bem mais simples no início, e mais tarde suas aplicaçõese conexões se tornam cada vez mais interessantes. Seu nome vem da relaçãoque ele tem com expressão das potências de binômios como veremos em breve.

3.3.1 Fatorial

Antes de tudo, um conceito bastante importante é o de fatorial. Dado umnúmero natural n escrevemos n! (n fatorial) para representar o produto dosinteiros positivos menores ou iguais a n, isto é, n! = n · (n−1) · (n−2) · · · 3 ·2 ·1.Por razões práticas, de�nimos 0! = 1.

Esta de�nição é muito prática para representar valores em problemas decombinatória.

Exercícios

1. Mostre que (n+ 1)! = (n+ 1) · n!.

2. Calcule 1!, 2!, 3!, 4!, 5!, 6!, 7!, 8! (para quem faz olimpíadas é útil lembrarpelo menos até 6!).

Observe que

Page 39: OBM por assunto: Os primeiros passos olímpicos · tenha muito a acrescentar na sua vida pois o foco do treinamento é justamente para alcançar esse nível de habilidade. ... Esse

3.3. NÚMEROS BINOMIAIS 39

i) n · (n− 1) · · · (k + 2) · (k + 1) =n!

k!.

ii) n · (n− 1) · · · (n− k + 2) · (n− k + 1) =n!

(n− k)!.

Exemplo: Quantos subconjuntos com 5 elementos o conjunto {1, 2, · · · , 9}possui?

Solução: Vamos dividir inicialmente o conjunto em 5 posição distintas.Como temos 9 números temos 9 possibilidade para escolher o número a serposto na primeira posição, 8 para a segunda, e repete o argumento até 5 paraa quinta. No entanto a ordem que os elementos se apresentam no conjunto nãoafetam o conjunto, por exemplo {1, 2, 3, 4, 5} = {3, 4, 1, 5, 2}. Assim devemosdescobrir quantas vezes repetimos a contagem de cada conjunto. Observe quedados 5 elementos temos 5! ordenações possíveis para esse quinteto, logo cadaconjunto foi contado 5! vezes, assim a quantidade de subconjunto que queremosé 9·8·7·6·5

5! = 9!4!·5! .

Exercício: Quantos subconjuntos com k elementos o conjunto {1, 2, · · · , n}possui?

OBM

2003. Cinco amigos, Arnaldo, Bernaldo, Cernaldo, Dernaldo e Ernaldo, devemformar uma �la com outras 30 pessoas. De quantas maneiras podemos formaresta �la de modo que Arnaldo �que na frente de seus 4 amigos. (Obs.: Osamigos não precisam �car em posições consecutivas.)

3.3.2 Número Binomial

De�nição 1 Dados n ∈ N e k ∈ N ∪ {0}, com k ≤ n, de�nimos o número(nk

),

lê-se binomial de n e k, como sendo(n

k

)=

n!

(n− k)! · k!.

É costume chamar o número(nk

)de n escolhe k, pelo fato de ser a quantidade

de subconjuntos que podemos construir escolhendo k elementos de um conjuntoque possui n elementos.

Proposição 1 (Propriedades dos números binomiais) Valem as seguintesidentidades

i)

(n+ 1

k + 1

)=n+ 1

k + 1

(n

k

);

ii)

(n+ 1

k

)=

n+ 1

n− k + 1

(n

k

);

iii)

(n

k

)=

(n

n− k

);

iv)

(n+ 1

k + 1

)=

(n

k + 1

)+

(n

k

)[Relação de Stifel].

Page 40: OBM por assunto: Os primeiros passos olímpicos · tenha muito a acrescentar na sua vida pois o foco do treinamento é justamente para alcançar esse nível de habilidade. ... Esse

40 CAPÍTULO 3. COMBINATÓRIA E PROBABILIDADE

Prova: A prova dessa proposição é trivial algebricamente pela de�nição viafatorial (Faça).

A seguir daremos provas combinatoriais dos itens iii) e iv), ilustrando ummétodo conhecido como contagem dupla, que consiste em contar uma coisa dedois modos diferentes para se obter uma identidade.

iii) Suponha que queremos montar um conjunto A com k elementos de{1, 2, ..., n}, podemos fazer isso escolhendo k elementos para por em A, temos(nk

)maneiras de fazer isso, ou escolhendo n− k elementos para �carem fora de

A, temos(n

n−k)maneiras de fazer isso. Assim

(nk

)=(n

n−k).

iv) Suponha que queiramos fazer um subconjunto A com k+ 1 elementos doconjunto {1, 2, ..., n + 1}. Sabemos que podemos fazer isso de

(n+1k+1

)maneiras.

Mas observe que para fazer isso, podemos por no conjunto A o elemento 'n+ 1'e em seguida escolher outros k elementos para por em A, temos

(nk

)maneiras de

fazer isso, ou podemos não por o elemento 'n+1', e montar o conjunto A apenascom elementos de {1, 2, ..., n},temos

(nk+1

)maneiras de fazer isso. Assim, segue

que(n+1k+1

)=(nk+1

)+(nk

). �

Exercícios

1. Use contagem dupla para provar que se 0 ≤ p ≤ n, entãop∑k=0

(n

k

)(n

n− k

)=

(2n

p

).

2. Prove que se p é primo, então o produto de p números inteiros positivosconsecutivos é múltiplo de p!.

3. Use um argumento combinatório para provar que os seguintes númerossão inteiros

a) (2n)!2n ;

b) (3n)!2n·3n ;

c) (n!)!(n!)(n−1)! .

2016. Janaína escreveu uma lista de 10 números inteiros positivos no quadro-negro e obteve todas as somas possíveis de dois desses números, veri�cando quetodas eram diferentes. O número de somas pares que ela obteve era igual a qua-tro vezes o número de somas ímpares. Qual é a maior quantidade de númerospares que poderia haver na lista de Janaína?

3.3.3 Justi�cando o nome

A seguir veremos porque esses números são chamados de números binomiais.Considere um binômio, ou seja, uma expressão do tipo x+y. O que acontece

se elevarmos essa expressão a n-ésima potência?

(x+ y)n = (x+ y)(x+ y)...(x+ y)

Page 41: OBM por assunto: Os primeiros passos olímpicos · tenha muito a acrescentar na sua vida pois o foco do treinamento é justamente para alcançar esse nível de habilidade. ... Esse

3.4. PRINCÍPIO DAS CASAS DOS POMBOS 41

Qual é o coe�ciente de xn nessa expressão? Para que obtenhamos o termoxkyn−k devemos multiplicar x em k dos parênteses, ou seja, temos n parêntesespara escolher em quais multiplicamos x (e os que não escolhermos multiplicare-mos y). Assim o coe�ciente de xkyn−k em (x+ y)n é

(nk

)donde

(x+ y)n =

(n

0

)xny0 +

(n

1

)xn−1y1 + ...+

(n

n− 1

)x1yn−1

(n

n

)x0yn.

3.4 Princípio das casas dos pombos

O princípio das casas dos pombos (PCP) é um teorema muito famoso e muitosimples que diz que se temos mais pombos do que casas de pombos (ou gaiolas),e vamos pôr todos os pombos dentro das casas, alguma casa terá mais que umpombo.

O princípio das casas dos pombos é também conhecido como princípio dasgavetas, é só trocar pombo por bolas e casa de pombo por gaveta. Esse teoremafoi inicialmente evidenciado pelo matemático alemão Johann Dirichlet, e porisso é às vezes mencionado com Princípio de Dirichlet.

Quero deixar claro que o princípio também vale para gatos ou peixes, masno caso de peixes sugiros que os ponha em aquários.

Teorema 1 (Prinicípio das casas dos pombos) Se pusermos k+ 1 pombosem k gaiolas, alguma gaiola terá pelo menos 2 pombos.

Em nome da liberdade dos animais não ponha pombos em gaiolas.

Prova:Suponha que tenhamos uma organização em que em cada gaiola pomos me-

nos que dois pombos, ou seja, uma quantidade menor ou igual a 1. Como temosk gaiolas teremos no máximo k pombos no total, mas isso contratria o fato determos k + 1 pombos.�

Exemplos triviais:Dadas treze pessoas, existem duas que tem o mesmo signo.Dadas oito vacas, duas nasceram no mesmo dia da semana.Dados três cães, dois deles tem o mesmo sexo.

O princípio das casas do pombos parece ridiculamente simples, e talvez oseja, mas isso não diminui o poder que ele possui. As aplicações são surpreen-dentes. A seguir veremos alguns exemplos para que possamos ver porque essespombos merecem ser respeitados.

Exemplo: Não importa como escolhemos 1010 elementos (distintos) do con-junto {1, 2, · · · , 2018} haverão 2 números consecutivos entre os escolhidos.

Solução: O pulo do gato no PCP está em escolher corretamente quemé casa e quem é pombo. Aqui queremos escolher 1010 elementos, que seria1009+1 precisamos então de 1009 casas, como queremos elementos consecutivos,

Page 42: OBM por assunto: Os primeiros passos olímpicos · tenha muito a acrescentar na sua vida pois o foco do treinamento é justamente para alcançar esse nível de habilidade. ... Esse

42 CAPÍTULO 3. COMBINATÓRIA E PROBABILIDADE

podemos considerar as casas sendo os conjuntos

{1, 2}, {3, 4}, {5, 6}, · · · , {2017, 2018}.

Observe que temos 1009 casas de pombos, para escolher 1009+1 pombos, entãodevemos escolher dois pombos de uma mesma casa. Assim teremos garantidodois pombos.

Algumas confusões surgem na cabeça das pessoas nesse problema. Umadelas: Eu não poderia ter os números 4 e 5, por exemplo, eles são consecutivose não estão na mesma casa. A resposta pra essa pergunta é "sim", mas o fatode você ter números consecutivos que não estão na mesma casinha no sentidoacima, não impede que existam TAMBÉM pombos na mesma casinha. Umacoisa não exclui a outra.

Outra confusão muito comum é a pessoa tentar alguns casos, ver que não dánaqueles casos e concluir que não é possível. Isto está ERRADO, a não ser quevocê olhe todos os exemplos de conjuntos, e como você já sabe calcular, existem(

20181010

)conjuntos. Acho que em uma prova você não vai conseguir escrever todos

antes do tempo acabar, a menos que seja uma prova muito longa.

Exemplo: Mostre que todo número natural possui um múltiplo formadoapenas por dígitos 1 e 0 na sua representação decimal.

Solução: Seja n um número qualquer �xo. Dado qualquer M existem npossibilidade de restos na divisão de M por n, são eles 0, 1, · · · , n − 1. Agoraconsidere a sequência

1, 11, 111, · · · , 111 · · · 11︸ ︷︷ ︸n1′s

Se algum desse números for múltiplo de n, então encontramos o número dese-jado. Se nenhum deles for múltiplo de n, então os possíveis valores dos restosda divisão de cada um deles por n pertencerá a {1, · · · , n− 1}, e como temos nnúmeros para n− 1 possíveis restos, dois deles devem ter o mesmo resto, sejam

an+ r = 111 · · · 11︸ ︷︷ ︸j1′s

< 111 · · · 11︸ ︷︷ ︸k1′s

= bn+ r

esses dois números. É claro que a diferença entre eles vai ser (b − a)n que émultiplo de n mas

111 · · · 11︸ ︷︷ ︸k 1′s

− 111 · · · 11︸ ︷︷ ︸j 1′s

= 111 · · · 11︸ ︷︷ ︸k−j 1′s

000 · · · 00︸ ︷︷ ︸j 0′s

.

Portanto 111 · · · 11︸ ︷︷ ︸k−j1′s

000 · · · 00︸ ︷︷ ︸j0′s

é um múltiplo de n formado só por 0's e 1's.

O princípio pode ser escrito de maneiras mais gerais, uma delas é

Teorema 2 (Prinicípio das casas dos pombos, segunda versão) Se puser-mos nk + 1 pombos em n gaiolas, alguma gaiola terá pelo menos k + 1 pombos.

Page 43: OBM por assunto: Os primeiros passos olímpicos · tenha muito a acrescentar na sua vida pois o foco do treinamento é justamente para alcançar esse nível de habilidade. ... Esse

3.4. PRINCÍPIO DAS CASAS DOS POMBOS 43

Prova: Exercício. Use um raciocínio similar ao da primeira versão.

Exemplo: (BORM) Seja S um conjunto de 2007 pontos no plano, tal queem qualquer subconjunto de três pontos de S, existem no mínimo dois pontoscom distância menor do que 1. Prove que existe um subconjunto de 1004 pontosde S que podem ser cobertos por um círculo de raio 1.

Solução: Sejam A e B dois pontos de S com máxima distância.Primeiro caso: Se AB ≤ 1, então para qualquer X ∈ S, temos AX ≤ 1 e um

disco com centro em A e raio 1 cobre todos os 2007 pontos e então problemaacabou!

Segundo caso (agora é que vai entrar o PCP): Suponha que AB > 1. Seja Xqualquer ponto em S − {A,B}. Pela hipótese do enunciado, na tripla A,B,Xexistem dois pontos com distância menor que 1. Logo, AX < 1 ou BX < 1,ou seja, cada ponto X ∈ S − {A,B} ou está num círculo de centro A e raio1, ou num círculo de centro B e raio 1; esses dois círculos serão as casas depombos e os 2005 = 2 × 1002 + 1 pontos em S − {A,B} serão os pombos.Então pelo princípio da casa dos pombos ao menos um destes círculos contém1003 pontos de S−{A,B}, e adicionando o centro, algum terá 1004 pontos de S.

Exercícios

1. Qual é o número mínimo de pessoas que devemos reunir para garantir queexistem duas pessoas que fazem aniversário no mesmo mês?

2. Dados 50 números menores do que 100, mostre que existem dois cuja somaé 99.

3. Existem 2017 pessoas numa sala, mostre que existem duas que conhecema mesma quantidade de pessoas. (Aqui estamos considerando que se umapesssoa A conhece a pessoa B, então a pessoa B tem que conhecer a pessoaA.)

4. Considere 5 pontos P1, P2, P3, P4, P5 no interior de um quadrado Q delado 1. Denote por dij a distância de Pi a Pj . Prove que a menor das

distâncias dij é menor que√

22 .

5. Dado um conjunto com 51 inteiros positivos menores ou iguais a 100,mostre que existe um elemento do conjunto que divide outro elemento doconjunto. (Dica: escreva cada elemento como N = 2km onde 2k é a maiorpotência de 2 que divide N e m é o maior divisor ímpar de N , e compareos fatores ímpares).

Para mais informação sobre o PCP veja a Revista Eureka! número 5.

OBM

1999. Uma caixa contém 100 bolas de cores distintas. Destas, 30 são vermelhas,30 são verdes, 30 são azuis e entre as 10 restantes, algumas são brancas e outrassão pretas. Qual é o menor número de bolas que devemos tirar da caixa, sem

Page 44: OBM por assunto: Os primeiros passos olímpicos · tenha muito a acrescentar na sua vida pois o foco do treinamento é justamente para alcançar esse nível de habilidade. ... Esse

44 CAPÍTULO 3. COMBINATÓRIA E PROBABILIDADE

lhes ver a cor, para termos a certeza de haver pelo menos 10 bolas da mesma cor?

2000. Uma caixa contém 900 cartões, numerados de 100 a 999. Retiram-seao acaso (sem reposição) cartões da caixa e anotamos a soma dos seus algaris-mos. Qual é a menor quantidade de cartões que devem ser retirados da caixa,para garantirmos que pelo menos três destas somas sejam iguais?

2009. Qual é o menor valor de n > 1 para o qual é possível colocar n peçassobre um tabuleiro n×n de modo que não haja duas peças sobre a mesma linha,mesma coluna ou mesma diagonal? As �guras a seguir mostram pares de peçasna mesma linha, na mesma coluna e na mesma diagonal em diversos tabuleiros.

2009. Um subconjunto de {1, 2, 3, · · · , 20} é superpar quando quaisquer doisde seus elementos têm produto par. Qual é a maior quantidade de elementos deum subconjunto superpar?

2011. Qual é a maior quantidade de números do conjunto {1, 2, 3, · · · , 20}que podemos escolher de modo que nenhum deles seja o dobro do outro?

2012. Em uma pesquisa de rua, cada entrevistado respondeu a quatro per-guntas, podendo sua resposta ser sim ou não, para cada uma das perguntas.Qual o número mínimo de entrevistados para garantirmos que duas pessoas res-ponderam igualmente a todas as perguntas?

2012. Quantos elementos tem o maior subconjunto de {1, 2, 3, · · · , 25} quenão contém dois números distintos cujo produto é um quadrado perfeito?

2015. Fabiana tem 55 cubos de mesmo tamanho, sendo 10 deles vermelhos,15 azuis e 30 verdes. Ela quer construir uma torre empilhando esses cubos demodo que dois cubos vizinhos tenham cores diferentes. No máximo, quantoscubinhos ela poderá empilhar?

2015. Dados cinco pontos no plano, sem três deles colineares, no mínimoquantos dos ângulos determinados por três desses cinco pontos são obtusos (ouseja, medem mais do que 90o)?

2015. Duas retas ou segmentos de retas no espaço são reversas quando nãoexiste um plano que contém ambas. Um dodecaedro regular é um poliedro comdoze faces pentagonais, todas regulares. Qual é a maior quantidade de elemen-tos de um conjunto S de arestas de um dodecaedro regular tal que quaisquerdois de seus elementos são reversos?

Page 45: OBM por assunto: Os primeiros passos olímpicos · tenha muito a acrescentar na sua vida pois o foco do treinamento é justamente para alcançar esse nível de habilidade. ... Esse

3.5. INVARIANTES 45

2016. Um colar é constituído por dois tipos de pérolas: as brancas e aspretas. Ele está aberto e disposto em uma mesa formando uma linha de pérolasconsecutivas. Duas sequências de três pérolas consecutivas são equivalentes seelas possuem exatamente as mesmas pérolas dispostas na mesma ordem ou emordem inversa. Por exemplo, se P e B indicam as cores das pérolas pretas ebrancas, respectivamente, a sequência PBBP contém duas sequências de trêspérolas equivalentes: as primeiras três, com a combinação PBB; e as últimastrês, com a combinação BBP. Qual a quantidade mínima de pérolas que o colardeve possuir para termos certeza de que existem duas sequências equivalentesindependentemente de como elas estejam distribuídas?

2016. Pedroso tem pilhas com 1, 2, 3, 4, 5, 6, 7, 8, 9 e 11 pedras (sim, elenão tem uma pilha com 10 pedras). Ele pode juntar duas pilhas de pedras.Quantas vezes, no mínimo, ele deve fazer essa operação para ter pilhas com asmesmas quantidades de pedras?

3.5 Invariantes

Às vezes quando um problema envolve uma mudança de estados é convenienteobservar aquilo que permanece depois da mudança, o que é invariante. Essaé uma estratégia muito comum quando se trata de problemas de olimpíadas.Essa análise é conveniente quando o resolvedor de problemas se depara com asseguintes perguntas:

1. É possível chegar em tal estado?

2. Encontre todos os estados �nais possíveis.

3. Encontre o período de algum ciclo.

Os exemplos a seguir deixarão mais clara a ideia apresentada.

Exemplo: Os números 1, 2, · · · , 14 estão escritos num quadro. Em cadarodada Ayane apaga dois números a e b e escreve o número |a − b| no quadro.É possível que Ayane obtenha 2 como resultado �nal?

Solução: Seja Si a soma dos números no quadro no passo i, é fácil calcularque S0 = 91, que é um número ímpar. Observe se num passo Ayane escolhe osnúmeros c, d com c < d para apagar ela então escreverá d− c = (c+ d)− 2c, ouseja, em cada passo é reduzido 2 min{a, b} de Si. Mas veja que se reduzimos umnúmero par de um número ímpar, o resultado é ímpar, ou seja, Si vai ser ím-par para todo i, logo não podemos ter Si = 2 para nenhum i. A resposta é "não".

Exemplo: Os cantos opostos de um tabuleiro de xadrez 8×8 são removidos.É possível cobrir os 62 quadrados restantes com dominós 2× 1 ?

Solução: A resposta é não. Se pintarmos as casas de branco e preto damaneira tradicional, temos que as duas casas retiradas são da mesma cor. Masum dominó cobre uma casa de cada cor, e assim não podemos cobrir 30 casas

Page 46: OBM por assunto: Os primeiros passos olímpicos · tenha muito a acrescentar na sua vida pois o foco do treinamento é justamente para alcançar esse nível de habilidade. ... Esse

46 CAPÍTULO 3. COMBINATÓRIA E PROBABILIDADE

de uma cor e 32 de outra.

OBM

1998. Hoje é sábado. Que dia da semana será daqui a 99 dias?

1998. Qual é o dígito das unidades do número 31998?

2000. Colocamos em ordem crescente os números escritos nas casas brancasdo tabuleiro a seguir (estamos mostrando apenas as suas quatro primeiras li-nhas). Assim, por exemplo, o nono número da nossa lista é 14. Qual é o 2000o

número da nossa lista?

2000. Em um jogo de duas pessoas, os jogadores tiram, alternadamente,1, 2, 3, 4 ou 5 palitos de uma pilha que inicialmente tem 1000 palitos. Ganhao jogador que tirar o último palito da pilha. Quantos palitos o jogador quecomeça deve tirar na sua jogada inicial de modo a assegurar sua vitória?

2003. Camila e Lara estão disputando o seguinte jogo num tabuleiro 4× 4:Camila marca algumas casas do tabuleiro e informa à Lara o número de casasmarcadas na vizinhança de cada casa do tabuleiro. Neste jogo, duas casasdistintas são consideradas vizinhas se possuem um lado ou um canto (vértice)em comum. Lara deve descobrir quais casas foram marcadas por Camila. Apósmarcar algumas casas, Camila passou para Lara o seguinte tabuleiro:

Qual é número de casas marcadas?

2003. Em um quadro negro escreve-se o número 1. As únicas alterações per-mitidas são substituí-lo pelo seu dobro ou pelo seu quadrado. Qual é o maiornúmero que pode ser obtido após efetuarmos 2003 alterações?

2006. Iniciando com o par (2048, 1024), podemos aplicar quantas vezes qui-sermos a operação que transforma o par (a, b) no par

(3a+b

4 , a+3b4

)então, dentre

os seguintes pares:

Page 47: OBM por assunto: Os primeiros passos olímpicos · tenha muito a acrescentar na sua vida pois o foco do treinamento é justamente para alcançar esse nível de habilidade. ... Esse

3.6. PROBABILIDADE 47

1) (1664, 1408)

2) (1540, 1532)

3) (1792, 1282)

4) (1537, 1535)

5) (1546, 1526)

A) Todos podem ser obtidos.

B) Apenas o par 4 não pode ser obtido.

C) Apenas o par 3 não pode ser obtido.

D) Existem exatamente dois pares que não podem ser obtidos.

E) Existem mais de dois pares que não podem ser obtido

3.6 Probabilidade

Falando sem rigor, no nosso contexto a probabilidade de que algo aconteça seráa razão entre o "tamanho do ambiente favorável" pelo "tamanho do ambientedas possibilidades".

Exemplo 1

Se queremos saber a probabilidade de escolher um número ímpar no con-junto {1, 2, 3, 4, 5}, o ambiente favorável será {1, 3, 5}, que tem três elementos,ou tamanho três no nosso sentido informal. Assim a probabilidade desejada será35 .

Exemplo 2

Qual é a probabilidade de escolher um número no intervalo [0, 25] e ele sermenor 18, 7 e maior que 13?

Nesse caso o conjunto total tem comprimento 25 enquanto que o conjuntoonde estão os números que queremos tem comprimento 5, 7, logo a probabilidadeserá 5,7

25 , ou se quisermos escrever de modo um pouquinho mais elegante 57250 .

Exemplo 3

Considere um qudrado ABCD de lado 2 e um semicírculo de raio 1 comdiâmetro AB dentro do quadrado. Qual é a probabilidade de escolhermos umponto do quadrado e ele estar dentro do semicírculo?

Neste caso, temos que o "tamanho total" a ser considerado é a área doquadrado, enquanto que o favorável será a área do semi-círculo. Assim a pro-babilidade será

π2

4 = π8 .

Agora que já demos a idéia geral do que será a probabilidade iremos partirpara exemplos que podem ser um pouco mais confusos a primeira vista. Olhandopara cada "tipo" de ambiente: conjuntos �nitos, segmentos, superfícies planas,etc.

Page 48: OBM por assunto: Os primeiros passos olímpicos · tenha muito a acrescentar na sua vida pois o foco do treinamento é justamente para alcançar esse nível de habilidade. ... Esse

48 CAPÍTULO 3. COMBINATÓRIA E PROBABILIDADE

3.6.1 Probabilidade Discreta

Consideraremos como probabilidade discreta a probabilidade em conjuntos �ni-tos. A probabilidade aqui será simplesmente

P (algo acontecer) =número de casos favoráveisnúmero de casos possíveis

.

Em essência não é necessário estudar combinatória para estudar probabili-dade. Essas duas coisas são apresentadas juntas pelo fato de que sem a ferra-menta da combinatória �caria difícil contar as quantidades de casos favoráveise de casos possíveis. Como a de�nição é incrivelmente simples, procederemospor meio de exemplos.

Exemplo Um dado comum é lançado. Qual é a probabilidade da face vol-tada para cima ser um número primo?

O conjunto das possibilidade é {1, 2, 3, 4, 5, 6}, que tem 6 elementos. Oconjunto dos casos favoráveis é {2, 3, 5}, que possui dois elementos. Logo a pro-babilidade desejada é 3

6 = 12 .

Exemplo Duas moedas comuns são lançadas simultaneamente. Qual é aprobabilidade das duas moedas darem o mesmo resultado.

Os possíveis resultados para as faces a mostra das moedas são cara que sim-bolizaremos por C, ou coroa, que simbolizaremos por K. Assim, o conjuntodas possibilidades nesse caso são (C,C), (C,K), (K,C) ou (K,K). Os casosfavoráveis aqui são então (C,C) ou (K,K), assim temos duas possibilidades emquatro e a probabilidade desejada é 1

2 .

Um tipo de coisa que pode deixar uma pessoa confusa com respeito a pro-babilidade é o seguinte tipo de problema:

Exemplo Sejam A = {1, 2, 3, 4, 5} e B = {2, 4, 6, 8}. Qual é a probabilidadede se escolher um elemento de A e ele pertencer a B?

A confusão surge quando se considera B de casos favoráveis. Nesse caso,deve analizar quantos elementos de A estão em B e não simplesmente quantoselementos estão em B. Os casos favoráveis, são sempre um subconjunto dos ca-sos possíveis. E nesse caso a probabilidade é 2

5 e não 35 como poderia imaginar

um leitor desatento.

Exemplo Em uma urna há duas moedas aparentemente iguais. Uma delasé uma moeda comum, com uma cara e uma coroa. A outra, no entanto, é umamoeda falsa, com duas caras. Suponhamos que uma dessas moedas seja sorte-ada e lançada. Se o resultado do lançamento é cara, qual é a probabilidade deque a moeda sorteada tenha sido a comum?

Aqui é natural que se cometa a confusão de que a probabilidade de escolher amoeda comum é 1

2 e por isso a resposta é 12 . De fato a probabilidade de escolher

Page 49: OBM por assunto: Os primeiros passos olímpicos · tenha muito a acrescentar na sua vida pois o foco do treinamento é justamente para alcançar esse nível de habilidade. ... Esse

3.6. PROBABILIDADE 49

a moeda comum é 12 . No entanto temos uma informação extra de que depois do

lançamento obtivemos uma cara. Assim nossas possibilidades são:1. Essa é a cara da moeda comum2. Essa é uma cara da moeda com duas caras3. Essa é a outra cara da moeda com duas carasAssim, o conjunto de casos possíveis tem 3 elementos, enquanto que o de

casos favoráveis tem apenas 1. Portanto a resposta é na verdade 13 .

3.6.2 Probabilidade Geométrica

Quando queremos calcular a probabilidade de escolher um ponto numa parte deum segmento, basta dividir o comprimento da região favorável pelo da regiãopossível. Analogamente para superfície, divide a área da região favorável pelaárea da região possível. Isso é uma de�nição. Como no caso de probabilidadediscreta iremos estudar o tema por meio de exemplos, já que só temos essa de-�nição.

Exemplo Seja A um ponto num plano. Suponha que escolhemos um pontoB nesse plano cuja distância a A é menor ou igual a 2. Qual é a probabilidadede que B esteja a uma distância menor ou igual a 1 de A?

O conjunto dos pontos a uma distância menor ou igual a d de A é o círculode centro A e raio d, cuja área como sabemos é πd2. Assim a área possível é4π, enquanto que a área favorável é π, logo a probabilidade é 1

4 .

O tipo de confusão com casos favoráveis que ocorre com os problemas deprobabilidade em conjuntos �nitos pode ocorrer também em probabilidade ge-ométrica. Vejamos a seguir um exemplo semelhante ao que vimos na primeirasubseção.

ExemploConsidere um quadrado ABCD de lado 2 e um círculo Γ de raio 1 com di-

âmetro AB. Qual é a probabilidadde de escolhermos um ponto do quadrado eele estar dentro do círculo?

Neste caso, temos que o "tamanho total" a ser considerado é a área do qua-drado, enquanto que o favorável será a área da parte de Γ que está dentro deABCD. Assim a probabilidade será

π2

4 = π8 , e não

π4 como se poderia imaginar..

Para mais problemas estranhos de probabilidade veja a Revista Eureka! nú-mero 1.

OBM

2001. Uma rifa foi organizada entre os 30 alunos da turma do Pedro. Para tal,30 bolinhas numeradas de 1 a 30 foram colocadas em uma urna. Uma delas foi,então, retirada da urna. No entanto, a bola caiu no chão e se perdeu e umasegunda bola teve que ser sorteada entre as 29 restantes. Qual a probabilidade

Page 50: OBM por assunto: Os primeiros passos olímpicos · tenha muito a acrescentar na sua vida pois o foco do treinamento é justamente para alcançar esse nível de habilidade. ... Esse

50 CAPÍTULO 3. COMBINATÓRIA E PROBABILIDADE

de que o número de Pedro tenha sido o sorteado desta segunda vez?

2002. Duas pessoas vão disputar uma partida de par ou ímpar. Elas nãogostam do zero e, assim, cada uma coloca 1, 2, 3, 4 ou 5 dedos com igual proba-bilidade. Qual é a probabilidade de que a pessoa que escolheu par ganhe?

2003. Beatriz, Isabele e Nicole estão disputando um jogo fazendo lançamen-tos sucessivos com uma moeda. Beatriz ganha se, em dois lançamentos consecu-tivos, o primeiro resultar cara e o segundo coroa. Isabele ganha se forem obtidasduas coroas em dois lançamentos consecutivos, e Nicole ganha se forem obtidasduas caras em dois lançamentos consecutivos. Elas fazem os lançamentos atéque uma das jogadoras seja vencedora. Qual(is) jogadora(s) possuem menoschances de ganhar o jogo?

2004. Dois cubos têm faces pintadas de ocre ou magenta. O primeiro cubotem cinco faces ocres e uma face magenta. Quando os dois cubos são lançados, aprobabilidade de as faces viradas para cima dos dois cubos serem da mesma cor(sim, ocre e magenta são cores!) é 1/2. Quantas faces ocres tem o segundo cubo?

2006. Uma colônia de amebas tem inicialmente uma ameba amarela e umaameba vermelha. Todo dia, uma única ameba se divide em duas amebas idên-ticas. Cada ameba na colônia tem a mesma probabilidade de se dividir, nãoimportando sua idade ou cor. Qual é a probabilidade de que, após 2006 dias, acolônia tenha exatamente uma ameba amarela?

2008. Arnaldo, Bernaldo, Cernaldo e Dernaldo embaralharam as 52 cartasde um baralho e distribuíram 13 cartas para cada um. Arnaldo �cou surpreso:"Que estranho, não tenho nenhuma carta de espadas."Qual a probabilidade deBernardo também não ter cartas de espadas?

2010. Um ponto P é escolhido ao acaso no interior de um quadrado QRST .Qual é a probabilidade do ângulo ∠RPQ ser agudo?

2012. O triângulo ABC tem lados AB = 6, AC = 8 e BC = 10. Escolhe-seum ponto X ao acaso no interior do triângulo ABC. Sejam pA, pB e pC asprobabilidades de que o vértice do triângulo ABC mais próximo de X seja A,Be C, respectivamente. Então

A) pA > pB = pC .B) pA > pB > pC .C) pA > pC > pB .D) pA < pB < pC .E) pA = pB = pC .

2013. Uma potência perfeita é um número inteiro da forma ab,a e b inteiros,b > 1. Seja f(n) a maior potência perfeita que não excede n. Por exemplo,f(7) = 4, f(8) = 8 e f(99) = 81. Sorteando ao acaso um número inteiro k com1 ≤ k ≤ 100, qual a probabilidade de ser um quadrado perfeito?

2014. Cada uma de 2014 bolas é pintada de azul, verde ou amarelo e écolocada aleatoriamente em uma de três urnas, uma azul, outra verde e a ter-

Page 51: OBM por assunto: Os primeiros passos olímpicos · tenha muito a acrescentar na sua vida pois o foco do treinamento é justamente para alcançar esse nível de habilidade. ... Esse

3.6. PROBABILIDADE 51

ceira amarela. Qual é a probabilidade de que cada urna contenha exatamenteas bolas com a sua respectiva cor?

2014. Na Esmeralândia, as moedas são retangulares. Qual é a probabilidadede uma moeda de lados 3 e 4 cair totalmente dentro de um ladrilho formadopor retângulos de lados 10 e 20?

2016. Humberto joga um dado honesto e obtém x1 pontos; Doisberto jogadois dados honestos e tira a média aritmética x2 dos resultados; Tresberto jogatrês dados honestos e tira a média aritmética x3 dos resultados. Todos os dadostêm faces numeradas de 1 a 6. Sendo p1, p2 e p3 as probabilidades de obterx1 ≥ 5, x2 ≥ 5 e x3 ≥ 5, respectivamente, então

A) p1 < p2 < p3.B) p1 < p3 < p2.C) p2 < p1 < p3.D) p2 < p3 < p1.E) p3 < p2 < p1.

Page 52: OBM por assunto: Os primeiros passos olímpicos · tenha muito a acrescentar na sua vida pois o foco do treinamento é justamente para alcançar esse nível de habilidade. ... Esse

52 CAPÍTULO 3. COMBINATÓRIA E PROBABILIDADE

Page 53: OBM por assunto: Os primeiros passos olímpicos · tenha muito a acrescentar na sua vida pois o foco do treinamento é justamente para alcançar esse nível de habilidade. ... Esse

Capítulo 4

Álgebra

A álgebra básica, que se estuda no ensino médio e que veremos aqui, é o quese pode considerar como a generalização da aritmética básica, passando dasoperações entre números para as operações entre variáveis. Inicialmente veremosalgumas manipulações algébricas que facilitarão a chegada a certos resultados eem seguida a de�nição e algumas propriedades de polinômios, cuja importânciana matemática em geral é extrema, sendo por exemplo, um tópico recorrentena álgebra avançada.

"Busquem conhecimento!"- ET Bilu.

4.1 Exercícios de Equações e sistemas

Nesta primeira seção esperamos que os problemas sejam resolvidos utilizandoapenas conhecimentos relacionados a resolução de equações e de sistemas deequações lineares. Certamente a di�culdade a ser superada é a interpretaçãodo enunciados das questões na linguagem de equações e sistemas de equaçõessendo necessário exercitar esta habilidade de "parafrasear".

4.1.1 Equações

1998. A média aritmética de seis números é 4. Quando acrescentamos um sé-timo número, a nova média é 5. Qual número que foi acrescentado?

1998. Barcas vão do Rio a Niterói em 25 minutos e lanchas fazem a viagemem 15 minutos. A que horas a barca que partiu do Rio às 10h01min é alcançadapela lancha que saiu do Rio às 10h07min?

1999. Um pequeno caminhão pode carregar 50 sacos de areia ou 400 tijolos.Se foram colocados no caminhão 32 sacos de areia, quantos tijolos pode aindaele carregar?

2001. Uma pera tem cerca de 90% de água e 10% de matéria sólida. Umprodutor coloca 100 quilogramas de pera para desidratar até o ponto em que aágua represente 60% da massa total. Quantos litros de água serão evaporados?

53

Page 54: OBM por assunto: Os primeiros passos olímpicos · tenha muito a acrescentar na sua vida pois o foco do treinamento é justamente para alcançar esse nível de habilidade. ... Esse

54 CAPÍTULO 4. ÁLGEBRA

(lembre-se: 1 litro de água tem massa de 1 quilograma).

2001. Um serralheiro tem 10 pedaços de 3 elos de ferro cada um, mostradosabaixo. Ele quer fazer uma única corrente de 30 elos. Para abrir e depois soldarum elo o serralheiro leva 5 minutos. Quantos minutos no mínimo ele levará parafazer a corrente?

2002. Durante sua viagem ao país das Maravilhas a altura de Alice sofreuquatro mudanças sucessivas da seguinte forma: primeiro ela tomou um gole deum líquido que estava numa garrafa em cujo rótulo se lia: "beba-me e �que25% mais alta". A seguir, comeu um pedaço de uma torta onde estava escrito:"prove-me e �que 10% mais baixa"; logo após tomou um gole do líquido de outragarrafa cujo rótulo estampava a mensagem: "beba-me e �que 10% mais alta".Finalmente, comeu um pedaço de outra torta na qual estava escrito:"prove-mee �que 20% mais baixa". Após a viagem qual era a altura de Alice?

2002. Marcelo leva exatamente 20 minutos para ir de sua casa até a escola.Certa vez, durante o caminho, percebeu que esquecera em casa a revista Eureka!que ia mostrar para a classe; ele sabia que se continuasse a andar, chegaria àescola 8 minutos antes do sinal, mas se voltasse para pegar a revista, no mesmopasso, chegaria atrasado 10 minutos. Que fração do caminho já tinha percorridoneste ponto?

2002. Vamos provar que 4 é maior que 4. Sejam a e b dois números tais quea > 4 e a = b.

1) Vamos subtrair 4 dos dois termos desta equação:

a = b

a− 4 = b− 4

2) Colocamos −1 em evidência no segundo membro da equação:

a− 4 = −1(−b+ 4)

a− 4 = −1(4− b)

3) Elevamos ambos os termos da equação ao quadrado:

(a− 4)2 = [−1 · (4− b)]2

(a− 4)2 = (−1)2(4− b)2

(a− 4)2 = 1 · (4− b)2

(a− 4)2 = (4− b)2

4) Extraímos a raiz quadrada dos dois membros da equação:

(a− 4)2 = (4− b)2

a− 4 = 4− b

Page 55: OBM por assunto: Os primeiros passos olímpicos · tenha muito a acrescentar na sua vida pois o foco do treinamento é justamente para alcançar esse nível de habilidade. ... Esse

4.1. EXERCÍCIOS DE EQUAÇÕES E SISTEMAS 55

5) Como a = b, substituímos b por a

a− 4 = 4− a

6) Resolvemos a equação:

a− 4 = 4− a2a = 8

a = 4

Como escolhemos a tal que a > 4, chegamos à inacreditável conclusão deque 4 > 4. Qual é o erro?

2007. Tintas pretas opacas absorvem 97% da luz, re�etindo o restante. Ci-entistas desenvolveram uma nova cobertura superpreta que é "dez vezes maispreta" que tintas pretas opacas, querendo dizer que ela re�ete 1/10 da luzre�etida pelas tintas pretas opacas. Que porcentagem de luz a nova coberturaabsorve?

2016. Numa maratona com 2016 participantes, o número de corredores quechegaram antes de Josias foi igual a um quarto do número de corredores quechegaram depois de Josias. Em que lugar chegou Josias?

4.1.2 Sistemas

1999. Um retângulo ABCD está dividido em quatro retângulos menores. Asáreas de três deles estão na �gura abaixo. Qual é a área do retângulo ABCD?

1999. Em um aquário há peixes amarelos e vermelhos: 90% são amarelose 10% são vermelhos. Uma misteriosa doença matou muitos peixes amarelos,mas nenhum vermelho. Depois que a doença foi controlada veri�cou-se que noaquário, 75% dos peixes vivos eram amarelos. Aproximadamente, que porcen-tagem dos peixes amarelos morreram?

1999. Hoje, 12/6/1999, Pedro e Maria fazem aniversário. No mesmo dia em1996, a idade de Pedro era 3/4 da idade de Maria. No mesmo dia em 2002, aidade de Pedro será igual à de Maria quando ele tinha 20 anos. Quantos anosMaria está fazendo hoje?

2000. Dois nadadores, inicialmente em lados opostos de uma piscina, come-çam simultaneamente a nadar um em direção ao outro. Um deles vai de um

Page 56: OBM por assunto: Os primeiros passos olímpicos · tenha muito a acrescentar na sua vida pois o foco do treinamento é justamente para alcançar esse nível de habilidade. ... Esse

56 CAPÍTULO 4. ÁLGEBRA

lado a outro da piscina em 45 segundos e o outro em 30 segundos. Eles nadamde um lado para outro por 12 minutos, sem perder qualquer tempo nas viradas.Quantas vezes eles passam um pelo outro (indo no mesmo sentido ou em senti-dos opostos) durante este tempo, contando as vezes em que se encontram nosextremos da piscina?

2000. O emir Abdel Azir �cou famoso por vários motivos. Ele teve mais de39 �lhos, incluindo muitos gêmeos. De fato, o historiador Ahmed Aab a�rmanum dos seus escritos que todos os �lhos do emir eram gêmeos duplos, exceto39; todos eram gêmeos triplos, exceto 39; todos eram gêmeos quádruplos, exceto39. Qual é o número de �lhos do emir?

2000. Alberto, Beatriz e Carlos correm numa pista circular. Todos saem aomesmo tempo e do mesmo lugar, cada um desenvolvendo velocidade constante.Alberto e Beatriz correm no mesmo sentido. Correndo no sentido oposto, Carlosencontra Alberto, pela primeira vez, exatamente 90 segundos após o início dacorrida e encontra Beatriz exatamente 15 segundos depois. Quantos segundossão necessários para que Alberto ultrapasse Beatriz pela primeira vez?

2000. De Itacimirim a Salvador, pela estrada do Coco, são 60 km. Às 11horas, a 15 km de Salvador, dá-se um acidente que provoca um engarrafamento,que cresce à velocidade de 4 km/h, no sentido de Itacimirim. A que horas, apro-ximadamente, devemos sair de Itacimirim para chegar a Salvador ao meiodia,sabendo que viajamos a 60 km/h, exceto na zona de engarrafamento, onde avelocidade é 6 km/h?

2002. Um comerciante comprou dois carros por um total de R$27.000, 00.Vendeu o primeiro com lucro de 10% e o segundo com prejuízo de 5%. No totalganhou R$750, 00. Quais foram os preços de compra?

2002. A média aritmética das idades de um grupo de médicos e advogados é40 anos. A média aritmética das idades dos médicos é 35 anos e a dos advogadosé 50 anos. Pode-se, então, a�rmar que:

A) O número de advogados é o dobro do número de médicos no grupo.B) O número de médicos é o dobro do número de advogados no grupo.C) Há um médico a mais no grupo.D) Há um advogado a mais no grupo.E) Existem as mesmas quantidades de médicos e advogados no grupo.

2003. Dois amigos, Augusto e Eduardo, atravessavam uma ponte onde pas-sava uma linha férrea. Quando tinham percorrido dois quintos da ponte, ou-viram o barulho de um trem que se aproximava por trás deles. Apavorados,começaram a correr, cada um para o seu lado. Tiveram sorte: Augusto, quetinha voltado, conseguiu sair da ponte no exato instante em que o trem nela iaentrar. Por sua vez, Eduardo, que continuou para a frente, conseguiu sair daponte no instante em que o trem também ia fazê-lo. Refeitos do susto, quandose encontraram, comentaram que isto só foi possível porque correram a 15km/he o trem estava a xkm/h. Qual é o valor de x?

Page 57: OBM por assunto: Os primeiros passos olímpicos · tenha muito a acrescentar na sua vida pois o foco do treinamento é justamente para alcançar esse nível de habilidade. ... Esse

4.1. EXERCÍCIOS DE EQUAÇÕES E SISTEMAS 57

2005. As letras O,B eM representam números inteiros. Se O×B×M = 240,O ×B +M = 46 e O +B ×M = 64, quanto vale O +B +M?

2006. No �m de 1994, Neto tinha a metade da idade de sua avó. A somados anos de nascimento dos dois é 3844. Quantos anos Neto completa em 2006?

2006. Quantos ternos de números reais x, y, z satisfazem o sistema abaixo?

x(x+ y + z) = 2005

y(x+ y + z) = 2006

z(x+ y + z) = 2007

2008. Em uma pista de corrida, cujo formato é de um polígono regular den vértices, numerados de 1 até n no sentido anti-horário, existem três pessoas:Nelly, Sônia e Penha, estando inicialmente todas em um mesmo vértice. Em umdado momento elas começam a caminhar pelos lados do polígono. Nelly cami-nha no sentido anti-horário, enquanto que Sônia e Penha caminham no sentidocontrário. Nelly cruza com Sônia pela primeira vez em um vértice e com Penhadois vértices à frente. A velocidade de Nelly é o dobro da velocidade de Sônia ea velocidade de Sônia é o dobro da velocidade de Penha. Quantos vértices temo polígono?

2008. Nove números são escritos em ordem crescente. O número do meioé a média aritmética dos nove números. A média aritmética dos 5 maiores é68 e a média aritmética dos 5 menores é 44. Qual é a soma de todos os números?

2009. Dois carros deixam simultaneamente as cidades A e B indo de umacidade em direção à outra, com velocidades constantes, e em sentidos opostos.As duas cidades são ligadas por uma estrada reta. Quando o carro mais rápidochega ao ponto médio M de AB, a distância entre os dois carros é de 96km.Quando o carro mais lento chega ao ponto M , os carros estão a 160km um dooutro. Qual a distância, em km, entre as duas cidades?

2010. Ana começou a descer uma escada no mesmo instante em que Beatrizcomeçou a subi-la. Ana tinha descido 3

4 da escada quando cruzou com Beatriz.No momento em que Ana terminar de descer, que fração da escada Beatriz aindaterá que subir?

2012. As massas de todos os pares possíveis formados com 5 estudantes são90kg, 92kg, 93kg, 94kg, 95kg, 96kg, 97kg, 98kg, 100kg e 101kg. Qual é a massado estudante de massa intermediária?

2014. Um caminhão tanque estava cheio de água, mas começou a vazar.Suponha que o consumo de combustível do caminhão seja diretamente propor-cional ao peso que carrega e que a vazão da água e a velocidade do caminhãosejam constantes. Após percorrer 200km, o caminhão estava com metade dacapacidade de água e gastou meio tanque de combustível. Se estivesse vazio,o caminhão gastaria, se percorresse a mesma distância nas mesmas condições,

Page 58: OBM por assunto: Os primeiros passos olímpicos · tenha muito a acrescentar na sua vida pois o foco do treinamento é justamente para alcançar esse nível de habilidade. ... Esse

58 CAPÍTULO 4. ÁLGEBRA

um sexto de tanque. Que fração do tanque ele gastaria se não houvesse o vaza-mento? Despreze a in�uência do peso do tanque no consumo de gasolina.

2015. Esmeralda e Jade saíram da secretaria da OBM e foram para o Jar-dim Botânico. As duas saíram ao mesmo tempo, Esmeralda de bicicleta e Jadecaminhando. A velocidade de Esmeralda é o quádruplo da velocidade de Jade,e as duas velocidades são constantes. Esmeralda chegou ao Jardim Botânico,esperou 5 minutos e depois voltou pelo mesmo caminho, encontrando Jade indo,bem na metade do caminho. Quanto tempo demora a caminhada de Jade dasecretaria até o Jardim Botânico?

2016. Quantas são as triplas ordenadas (a, b, c) de reais tais que

a =b

c+c

b, b =

c

a+a

c, c =

a

b+b

a.

4.2 Produtos Notáveis e Fatorações Notáveis

Já pensou entrar em um jogo de cartas sem saber bem quais são as jogadaspermitidas? É por isso que decidimos apresentar, ou apenas lembrar, algumaspropriedades algébricas básicas que poderão ser bastante úteis nesta seção! Que(re)comecem os jogos.

4.2.1 Lista de produtos notáveis

A seguir listamos os produtos notáveis básicos.Quadrado da Soma:(a+ b)2 = a2 + 2ab+ b2

(a+ b+ c)2 = a2 + b2 + c2 + 2ab+ 2bc+ 2caQuadrado da diferença:(a− b)2 = a2 − 2ab+ b2

Cubo da soma(a+ b)3 = a3 + 3a2b+ 3ab2 + b3 = a3 + b3 + 3ab(a+ b)Cubo da diferença(a− b)3 = a3 − 3a2b+ 3ab2 − b3Diferença de quadradosa2 − b2 = (a+ b)(a− b)Soma dos cubosa3 + b3 = (a+ b)(a2 − ab+ b2)Diferença dos cubosa3 − b3 = (a− b)(a2 + ab+ b2)Identidade de Sophi Germaina4 + 4b4 = (a2 + 2b2 + 2ab)(a2 + 2b2 − 2ab)Dê um nome pra essaab+ a+ b+ 1 = (a+ 1)(b+ 1)

Exemplo: Sejam x, y, z ∈ R tais que x+y+z = 0. Mostre que x3+y3+z3 =3xyz.

Page 59: OBM por assunto: Os primeiros passos olímpicos · tenha muito a acrescentar na sua vida pois o foco do treinamento é justamente para alcançar esse nível de habilidade. ... Esse

4.2. PRODUTOS NOTÁVEIS E FATORAÇÕES NOTÁVEIS 59

Solução:

x+ y + z = 0⇒x+ y = −z ⇒

(x+ y)3 = −z3 ⇒x3 + y3 + 3xy(x+ y) = −z3 ⇒

x3 + y3 − 3xyz = −z3 ⇒x3 + y3 + z3 = 3xyz

Para aprender mais sobre produtos notáveis veja a Eurka 27 pág.32-37.

Exercícios

1. Veri�que cada identidade da lista acima.

2. Determine a expressão que deve ser multiplicada por x 3√

2 + 2 3√x para

obtermos 2x(x2 + 4).

3. Racionalize o denominador de1√

2 + 3√

3.

4. (Cone Sul - 1992 adaptado) De�ne-se o conjunto de 100 números1, 1/2, 1/3, · · · , 1/100. Eliminan-se dois elementos quaisquer a e b desteconjunto e se inclui, no conjunto, o número a⊗b = a+b+ab �cando assimum conjunto com um elemento a menos. Depois de 99 destas operações,�ca só um número ρ.

a) Mostre que a⊗ b = b⊗ a.b) Mostre que (a⊗ b)⊗ c = a⊗ (b⊗ c).c) Determine os possíveis valores de ρ.

OBM

2002. Se xy = 2 e x2 + y2 = 5, então quanto vale x2

y2 + y2

x2 ?

2004. Sejam

a =12

2+

22

3+

32

5+ · · ·+ 10012

2001

b =12

3+

22

5+

32

7+ · · ·+ 10012

2003.

Qual é o inteiro mais próximo de a− b?

2005. Os inteiros positivos x e y satisfazem a equação√x+

1

2

√y −

√x− 1

2

√y = 1.

Qual das alternativas apresenta um possível valor de y?A) 5 B) 6 C) 7 D) 8 E) 9

Page 60: OBM por assunto: Os primeiros passos olímpicos · tenha muito a acrescentar na sua vida pois o foco do treinamento é justamente para alcançar esse nível de habilidade. ... Esse

60 CAPÍTULO 4. ÁLGEBRA

2006. Os dois números reais a e b são não nulos e satisfazem ab = a − b.Assinale a alternativa que exibe um dos possíveis valores de a

b + ba − ab.

A) −2 B) −12 C) 1

3 D) 12 E) 2

2007. Se x é real positivo e 1 + (x2 + x)(x2 + 5x+ 6) = 1812, então qual é ovalor de x(x+ 3)?

2007. Sejam a, b e c números tais que

a2 − ab = 1

b2 − bc = 1

c2 − ca = 1.

Qual é o valor de abc(a+ b+ c)?

2009. Se x2 = x+ 3 então x3 é igual a:A) x2 + 3; B)x+ 4; C) 2x+ 2; D) 4x+ 3; E) x2 − 2.

2010. Os números x e y são distintos e satisfazem x − 1x = y − 1

y , entãoquanto vale xy?

2011. Qual é o valor da expressão 201120112 + 201120032 − 16× 20112007?A)2× 201120072

B) 2× 201120032

C) 2× 20112007D) 2× 20112003E) 2× 201120112

2011. Sendo a e b inteiros tais que (1 +√

2)2011 = a + b√

2 ,(1 +√

2)2010 éigual a

A) a+ 2b+ (a− b)√

2B) a− 2b+ (a− b)

√2

C) a+ 2b+ (b− a)√

2D) 2b− a+ (a− b)

√2

E) a+ 2b− (a+ b)√

2

2012. Se x2 = 2x+ 4, então (x+ 1)−1 é igual a:A) x+ 2; B) x− 3; C) x− 1; D) 2x+ 5; E) 3x+ 5.

2012. Quantas soluções reais têm o sistema abaixo?

x+ y =1

z2

y + z =1

x2

z + x =1

y2.

2013. Sejam a, b, reais positivos tais que a+2bb = a+b

a . Quanto vale (a+b)2

a ?

Page 61: OBM por assunto: Os primeiros passos olímpicos · tenha muito a acrescentar na sua vida pois o foco do treinamento é justamente para alcançar esse nível de habilidade. ... Esse

4.3. EQUAÇÃO DO SEGUNDO GRAU 61

2013. Determine x + y, onde x e y são reais, sabendo que x3 + y3 = 9 exy2 + x2y = 6.

4.2.2 Completando Quadrados

Quando se deseja resolver uma equação de segundo grau, ou simpli�car umexpressão do tipo ax2 + bx + c em geral é muito útil fazer o que chamamosde completar quadrado. O propósito imediato de completar quadrado é obteruma expressão do tipo (αx + β)2 + γ, se a ≥ 0 ou do tipo −(αx + β)2 + γ sea < 0 (com α, β, γ não necessariamente positivos). Esse tipo de coisa facilita naresolução de diversos problemas, entre eles a equação do segundo grau.

Começaremos com exemplos ilustrativos e depois iremos para o caso geral.

Exemplo Simpli�car a expressão x2 + 8x− 1.

Queremos escrever x2 +8x−1 = (αx+β)2 +γ. Observe que (αx+β)2 +γ =α2x2 + 2αβx+ β2 + γ. Assim queremos que

α2x2 + 2αβx+ β2 + γ = x2 + 8x− 1.

A resposta a ser obtida não precisa ser única, apenas queremos uma resposta.Assim poderíamos considerar α = 1 para que os termos que acompanham x2 seigualem. Observe que se tomamos α = −1 o mesmo ocorre, mas tomaremos ovalor 1 por simplicidade.

Se α = 1 temos

x2 + 2βx+ β2 + γ = x2 + 8x− 1

⇒ 2βx+ β2 + γ = 8x− 1.

Agora vamos determinar β. De modo análogo queremos que 2β = 8, paraigualarmos os coe�cientes do termo x. Logo temos β = 4 e obtemos

2 · 4x+ 42 + γ = 8x− 1⇒ 16 + γ = −1

Finalmente escolhemos o γ adequado, que será então γ = −17.Assim podemos escrever a expressão x2 + 8x− 1 = (x+ 4)2 − 17.

Exercício Simpli�que, de modo análogo ao caso acima, as expressõesa) −x2 + 12x− 13;b) 2x2 + 7x+ 9;c) 4x2 + 4x+ 1;d) x4 + 8x2 + 7.

4.3 Equação do Segundo Grau

A seguir veremos como deduzir a conhecida fórmula da solução da equação dosegundo grau. A dedução se baseará no que vimos anteriormente sobre comple-tar quadrado.

Page 62: OBM por assunto: Os primeiros passos olímpicos · tenha muito a acrescentar na sua vida pois o foco do treinamento é justamente para alcançar esse nível de habilidade. ... Esse

62 CAPÍTULO 4. ÁLGEBRA

Começaremos pelos casos mais simples, que são bem fáceis e em seguidausaremos esses casos e o completamento de quadrado para deduzir a fórmulageral.

Exemplo 1.As soluções da equação x2 = γ com γ ≥ 0 são x1 =

√γ e x2 = −√γ.

A equação x2 = γ com γ < 0 não temos solução real.

Exercícios

1. Calcule os possíveis valores de x nas equações abaixo

a) x2 = 4;

b) x2 = 7;

c) (x+ 1)2 = 9;

d) (x+ 2)2 = 13.

Exemplo 2.Resolva a (αx+ β)2 = γ, com α 6= 0, γ > 0.

Solução: (αx + β)2 = γ ⇐⇒ (αx + β) = ±√γ ⇐⇒ αx = −β ±√γ. Ouseja, as soluções são x1 =

−β+√γ

α ou x2 =−β−√γ

α .

Exercícios

1. Calcule os possíveis valores de x nas equações abaixo

a) (x+ 3)2 = 36;

b) x2 + 2x+ 1 = 25;

c) x2 + 8x+ 16 = 169;

d) x4 + 2x2 + 1 = 10.

Agora vamos analizar o caso geral, no qual nos é dado uma equação do se-gundo grau.

Exemplo 3.Sejam a, b, c números reais, com a 6= 0. Encontre as soluções da equação

ax2 + bx+ c = 0.

Solução: Nosso objetivo será reduzir essa equção a uma do tipo do Exemplo2. Observe que (αx+β)2 = α2x2 +2αβx+β2. Como na equação que desejamosresolver não sabemos se a > 0, para escrever a = α2, o que faremos é multiplicara equação por a, obtendo a2x2 + abx+ ac = 0. Fazemos a2x2 = α2x2, faremosentão α = a.

Page 63: OBM por assunto: Os primeiros passos olímpicos · tenha muito a acrescentar na sua vida pois o foco do treinamento é justamente para alcançar esse nível de habilidade. ... Esse

4.3. EQUAÇÃO DO SEGUNDO GRAU 63

O segundo passo será investigar o valor de 2αβx = 2aβx. Observe que comoesse termo é o que acompanha x devemos ter 2aβx = abx, portanto 2β = b, ouseja, β = b

2 .

O passo �nal é olhar para β2. Veja que nesse caso devemos ter β2 = b2

4 ,e para termos uma expressão idêntica a um quadrado da soma, iremos somarb2

4 e obviamente temos que subtrair o mesmo valor, pois não podemos alterar ovalor da expressão. Assim obtemos que a nossa equação é equivalente a a2x2 +

abx+ b2

4 −b2

4 + ac = 0, agora vamos fazer conta:

ax2 + bx+ c = 0

⇐⇒ a2x2 + abx+ ac = 0

⇐⇒ a2x2 + abx+b2

4− b2

4+ ac = 0

⇐⇒ a2x2 + abx+b2

4=b2

4− ac

⇐⇒(ax+

b

2

)2

=b2 − 4ac

4

Assim, se b2 − 4ac < 0, a equação não possui solução real (mas possui soluçãocomplexa), pois o lado direito será negativo, e um número real elevado quadradonão pode ser negativo. Assim, se b2 − 4ac ≥ 0 temos

ax2 + bx+ c = 0

⇐⇒(ax+

b

2

)2

=b2 − 4ac

4

⇐⇒(ax+

b

2

)=±√b2 − 4ac

2

⇐⇒ ax = − b2±√b2 − 4ac

2

⇐⇒ ax =−b±

√b2 − 4ac

2

⇐⇒ x =−b±

√b2 − 4ac

2a.�

A expressão x = −b±√b2−4ac

2a , como você provavelmente já sabe, é conhe-cida (no Brasil) como fórmula de Bháskara, e é a fórmula que dá a solução daequação do segundo grau. É importante saber decorada essa fórmula, talvezmais até do que saber deduzí-la. É usual escrever as duas soluções da equaçãocomo x1 = −b+

√b2−4ac

2a e x2 = −b−√b2−4ac

2a . Quando b2 − 4ac = 0, x1 = x2, e éconvencional dizer que a equação tem duas soluções iguais (e não que tem ape-nas uma solução, apesar de dar na mesma e das duas coisas estarem corretas)apenas para dar ênfase de que a equação é de segundo grau.

Exercícios

1. Calcule em termos de a, b e c

Page 64: OBM por assunto: Os primeiros passos olímpicos · tenha muito a acrescentar na sua vida pois o foco do treinamento é justamente para alcançar esse nível de habilidade. ... Esse

64 CAPÍTULO 4. ÁLGEBRA

a) x1 + x2;

b) x1x2.

2. Encontre todas as soluções reais de cada equação abaixo:

a) x2 − 12x+ 36 = 0;

b) x2 − x+ 6 = 0;

c) x4 − 5x2 + 4 = 0;

d) (x2 − 8x+ 11)2 − 7(x2 − 8x+ 11) + 10 = 0;

e) (x− 3)(x− 2) = x+ 4.

3. Determine se existe um retângulo com

a) Perímetro 10 e área 6;

b) Perímetro 10 e área 7.

4. Encontre os valores das expressões abaixo

a)√

2 +√

2 +√

2 + · · ·;

b)1

1 +1

1 + · · ·

.

OBM

1999. Sendo a 6= b e b 6= 0, sabe-se que as raízes da equação x2 + ax+ b = 0 sãoexatamente a e b. Então, quanto vale a− b?

1998. Qual é a diferença entre a maior raiz e a menor raiz da equação(2x− 45)2 − (x− 21)2 = 0?

2001. Qual é a soma dos valores reais de x tais que x2 + x+ 1 = 156x2+x?

2002. Seja α a maior raiz de x2 + x− 1 = 0. Qual é o valor de α5 − 5α?

2002. Os valores de x, y e z que satisfazem às equações x+ 1y = 5, y+ 1

z = 1

e z + 1x = 2. Quanto vale x+ 3y + 2z?

2004. Qual é conjunto das raízes reais da equação√x+ 2

√x− 1 +

√x− 2

√x− 1 = 2?

2007. O conjunto dos valores de c para os quais a equação√x =

√x+√c

possui solução real está contido em:A) [−1,∞[ ; B) ]−∞, 1]; C) [−3, 2]; D) [−2, 3]; E) Z.

Page 65: OBM por assunto: Os primeiros passos olímpicos · tenha muito a acrescentar na sua vida pois o foco do treinamento é justamente para alcançar esse nível de habilidade. ... Esse

4.4. BINÔMIO DE NEWTON 65

2008. O número inteiro positivo a e o número 1a localizam-se na reta da

seguinte maneira:

Qual é a soma desses dois números?

2014. As raízes da equação x2 − ax + b = 0 são diferentes de zero e são osquadrados das raízes da equação x2 − bx + a = 0. As raízes das equações nãosão necessariamente reais, mas a e b são reais. Quanto vale a?

2015. Um número é dito impadrático quando é raiz de uma equação quadrá-tica com coe�cientes inteiros ímpares. Por exemplo, 1+

√5

2 é impadrático, poisé raiz da equação x2 − x− 1 = 0. Qual dos números a seguir é impadrático?

A)1 +√

3

2; B)

1 +√

5

5; C)

1 +√

6

2; D)

1−√

7

4; E)

1−√

13

6.

2016. Qual das equações abaixo resolve o problema a seguir?"Uma quantidade x de amigos resolveu fazer uma viagem juntos, dividindo

igualmente suas despesas, no total de 6000 reais. Entretanto, na última hora,três dos amigos desistiram e cada um dos que foram viajar teve que arcar comuma despesa extra de 100 reais. Quantos amigos eram?"

A) x2 − 12x = 0.B) x2 − 3x− 180 = 0.C) x2 = 144.D) x2 − 5x+ 6 = 0.E) x2 − 100x+ 6000 = 0.

4.4 Binômio de Newton

Vimos anteriormente como calcular (x + y)n para n = 2 e n = 3. A seguiriremos ver como fazer isso pra n qualquer.

Teorema 3

(x+ y)n =

(n

0

)xny0 +

(n

1

)xn−1y1 + ...+

(n

n− 1

)x1yn−1 +

(n

n

)x0yn.

Prova: Indução. Use que(n+1p+1

)=(np+1

)+(np

).

Exercícios

1. Calcule usando o binômio de Newton (x+y)k com cada k ∈ {2, 3, 4, 5, 10, 12}.

2. Mostre que (a+ b)7 − a7 − b7 = 7ab(a+ b)(a2 + ab+ b2)2.

Page 66: OBM por assunto: Os primeiros passos olímpicos · tenha muito a acrescentar na sua vida pois o foco do treinamento é justamente para alcançar esse nível de habilidade. ... Esse

66 CAPÍTULO 4. ÁLGEBRA

4.5 Polinômios

Esta seção trás problemas que demandam técnicas pouco, ou nada, exploradano ensino tradicional mas, como prova da minha boa intenção, apresentareialguns resultados importantes que tornarão possível um leitura agradável dosproblemas que compõem esta seção.

4.5.1 Conceitos Básicos

Rasamente falando um polinômio é uma expressão do tipo p(x) = anxn + ...+

a1x+ a0.

Os termos a0, a1, · · · , an são chamados os coe�cientes do polinômio. O termoa0 é chamado coe�ciente independente, equanto que termo an é chamado coe-�ciente líder, e é tomado como sendo diferente de 0 para evitar ambiguidades.Se um polinômio tem coe�ciente líder igual a 1, ele é chamado de polinômiomônico. O x é chamado variável, e o maior expoente da variável, no caso acima,o n, é chamado de grau do polinômio e é representado por deg(p). Quandonão �zermos menção sobre os coe�ciente de um polinômio, estaremos dizendoimplicitamente que os coe�cientes são números rais.

Dizemos que dois polinômios f e g são iguais ou idênticos, denotando f ≡ gquando tem os mesmos coe�cientes. Usamos a notação f 6≡ g para indicar queos polinômios não são identicos, ou seja, que possuem algum coe�ciente distinto.Um polinômio f é dito identicamente nulo quando f ≡ 0, ou seja, quando temtodos os coe�cientes nulos.

Algumas vezes é conveniente considerar que o polinômio nulo tem grau −∞,porque isso permite que as contas com respeito ao grau �cam consistentes. Ire-mos proceder assim, nesse texto.

Exemplo: O polinômio q(y) = y3 + 2y2 + 3y + 7 é um polinômio mônico,de grau três na variável y, cujo coe�ciente independente é igual a 7.

Podemos ver um polinômio p(x) = anxn + ... + a1x + a0 como uma função

p : R→ R que leva cada α ∈ R em p(α) = anαn + ...+ a1α+ a0.

Dizemos que f e g são iguais no ponto a quando ao fazermos a substituiçãoda variável por a resulta no mesmo resultado para os dois polinômios, ou seja,f(a) = g(a). Utilizamos a notação ≡ para igualdade de polinômios para evitarconfusão com a igualdade nos pontos especí�cos.

Exemplo: No caso do q(y) acima temos q(5) = 53 + 2 · 52 + 3 · 5 + 7 = 197.

De�nição 2 Uma raiz de um polinômio p(x) é um número a tal que p(a) = 0.

Observe que resolver uma equação do primeiro ou do segundo grau é o mesmoque encontrar as raízes de um polinômio de primeiro ou de segundo grau.

Teorema 4 (Algoritmo de Euclides para polinômios) Sejam f(x) e g(x)dois polinômios com deg(g) ≥ 0. Então existe um único par de polinômios

Page 67: OBM por assunto: Os primeiros passos olímpicos · tenha muito a acrescentar na sua vida pois o foco do treinamento é justamente para alcançar esse nível de habilidade. ... Esse

4.5. POLINÔMIOS 67

q(x), r(x) tais que deg(r) < deg(g) e

f(x) = q(x) · g(x) + r(x).

Um aplicação importante do teorema acima é o

Teorema 5 (D'Alembert) O número a é uma raíz de p(x) se, e somente se,existe um polinômio q(x) tal que p(x) = (x− a)q(x).

Prova: Uma direção é trivial. Para a outra use o Algoritmo de Euclides.

Exemplo (USO): Sejam a, b, c três inteiros positivos distintos, e seja p umpolinômio com coe�cientes inteiros. Mostre que nesse caso não é possível queas três igualdades abaixo sejam satisfeitas:

p(a) = b, p(b) = c, p(c) = a.

Solução: Seja q(x) = p(x)−b. Como q(a) = 0, pelo teorema de D'Alembertexiste um polinômio p1(x) tal que p(x)−b = q(x) = (x−a)p1(x). Analogamentetemos p(x)− b = (x− c)p2(x) e p(x)−a = (x− c)p3(x). Entre os números a, b, cexistem dois cujo módulo da diferença é máximo, suponha que sejam a e c.Então

|a− b| < |a− c|.

Se trocarmos x por c em q(x), temos que

a− b = p(c)− b = (c− a)p1(c)⇒ |a− b||a− c|

= |p1(c)| ≥ 1.

pois p1(c) é inteiro. Daí segue que |a− b| ≥ |a− c|, o que é uma contradição.

Exercícios

1. Calcule p(0) e p(1) onde

a) p(x) = 3x3 + 2x2 + x;

b) p(x) = x4 + 4x3 + 7;

c) p(x) = 2x5 + 2x4 − 3x3 + x2 + 5x− 2.

2. Calcule o resto da divisão de p(x) por q(x)

a) p(x) = 3x4 + 4x3 + 2x2 + x+ 1, q(x) = x2 + 1;

b) p(x) = 2x5 + x3 + x, q(x) = x− 2.

3. Encontre um polinômio cujas raízes são 1, 2, 2017.

4. Encontre um polinômio com coe�cientes inteiros que possua√

2+√

3 comoraiz.

5. Encontre um polinômio tal que para todo x ∈ R vale a equação

(x+ 1)p(x) = (x− 10)p(x+ 1).

6. Seja p(x) um polinômios com coe�cientes inteiros. Se p(a) = p(b) = p(c) =−1, com a, b, c ∈ Z, então p(x) não possui raiz inteira.

Page 68: OBM por assunto: Os primeiros passos olímpicos · tenha muito a acrescentar na sua vida pois o foco do treinamento é justamente para alcançar esse nível de habilidade. ... Esse

68 CAPÍTULO 4. ÁLGEBRA

4.5.2 Teorema Fundamental da Álgebra

Teorema 6 (Fundamental da Álgebra) Todo polinômio com coe�cientes com-plexos (em particular com coe�cientes reais) possui pelo menos uma raíz com-plexa.

Prova: Omitida.

A demonstração desse teorema, cujo enunciado é bastante simples requerconhecimentos superiores de matemática (onde de�nimos conhecimento supe-rior como aquele que se obtém no ensino superior). Costuma-se brincar queo Teorema Fundamental da Álgebra (TFAl) nem é fundamental nem é de ál-gebra, este último vem do fato que suas demonstrações usuais são da área damatemática conhecida como Análise, que é o estudo das aproximações.

Teorema 7 (Fundamental da Álgebra, segunda versão) Se p(x) é um po-linômio de grau n, então podemos escrever p(x) = an(x−x1)(x−x2) · · · (x−xn),onde an é o coe�ciente líder de p e x1, x2, · · · , xn são as raízes (não necessari-amente distintas de p(x)).

Prova: Use o TFAl e indução.

Exemplo:Existem poliômios que não possuem raiz real, por exemplo x2 + 1. É fácil

produzir mais polinômios que não tem raiz real, criando equações do segundograu com o ∆ negativo. Se multiplicarmos dois polinômios que não possuemraízes reais o produto não terá raiz real, assim, podemos criar polinômio dequalquer grau par sem raízes reais.

Quanto a polinômios de grau ímpar temos o seguinte :

Teorema 8 Se p(x) é um polinômio com grau ímpar e coe�cientes reais, entãop(x) possui uma raíz real.

Proposição 2 Sejam p(x) = anxn + an−1x

n−1 + · · ·+ a1x+ a0. Então a soma

das raízes de p é −an−1

ane o produto das raízes de p é

a0

an.

Prova: Use a segunda versão do TFAl.

Exercício: Se deg(p) ≤ n e p possui n+ 1 raízes, então p ≡ 0.

Teorema 9 (Princípio da Identidade) Sejam p e q dois polinômios de graun, e suponha que existam n+ 1 números a1 < · · · < an+1 tais que p(ai) = q(ai)1 ≤ i ≤ n+ 1, então p ≡ q.

Prova: Use o exercício acima.

Exercícios

1. Dê um exemplo de polinômio com coe�ciente reais que não possui raízreal.

2. Dê dois exemplos de polinômios com grau par que possuem raiz real.

Page 69: OBM por assunto: Os primeiros passos olímpicos · tenha muito a acrescentar na sua vida pois o foco do treinamento é justamente para alcançar esse nível de habilidade. ... Esse

4.5. POLINÔMIOS 69

3. Se p(x) e q(x) são dois polinômios mônicos de grau n que possuem asmesmas raízes (com as mesmas multiplicidades) então eles são iguais.

4. Dê exemplos de polinômios distintos de mesmo grau que possuem as mes-mas raízes.

5. Encontre dois polinômios p 6≡ q tais que p(i) = q(i) para i ∈ {1, 2, 3}.

6. Encontre todos os polinômios p(x) tais que p(n) = 2n para qualquer n ∈ N.

7. Determine todos os polinômios p(x) tais que p(x2 + 1) = (p(x))2 + 1 ep(0) = 0.

OBM

1998. Qual é a soma das raízes reais de x3 + 3x2 + 3x− 1 = 0?

2000. Seja P (x) = a2000x2000 + a19999x1999 + a1998x

1998 + · · · + a1x + a0.Então a2000 + a1998 + a1996 + · · ·+ a0 é igual a

A)P (1)−P (−1)2 . B)P (1)+P (−1)

2 . C) P (2000) + P (1998) + · · ·+ P (0).D)P (0) · P (1). E) P (−1) · P (1).

2013. Um polinômio p(x) é par quando p(−x) = p(x), para todo x real.Qual é o número máximo de soluções reais da equação p(x) = k, sendo p umpolinômio par não constante com coe�cientes não negativos e k um real �xado?

2014. Quanto vale a soma das raízes da equação 11+x + 2

2+x + 33+x = 1?

2015. O polinômio não constante P (x) tem coe�cientes inteiros e é tal queP (0) = 2015. No máximo quantas raízes inteiras distintas tem P (x)?

Page 70: OBM por assunto: Os primeiros passos olímpicos · tenha muito a acrescentar na sua vida pois o foco do treinamento é justamente para alcançar esse nível de habilidade. ... Esse

70 CAPÍTULO 4. ÁLGEBRA

Page 71: OBM por assunto: Os primeiros passos olímpicos · tenha muito a acrescentar na sua vida pois o foco do treinamento é justamente para alcançar esse nível de habilidade. ... Esse

Capítulo 5

Análise

A análise pode ser vista como o estudo das aproximações, em muito caso, decomo podemos compreender funções por meio das suas aproximações. Essasaproximações são realizadas por meio de desigualdades. No contexto mais amploda matemática, a análise além de ser um objeto interessante por si só, é umadas ferramentas mais relevantes na formalização dos conceitos em geral, comono caso da probabilidade e da geometria.

"É mais de oito mil!"- Vegeta.

5.1 Desigualdes

Em alguns problemas não é viável encontrar a solução exata e basta que o re-sultado seja aproximado por algo mais simples e neste sentido as desigualdadesdesempenham um papel importante! Apresentaremos nesta seção algumas de-sigualdades essenciais para o avanço nesta jornada em busca de experiência naresolução de problemas!

5.1.1 Exercícios de aquecimento

Exemplo:

a) Encontre n ∈ N tal que√n+ 1−

√n < 1

100 ;

b) Seja ε > 0, encontre n ∈ N tal que√n+ 1−

√n < ε;

c) Mostre que se n ∈ N então√n+ 2−

√n+ 1 <

√n+ 1−

√n.

Solução:

71

Page 72: OBM por assunto: Os primeiros passos olímpicos · tenha muito a acrescentar na sua vida pois o foco do treinamento é justamente para alcançar esse nível de habilidade. ... Esse

72 CAPÍTULO 5. ANÁLISE

Observe que

√n+ 1−

√n = (

√n+ 1−

√n)

√n+ 1 +

√n√

n+ 1 +√n

=n+ 1− n√n+ 1 +

√n

=1√

n+ 1 +√n.

Além disso√n+ 1 >

√n⇒

√n+ 1 +

√n > 2

√n

⇒ 1√n+ 1 +

√n<

1

2√n.

a) Agora, podemos procurar n tal que1

2√n< 1

100 , que é

1

2√n<

1

100⇐⇒ 1√

n<

1

50

⇐⇒√n > 50

⇐⇒ n > 2500.

b) Dado ε > 0 qualquer basta buscarmos1

2√n< ε, que é

1

2√n< ε ⇐⇒ 1√

n< 2ε

⇐⇒√n >

1

⇐⇒ n >1

4ε2.

O que provamos foi que não importa o quão pequeno seja ε, existe um n talque√n+ 1−

√n < ε.

c) Para o último item

√n+ 2−

√n+ 1 <

√n+ 1−

√n ⇐⇒

√n+ 2 +

√n < 2

√n+ 1

⇐⇒ (√n+ 2 +

√n)2 < (2

√n+ 1)2

⇐⇒ n+ 2 + 2√n+ 2 ·

√n+ n < 4(n+ 1)

⇐⇒√n+ 2

√n < n+ 1

⇐⇒ (n+ 2)n < (n+ 1)2

⇐⇒ n2 + 2n < n2 + 2n+ 1

⇐⇒ 0 < 1.

Como 0 < 1 segue que os passos anteriores são verdade, e assim√n+ 2−

√n+ 1 <

√n+ 1−

√n,

como queríamos.

Page 73: OBM por assunto: Os primeiros passos olímpicos · tenha muito a acrescentar na sua vida pois o foco do treinamento é justamente para alcançar esse nível de habilidade. ... Esse

5.1. DESIGUALDES 73

Exercícios

1. Sejam x e y números reais. Mostre

a) a Desigualdade Triângular, isto é,

|x+ y| ≤ |x|+ |y|;

b) que |x− y| ≥ |x| − |y| e |x− y| ≥ ||x| − |y||.

2. Mostre que para n ≥ 3(1 +

1

n

)n<

(1 +

1

n+ 1

)n+1

.

Dica: Use a expansão em binômio de Newton.

OBM

1998. Quantos são os números inteiros x que satisfazem à inequação 3 <√x <

7?

2000. Sejam a e b números reais positivos tais quea

b< 1. Então

a+ 1

b+ 1

A) é igual aa

b+ 1. B) é igual a

a

b. C) é menor que

a

b.

D) é maior quea

bmas menor que 1. E) pode ser maior que 1.

2002. Uma escola vai organizar um passeio ao zoológico. Há duas opções detransporte. A primeira opção é alugar "vans": cada van pode levar até 6 crian-ças e seu aluguel custa R$60, 00. A segunda opção é contratar uma empresa parafazer o serviço: a empresa usa ônibus com capacidade para 48 crianças e cobraR$237, 00, mais R$120, 00 por ônibus utilizado. A escola deve preferir a em-presa de ônibus se forem ao passeio pelo menos N crianças. Qual é o valor de N?

2003. O número 19AB, onde A e B são dígitos, é um quadrado perfeito.Qual é o valor de

√AB da raiz quadrada do número cuja representação decimal

é AB?

2005. Sejam A = 10(log10 2005)2 , B = 20053 e C = 2√

2005 . EntãoA) A < B < C.B) A < C < B.C) B < A < C.D)B < C < A.E) C < A < B.

2005. A �gura mostra um cubo de aresta 1 no qual todas as doze diagonaisde face foram desenhadas. Com isso, criou-se uma rede com 14 vértices (os 8vértices do cubo e os 6 centros de faces) e 36 arestas (as 12 arestas do cubo emais 4 sobre cada uma das 6 faces). Qual é o comprimento do menor caminhoque é formado por arestas da rede e que passa por todos os 14 vértices?

Page 74: OBM por assunto: Os primeiros passos olímpicos · tenha muito a acrescentar na sua vida pois o foco do treinamento é justamente para alcançar esse nível de habilidade. ... Esse

74 CAPÍTULO 5. ANÁLISE

2006. O inteiro positivo x é múltiplo de 2006 e√x está entre 2005 e 2007.

Qual é o número de possíveis valores de x?

2006. Qual é o menor valor que a expressão√x2 + 1 +

√(y − x)2 + 1 +

√(z − y)2 + 1 +

√(10− z)2 + 9

pode assumir, sendo x, y e z reais?

2008. Sendo x = 10−2008, assinale a alternativa que apresenta o maior valor.A) 1

x . B) 1x(x+1) . C) 1

1+ 1

1+ 1x

. D) x. E) xx+ 1

x

.

2010. Qual das seguintes frações é mais próxima de√

7 ?A) 3

1 . B) 52 . C) 8

3 . D) 135 . E) 18

7 .

2010. Os números a e b são reais não negativos tais que a3 + a < b − b3.Então

A) b < a < 1. B) a = b = 1. C) a < 1 < b. D) a < b < 1. E) 1 < a < b.

2010. Sendo n = 20102010, entãoA) n! < nlogn < (log n)n.B) nlogn < n! < (log n)n.C) (log n)n < nlogn < n!.D) (log n)n < n! < nlogn.E) nlogn < (log n)n < n!.

2010. Qual é o maior valor de xy2 se x e y são reais positivos cuja soma é 3?

2010. Qual é o menor valor positivo de 21m2 − n2 para m e n inteiros posi-tivos?

2011. Numa padaria, uma lata de 200g de achocolatado em pó CHOCOBMcusta R$3, 00, uma lata de 400g custa R$5, 00 e a de 800g custa R$9, 00. Laraprecisa de 1, 2kg de CHOCOBM para fazer um enorme bolo. Qual das opçõesa seguir é a maneira mais econômica de comprar 1, 2kg de CHOCOBM nessapadaria?

A) 6 latas de 200g.B) 1 lata de 400g e 1 lata de 800g.C) 4 latas de 200g e 1 lata de 400g.D) 2 latas de 200g e 1 lata de 800g.

Page 75: OBM por assunto: Os primeiros passos olímpicos · tenha muito a acrescentar na sua vida pois o foco do treinamento é justamente para alcançar esse nível de habilidade. ... Esse

5.1. DESIGUALDES 75

E) 2 latas de 200g e 2 latas de 400g.

2011. Sendo a e b reais tais que 0 < a ≤ 1 e 0 < b ≤ 1, qual é o maior valorque ab

a+b pode assumir?

2011. A calculadora de Esmeralda está quebrada: quando ela aperta o botão"√", a calculadora faz, ao acaso, uma das duas seguintes operações: tirar a raizquadrada (como deveria fazer) ou dividir o número por 100 (como não deveriafazer). Esmeralda digitou o número 201120112011 na calculadora e começou aapertar o botão repetidamente. Quantas vezes, no máximo, Esmeralda apertao botão "√"até aparecer pela primeira vez um número menor que 2?

2012. Na expressão M×A×T×E×MA×T×I×C×A , letras diferentes representam dígitos di-

ferentes e letras iguais representam dígitos iguais. Qual é o maior valor possíveldesta expressão?

2012, Para Mariazinha, existem somente quatro números que ela consideraatraentes: 1, 3, 13e31. Um outro número será quase atraente somente se puderser expresso como soma de pelo menos um de cada um dos quatro númerosatraentes. Por exemplo, 1 + 3 + 3 + 3 + 13 + 31 = 54 é quase atraente. Nomínimo, quantos números atraentes devem ser somados para mostrar que 2012é um número quase atraente?

2013. Qual dos seguintes números é o mais próximo da quantidade de alga-rismos de 3400?

A) 100. B) 150. C) 200. D) 240. E) 300.

2013. Para quantos inteiros positivos k menores que 2013, existem inteirosa, b e c, não necessariamente distintos, satisfazendo

a2 + b+ c = b2 + c+ a = c2 + a+ b = k?

2014. Assinale a alternativa que apresenta o maior dos cinco números.A)20145. B)30154. C)20163. D)50172. E)60181.

2015. Inicialmente, na tela de um computador, estão escritos os números1 e 2. A cada segundo, esses dois números são trocados pela soma de seusquadrados e pelo dobro de seu produto. Depois de aproximadamente quantotempo um desses dois números vai ser maior do que a quantidade de átomos noplaneta Terra, que é cerca de 1050?

A) Sete segundos.B) Sete horas.C) Sete dias.D) Sete meses.E) Sete anos.

2016. A soma de 2016 números inteiros positivos é maior ou igual ao produtodos mesmos 2016 números inteiros positivos. Pelo menos quantos desses númerossão iguais a 1?

Page 76: OBM por assunto: Os primeiros passos olímpicos · tenha muito a acrescentar na sua vida pois o foco do treinamento é justamente para alcançar esse nível de habilidade. ... Esse

76 CAPÍTULO 5. ANÁLISE

5.1.2 Desigualdade entre as médias

Como sabemos se x ∈ R, temos que x2 ≥ 0. Essa desigualdade será a base daprova da desigualdade entre a média aritimética (MA) e média geométrica (MG).Quem dará a prova para essa desigualdade será o próprio leitor. A seguir temosuma lista de exercícios que levarão o leitor a ter familiaridade com o métodode prova, começando pelo caso mais simples, onde só temos dois elementos namédia, e complexi�cando aos poucos. No �m, enunciaremos a versão geral quepode ser provada usando as mesmas ideias aprendidas nos exercícios.

Exercícios

1. Use x = (√a−√b), com a, b > 0 para provar que

a+ b

2≥√ab.

2. Aplique a desigualdade acima duas vezes para mostrar que se a, b, c, d > 0então

a+ b+ c+ d

4≥ 4√abcd.

3. a) Use d = 3√abc;

b) Use d = a+b+c3 ;

na desigualdade acima e mostre que se a, b, c > 0, então

a+ b+ c

3≥ 3√abc.

4. Mostre que (a+ b)(b+ c)(c+ a) ≥ 8abc.

5. Mostre que a2 + b2 + c2 ≥ ab+ bc+ ca.

6. Mostre que a média geométrica é maior ou igual a média harmônica(MG ≥ MH), ou seja,

√ab ≥ 2

1a + 1

b

.

7. Mostre que

a) (x1y1 + x2y2)2 ≤ (x21 + x2

2)(y21 + y2

2),

b) (x1y1 + x2y2 + x3y3)2 ≤ (x21 + x2

2 + x23)(y2

1 + y22 + y2

3).

Teorema 10 (Desigualdade MA ≥ MG) Sejam a1, ..., an > 0, então

a1 + a2 + ...+ ann

≥ n√a1a2...an.

Prova: Exercício. Indução forte: suponha que provou pra todos os valores≤ 2k, prove para n = 2k + 2 e em seguida para n = 2k + 1 (use as idéias den = 4 e depois de n = 3).

Exercício: Mostre a desigualdade MG ≥MH geral:

n√a1 · · · an ≥

n1a1

+ · · ·+ 1an

.

Page 77: OBM por assunto: Os primeiros passos olímpicos · tenha muito a acrescentar na sua vida pois o foco do treinamento é justamente para alcançar esse nível de habilidade. ... Esse

5.1. DESIGUALDES 77

OBM

1998. Quais são os valores reais de x que satisfazem a inequação

√x+

√1

x≤ 2?

2000. Se x e y são números reais positivos, qual dos números a seguir é omaior?

A) xy B) x2 + y2 C) (x+ y)2 D)x2 + y(x+ y) E)x3 + y3

x+ y

2009. Sabe-se que 2x2 − 12xy + ky2 ≥ 0 para todos x, y reais. Qual é omenor valor real de k?

2012. A soma de dois inteiros positivos é 2012. Qual é a diferença entre omaior e o menor valores possíveis do produto dos dois números?

5.1.3 Desigualdade de Cauchy-Schwarz

Teorema 11 (Desigualdade de Cauchy-Schwarz) Sejam x1, · · · , xn, y1, · · · , ynreais quaisquer, vale a desigualdade

(x1y1 + x2y2 + · · ·+ xnyn)2 ≤ (x21 + x2

2 + · · ·+ x2n)(y2

1 + y22 + · · ·+ y2

n).

Prova:

(x1y1 + x2y2 + · · ·+ xnyn)2 =

n∑i=1

x2i y

2i +

∑i<j

2xiyixjyj

≤n∑i=1

x2i y

2i +

∑i<j

(x2i y

2j + x2

jy2i )

=

n∑i=1

n∑j=1

x2i y

2j

= (x21 + x2

2 + · · ·+ x2n)(y2

1 + y22 + · · ·+ y2

n).

Exercícios

1. Considere a função f(x) = (a1x+ b1)2 + (a2x+ b2)2 + · · · (anx+ bn)2

a) Qual é o valor mínimo que essa função pode atingir?

b) Escreva a equação do segundo grau f(x) = 0 na forma ax2 +bx+c = 0.

c) Calcule o ∆ dessa equação.

d) Prove que o ∆ ≤ 0.

e) Prove a Desigualdade de Cauchy Schwarz usando os itens anteriores.

2. Mostre que MA ≥MH, ou seja,

a1 + · · ·+ ann

≥ n1a1

+ · · ·+ 1an

.

Page 78: OBM por assunto: Os primeiros passos olímpicos · tenha muito a acrescentar na sua vida pois o foco do treinamento é justamente para alcançar esse nível de habilidade. ... Esse

78 CAPÍTULO 5. ANÁLISE

3. Mostre que1

a+

1

b+

4

c+

16

d≥ 64

a+ b+ c+ d.

4. Se a1 + a2 + · · ·+ an = n, então a41 + a4

2 + · · ·+ a4n ≥ n.

5. Se p1, p2, · · · , pn, x1, x2, · · · , xn são 2n números positivos, então

(p1x1 + · · ·+ pnxn)2 ≤ (p1 + · · ·+ pn)(p1x21 + · · ·+ pnx

2n).

6. Mostre que se a, b, c ≥ 0, então

(a2b+ b2c+ c2a)(ab2 + bc2 + ca2) ≥ 9a2b2c2.

7. Prove a desigualdade de Mikowski√√√√ n∑i=1

(ai + bi)2 ≤

√√√√ n∑i=1

a2i +

√√√√ n∑i=1

b2i .

Observação: A desigualdade de Minkowski vale em contextos muito maisgerais, esse é apenas um caso particular. Se você deseja um caso um pouco maisgeral e outras desigualdades interessantes consulte a Revista Eureka! número 5.

5.2 Função Parte Inteira

Muito útil e algumas vezes esquecidas! Apresentaremos a função parte inteirae alguns resultados importantes sobre esta função.

De�nição 3 A parte inteira de um número x ∈ R, bxc, é o maior inteiro ntal que n ≤ x < n+ 1.

Podemos então ver parte inteira como uma função b·c : R → Z que levaum número x na parte inteira de x, bxc, ou seja, no maior inteiro n tal quen ≤ x < n + 1. Isso signi�ca, em particular, que se m ∈ Z e m ≤ x entãom ≤ bxc.

Além da parte inteira temos também a parte fracionária de um númerodenotada e de�nida pela expressão abaixo

{x} := x− bxc.

Exemplos:1. Observe que se x > 0, bxc é o número antes da vírgula na representação

decimal de x, por exemplo b2, 5c = 2, bπc = 3.2. No entanto se x < 0 a parte inteira não é o número que vem antes da

vírgula, e sim esse número subtraído de 1, pois bxc ≤ x,∀x ∈ R. Assim temospor exemplo que b−2, 5c = −3.

Em geral, quando se deseja fazer cálculos envolvendo parte inteira, trabalha-se com a desigualdade da de�nição, ou seja, com o fato de que bxc ≤ x < bxc+1

Page 79: OBM por assunto: Os primeiros passos olímpicos · tenha muito a acrescentar na sua vida pois o foco do treinamento é justamente para alcançar esse nível de habilidade. ... Esse

5.2. FUNÇÃO PARTE INTEIRA 79

e com o fato de que bxc ∈ Z.

Exemplo: Calcule b√πc.

Como 1 ≤ π < 4 temos 1 =√

1 ≤√π <√

4 = 2, logo b√πc = 1.

A seguir veremos algumas propriedades e �cará explícito o que queremosdizer com o parágrafo anterior.

Proposição 3 (Propriedades da parte inteira) Dado x, y ∈ R e n ∈ Nvalem as seguintes expressões

1. x > y ⇒ bxc ≥ byc

2. bx+ nc = bxc+ n;

3. bx+ yc ≥ bxc+ byc;

4. bn · xc ≥ n · bxc;

5. bxcn ≥ bxnc.

Prova:Sabemos que m = bxc se e somente se m ≤ x < m + 1 e que bxc é o maior

inteiro que não excede x.

1. Observe que byc ∈ Z e byc ≤ y < x. Logo bxc ≥ byc.

2. Observe que bxc ≤ x ≤ bxc+ 1⇒ bxc+ n ≤ x+ n ≤ (bxc+ n) + 1, alémdisso bxc+ n ∈ Z. Portanto bx+ nc = bxc+ n.

3. Exercício.

4. Use o item acima e indução.

5. Use o item acima.

Teorema 12 (Fórmula de Polignac) Seja p um número primo. A maiorpotência m de p tal que pm|n! é dada por

m =

∞∑j=1

⌊n

pj

⌋.

Exemplo:Determine com quanto zeros termina 1000!.

Solução:O problema é equivalente a determinar qual é a maior potência de 10 que

divide 1000! e como há mais fatores 2 do que fatores 5, o expoente da maiorpotência de 10 coincide com o expoente da maior potência de 5 que divide 1000!,ou seja, ⌊

1000

5

⌋+

⌊1000

52

⌋+

⌊1000

53

⌋+

⌊1000

54

⌋= 249.

Page 80: OBM por assunto: Os primeiros passos olímpicos · tenha muito a acrescentar na sua vida pois o foco do treinamento é justamente para alcançar esse nível de habilidade. ... Esse

80 CAPÍTULO 5. ANÁLISE

Observe que para potências com expoente maior que k > 4 teremos 0 < 10005k

<

1, logo b 10005kc = 0 (por uma razão análoga a essa a soma terá sempre uma

quantidade �nita de termos). Assim 1000! termina em 249 zeros.

OBM

2000. Qual é número de inteiros positivos x para os quais bx 12 c+ bx 1

3 c = 10?

2003. Calcule⌊

22003 + 32003

22001 + 32001

⌋.

2007. Se x + byc + {z} = 4, 2 , y + bzc + {x} = 3, 6 e z + bxc + {y} = 2 ,quanto vale x− y + z?

2008. Qual o número de soluções reais do sistema

x · |x|+ y · |y| = 1 e bxc+ byc = 1?

2015. Qual menor valor de n para o qual n! termina em 2016 zeros?

2016. Quantas soluções tem a equação bxc − 2016{x} = 38?

5.3 Sequências

Tão natural quanto contar é colocar as coisa em sequência! Iniciamos a seçãoapresentando problemas que exigem apenas intuição e em seguida relembramosdois tipos de sequências mais populares.

5.3.1 OBM

2003. Considere a seqüência oscilante:

1, 2, 3, 4, 5, 4, 3, 2, 1, 2, 3, 4, 5, 4, 3, 2, 1, 2, 3, 4, · · ·

Qual é o 2003o termo desta seqüência?

2006. Uma seqüência tem 9 números reais, sendo o primeiro 20 e o último6. Cada termo da seqüência, a partir do terceiro, é a média aritmética de todosos anteriores. Qual é o segundo termo da seqüência?

2007. Seja {an} uma seqüência na qual cada termo é de�nido como o dobroda soma dos algarismos do termo anterior, mais uma unidade. Por exemplo,se an = 234, então an+1 = 2(2 + 3 + 4) + 1. Se, a1 = 1 qual é o valor dea31 + a32 + a33 + a34 + a35?

2014. Uma sequência xn tem como primeiros termos x0 = x1 = 2 e os de-mais termos de�nidos por xn+2 = 2xn+1 +xn. Qual é o valor de x0−x1 +x2−x3 + · · · − x2013 + x2014?

Page 81: OBM por assunto: Os primeiros passos olímpicos · tenha muito a acrescentar na sua vida pois o foco do treinamento é justamente para alcançar esse nível de habilidade. ... Esse

5.3. SEQUÊNCIAS 81

5.3.2 Progressões Aritmética e Geométrica

Os primeiros exemplos de recorrências que nos são apresentados na escola (etalvez os únicos) são as progressões aritméticas e geométricas (PA e PG).

Começaremos resolvendo o clássico problema de calcular a soma dos n pri-meiro números naturais. Esse problema é lendário, procure sua história e nãoconseguirá dormir em paz hoje.

Exemplo: Calcule S = 1 + 2 + 3 + ...+ 100.

Solução de Gauss: Observe que S = 100 + 99 + ... + 3 + 2 + 1. Entãosomando as duas expressões "verticalmente" temos

S = 1 + 2 + · · ·+ 99 + 100

+S = 100 + 99 + · · ·+ 2 + 1

2S = 101 + 101 + · · ·+ 101 + 101 = 100 · 101 = 10100

Logo S = 5050.

Exercício Calcule Sn = 1 + 2 + · · ·+ n.

De�nição 4 Uma PA é uma sequência com um termo inicial a1 e tal que paracada n ∈ N, an+1 = an + r, onde r ∈ R.

Exercício a) Considere a PA com a1 = 7 e r = 4. Calcule S23 = a1 + · · ·+a23.

b) No caso acima, determine Sn = a1 + · · ·+ an.c) Calcule a soma dos n primeiros termos de uma PA de razão r e termo

inicial a1: Sn = a1 + ...+ an.

De�nição 5 Uma PG é uma sequência com um termo inicial a0 e tal que paracada n ∈ N, an = an−1q, onde q ∈ R.

Exemplo: Calcule Sn = 1 + 2 + 22 + 23 + ...+ 2n.

Solução: Observe que

Sn + 2n+1 = 1 + 2 + 22 + 23 + ...+ 2n + 2n+1 = 1 + 2Sn,

logo Sn = 2n+1 − 1.

Exercícios

1. Calcule 1− 22 + 32 − · · ·+ (2n− 1)2 − 2n2.

2. Mostre que ∀n ∈ N vale 12018 + 22018 + · · ·+ n2018 <n2019

2.

3. Seja a ∈ R. Calcule Sn = 1 + a+ a2 + a3 + ...+ an.

Page 82: OBM por assunto: Os primeiros passos olímpicos · tenha muito a acrescentar na sua vida pois o foco do treinamento é justamente para alcançar esse nível de habilidade. ... Esse

82 CAPÍTULO 5. ANÁLISE

4. Sejam a, b ∈ R. Mostre que

an − bn = (a− b)(an−1 + an−2b+ an−3b2 + · · ·+ abn−2 + bn−1).

Dica: an − bn = bn((ab )n − 1).

5. Calcule a soma dos termos de uma PG: Sn = a0 + a1 + ...+ an.

6. a) Mostre que 2n ≤ n!,∀n ≥ 4.

b) Dado n ∈ N mostre que 10! + 1

1! + 12! + · · ·+ 1

n! < 3.

7. Mostre que existe n ∈ N tal que 11 + 1

2 + 13 + · · ·+ 1

n > 100.

8. Sejam a1 = 5 e an = 3an−1 + 3. Calcule

S2017 = a0 + a1 + ...+ a2017.

Dica: bn = an3n .

9. Sejam a1 = 4 e an = 4an−1 + 7. Calcule∑mk=1 ak.

10. (Moldávia 1998) A sequência (an), n ∈ N veri�ca as relações a1 =1

2,

an =an−1

2nan−1 + 1para todo número natural n > 1 . Calcule

a1 + a2 + ...+ a1998.

Dica: bn = 1an.

11. (Putnam) Sejam x1, x2, x3, · · · uma sequência de números reais não ne-

gativos satisfazendo xn =xn−2xn−1

2xn−2 − xn−1para n = 3, 4, 5, · · · . Estabeleça

condições necessárias e su�cientes em x1 e x2 para xn ser inteiro parain�nitos valores de n.

12. (Cone Sul 1996) Considere uma sequência de números reais de�nidapor:

an+1 = an +1

anpara n = 0, 1, 2, · · ·

Demostre que, qualquer que seja o número real positivo a0, tem-se quea1996 é maior que 63.Dica: bn = a2

n.

13. (Lista Cone Sul 2016) Seja {an}n≥1 uma sequência tal que a1 = 4 e,para todo n inteiro positivo:

an+1 =an2

+2

an.

Determine an em função de n.

Page 83: OBM por assunto: Os primeiros passos olímpicos · tenha muito a acrescentar na sua vida pois o foco do treinamento é justamente para alcançar esse nível de habilidade. ... Esse

5.4. SÉRIES 83

OBM

2001. São escritos todos os números de 1 a 999 nos quais o algarismo 1 apareceexatamente 2 vezes (tais como, 11, 121, 411, etc). Qual é a soma de todos estesnúmeros?

2005. Dois números inteiros são chamados de primanos quando pertencem auma progressão aritmética de números primos com pelo menos três termos. Porexemplo, os números 41 e 59 são primanos pois pertencem à progressão aritmé-tica (41; 47; 53; 59) que contém somente números primos. Assinale a alternativacom dois números que não são primanos.

A) 7 e 11B) 13 e 53C) 41 e 131D) 31 e 43E) 23 e 41

2009. Para cada número natural n, seja Sn a soma dos dez primeiros múlti-plos positivos de n. Por exemplo, S2 = 2+4+6+8+10+12+14+16+18+20.Quanto é S1 + S2 + S3 + · · ·+ S10?

2013. O triângulo aritmético de Fibonacci é formado pelos números ímparesinteiros positivos a partir do 1 dispostos em linhas com ordem crescente em cadalinha e pulando para a linha seguinte. A linha n possui exatamente n números.Veja as quatro primeiras linhas.

Linha1 :1

Linha2 :3 5

Linha3 :7 9 11

Linha4 :13 15 17 19

· · ·

Em qual linha aparecerá o 2013?

5.4 Séries

Tem alguns que gostam de �lmes e outros de SÉRIES... Não duvido que vocêsaiba somar mas não custa conhecer técnicas que ajudam bastante na tarefa desomar.

5.4.1 Somas telescópicas

Uma soma telescópica é uma expressão do tipo

n∑k=1

[f(k)−f(k−1)] = (f(1)−f(0))+(f(2)−f(1))+· · ·+(f(n)−f(n−1)) = f(n)−f(0).

Agora você deve estar dizendo pra si mesmo "isso é muito trivial: o resultadoé simplesmente f(n) − f(0)". Pois é, não é tãaaaao trivial. A questão é que a

Page 84: OBM por assunto: Os primeiros passos olímpicos · tenha muito a acrescentar na sua vida pois o foco do treinamento é justamente para alcançar esse nível de habilidade. ... Esse

84 CAPÍTULO 5. ANÁLISE

função f não surgirá explicitamente nos exercícios e problemas. O desa�o serájustamente encontrar quem é a tal da f . Veja o

Exemplo:Calcule

Sn =

n∑k=1

1

k(k + 1).

Solução:Este tipo de soma telescópica é o que vem com uma coisa que chamamos

uma fração que pode ser decomposta como frações parciais. Observe que nanossa fração temos um produto de monômios que nós escreveremos como umasoma de monômios

1

k(k + 1)=A

k+

B

k + 1,

o desa�o será encontrar A e B que tornem essa a�rmação verdade independen-temente de qual for o valor de k. Vamos expandir

1

k(k + 1)=A

k+

B

k + 1⇐⇒

1 = (k + 1)A+ kB ⇐⇒1 = k(A+B) +A

Uma maneira de fazer com que a expressão acima não dependa de k é fazendocom que o termo que o acompanha seja nulo, ou seja, A+B = 0, o que obrigaA a ser igual a 1. Assim, fazendo A = −B = 1 vemos que

1

k(k + 1)=

1

k− 1

k + 1,

e assim

n∑k=1

1

k(k + 1)=

n∑k=1

1

k− 1

k + 1

e agora é fácil ver que

Sn = 1− 1

n=n− 1

n.

Perceba que no caso acima, a função (ou sequência) é f(k) = −1k . Dizer

explicitamente quem é a cara da função não é estritamente necessário, tanto éque não falamos sobre a cara da função até que o problema já estivesse com-pletamente resolvido, e só o mencionamos por uma questão didática, para que�que explícito para você leitor o que está acontecendo.

A seguir, deixamos alguns exercícios que envolvem as ideias de somas eprodutos telescópicos e que envolvem a ideia de decompor uma fração em fraçõesparciais.

Page 85: OBM por assunto: Os primeiros passos olímpicos · tenha muito a acrescentar na sua vida pois o foco do treinamento é justamente para alcançar esse nível de habilidade. ... Esse

5.4. SÉRIES 85

Exercícios

1. Escreva as frações abaixo como somas de frações cujos denominadores sãopolinômios de grau 1:

a)1

x(x+ 2);

b)1

x(x+ 1)(x+ 2);

c)1

(3x+ 1)(4x+ 3).

2. Calcule os somatários a seguir

a)n∑k=1

1

(3k − 2)(3k + 1).

b)n∑k=1

2k + 1

k(k + 1)(k + 2).

3. Calcule os produtórios

a)n∏k=1

(1− 1

k

);

b)n∏k=1

k3 − 1

k3 + 1.

4. Calcule

a)n∑k=1

k2; (Dica: Tome f(k) = k3.)

b)n∑k=1

k3;

c) 12! + 2

3! + · · · 20172018! .

5. Dado n ∈ N mostre que

a) 11.2 + 1

2.3 + 13.4 + · · ·+ 1

n.(n+1) < 1;

b) 112 + 1

22 + 132 + · · ·+ 1

n2 < 2.

OBM

2007. Qual dos inteiros positivos abaixo satisfaz a seguinte equação:

4

n4+

5

n4+

6

n4+ · · ·+ n− 6

n4+n− 5

n4+n− 4

n4= 309?

A) 2007. B) 309. C) 155. D) 25. E) 5.

2010. Para cada subconjunto A de B = {1, 2, 3, 4, 5, 6, 7, 8, 9, 10}, seja p(A)o produto de seus elementos. Por exemplo, p(1, 2, 4, 5) = 40 e p(B) = 10! =1 · 2 · 3 · · · 10. Por convenção, adote p(∅) = 1. Quanto vale a soma de todos os

Page 86: OBM por assunto: Os primeiros passos olímpicos · tenha muito a acrescentar na sua vida pois o foco do treinamento é justamente para alcançar esse nível de habilidade. ... Esse

86 CAPÍTULO 5. ANÁLISE

210 produtos p(A)?

2012. O número e, uma das constantes mais importantes da Matemática,pode ser de�nido por:

e =

∞∑n=0

1

n!=

1

0!+

1

1!+

1

2!+

1

3!· · ·

Quanto vale o número

∞∑n=0

(n+ 1)2

n!=

12

0!+

22

1!+

32

2!+

42

3!· · ·?

2014. A sequência de Fibonacci é de�nida recursivamente por Fn+2 = Fn+1+Fn para n ∈ Z e F1 = F2 = 1. Então(

1− F 22

F 23

)·(

1− F 23

F 24

)·(

1− F 24

F 25

)· · · · ·

(1− F 2

2003

F 22014

)é igual a:

A)F2016

F 22013

. B)F2014

F2013. C)

F 22015

F 22013

. D)F2015

2. E)

F2015

2F2013F2014.

2015. Sabendo que

∞∑n=1

1

n2=

1

12+

1

22+

1

32+ · · · = π2

6

quanto vale∞∑n=1

1

n2(n+ 1)2=

1

1222+

1

2232+

1

3244+ · · ·?

5.4.2 Somas com Números Binomiais

A seguir daremos exemplos para alertar o leitor de ideias que podem ser utili-zadas para resolver somatórios que envolvem números binomiais:

Exemplo Calculen∑k=0

(n

k

).

Sabemos que (x+ y)n =∑nk=0

(nk

)xn−kyk, logo

n∑k=0

(n

k

)= (1 + 1)n = 2n.

Exemplo Calculen∑k=1

k

(n

k

).

Page 87: OBM por assunto: Os primeiros passos olímpicos · tenha muito a acrescentar na sua vida pois o foco do treinamento é justamente para alcançar esse nível de habilidade. ... Esse

5.5. FUNÇÕES 87

Lembre que(nk

)= n

k

(n−1k−1

), logo

n∑k=1

k

(n

k

)=

n∑k=1

kn

k

(n− 1

k − 1

)

=

n∑k=1

n

(n− 1

k − 1

)

= n

n−1∑k=0

(n− 1

k

)= n · 2n−1

Exercícios

1. Calculen∑k=0

(−1)k(n

k

).

2. Calculen∑k=1

1

k

(n

k

).

3. Calcule 1× 2

(n

2

)+ 2× 3

(n

3

)+ · · ·+ (n− 1)× n

(n

n

).

Para mais exemplos e exercícios sobre somatórios veja o capítulo 5 do livroSolving Problems Throughout Problems de Loren C. Larson.

5.5 Funções

Em um certo ponto da vida de um matemático, o que se estuda é basicamentecomo o comportamento de uma estrutura in�uencia em outra, ou seja, funções.Agora você conhece a função do estudo de funções e aprenderá como funcionamas propriedades de funções em problemas de matemática!

5.5.1 Introdução

Uma função é um trio A,B, f , onde A e B são conjuntos e f : A→ B é uma leique relaciona cada elemento de A a um único elemento de B. Não entraremosem mais detalhes sobre o que é uma função.

A lei de uma função f : A→ B não precisa ser dada por uma única fómula,a única obrigação é aquela que mencionamos cada elemento a ∈ A deve serassociado a um único elemento f(a) ∈ B. Assim podemos de�nir, por exemploas funções f : R→ R por

f(x) =

x se x < −1,6 + 2x −1 ≤ x ≤ 1,x3 x > 1.

ou

f(x) =

{x se x ∈ Q,−x se x ∈ R\Q.

Page 88: OBM por assunto: Os primeiros passos olímpicos · tenha muito a acrescentar na sua vida pois o foco do treinamento é justamente para alcançar esse nível de habilidade. ... Esse

88 CAPÍTULO 5. ANÁLISE

ou

f(x) =

5 se x = −31π,

6 + 2x −2, 138 ≤ x ≤√

2,x3 se x é qualquer outro valor.

OBM

2003. Qual é o número de funções f : R→ R tal que ∀x ∈ R, f(x)(f(x)−x) = 0?

5.5.2 Substituição

A seguir daremos um exemplo de como resolver uma equação funcional simplesusando substituição. A técnica consiste do seguinte: fazer substituição das va-riáveis para obter restrições sobre a função e chegar a um conjunto de restriçõesque obrigam a função a ser uma função especí�ca, tendo sempre em mente quea lei de uma função não tem que ser sempre dada por uma lei bonitinha, ouseja, não é su�cienta achar que a solução é única, você tem que provar que elaé.

O problema a seguir foi retirado do banco de questões de treinamento paraa XV olimpíada rioplatense de matemática, feito pelo Grupo Teorema, de For-taleza.

Exemplo: Determine todas as funções f : R→ R que satisfazem as seguin-tes condições:

f(x) ≤ x e f(x+ y) ≤ f(x) + f(y),

para todos os reais x, y.

Solução:. Fazendo x = y = 0, obtemos f(0) ≤ 0 na primeira inequaçãoe f(0) ≤ f(0) + f(0) na segunda. Isto nos dá f(0) = 0. Agora, fazendoy = −x, �camos com 0 = f(0) ≤ f(x) + f(−x) ou ainda, f(x) ≥ −f(−x). Mas,observe que, trocando x por −x no primeira inequação do enunciado, temosf(−x) ≤ −x. Logo, f(x) ≥ −f(−x) ≥ x, de modo que, para todo x real,x ≤ f(x) ≤ x. Isto nos dá f(x) = x, para todo x ∈ R.

De�nição 6 Um função é dita injetiva se para todos x 6= y elementos dodomínio tivermos f(x) 6= f(y).

Exemplo: Encontre todas as funções f : R→ R tais que :

f(f(xy − x)) + f(x+ y) = y · f(x) + f(y).

Solução: Faremos alguma substituições em busca de encontrar restriçõesem f . Faça x = y = 0 na equação inicial para obter:

f(f(x · 0− x)) + f(x+ 0) = 0 · (f(x)) + f(0)

f(f(−x)) + f(x) = f(0). (∗)

Ou seja, qualquer função que obedece a equação do enunciado tem que obedecera equação acima para todo x ∈ R. Em particular segue que f(f(0)) = 0.

Page 89: OBM por assunto: Os primeiros passos olímpicos · tenha muito a acrescentar na sua vida pois o foco do treinamento é justamente para alcançar esse nível de habilidade. ... Esse

5.5. FUNÇÕES 89

Fazendo agora y = 1, x = 0 na equação inicial, obtemos:

f(f(0 · 1− 0)) + f(0 + 1) = 1 · (f(0)) + f(1)

f(f(0)) + f(1) = f(0) + f(1)

f(f(0)) = f(0)

0 = f(0). (∗∗)

Assim, usando (∗∗) e fazendo x = −z em (∗) temos

f(−z) = f(f(z)). (∗ ∗ ∗)

Faça y = 1 na equação do enuciado para obter:

f(f(x · 1− x)) + f(x+ 1) = 1 · (f(x)) + f(1)

f(f(0)) + f(x+ 1) = f(x) + f(1)

f(x+ 1) = f(x) + f(1).

Faça agora x = 1 na equação inicial obtemos:

f(f(1 · y − 1)) + f(1 + y) = y(f(1)) + f(y)

f(f(y − 1)) + f(y) + f(1) = y · f(1) + f(y)

f(f(y − 1)) + f(1) = y · f(1)

f(f(y − 1)) = (y − 1) · f(1).

Fazendo y = z + 1, e usando (∗ ∗ ∗) obtemos

−f(−z) = f(f(z)) = zf(1)

⇒ f(−z) = −zf(1).

Em particular f(f(1)) = f(1). Se f(1) = 0, então, fazendo z = −x temos

f(x) = xf(1) = x · 0 = 0, ∀x ∈ R.

Agora, suponha que, f(1) 6= 0. Se a, b ∈ R são tais que f(a) = f(b) = c,então

af(1) = f(f(a)) = f(c) = f(f(b)) = bf(1)⇒ a = b,

logo a função é injetiva. Daí se a = f(1) temos

f(a) = f(1)⇒ a = 1⇒ f(1) = 1,

e assim

f(x) = xf(1) = x.

Portanto a função as únicas funções que podem satisfazer a equação do enun-ciado são a função f(x) ≡ 0 e a função f(x) = x. Testando as duas na equaçãodo enunciado vemos que as duas são de fato solução.

Substituições usuaisVamos considerar que estamos olhando uma equação funcional na variável

x. É útil considerar as substituições de x pelos valores 0, 1,−x,±y, f(y), éútil também observar as particularidades do problema, substituir x por umaconstante conveniente relacionada ao problema especí�co.

No exemplo acima �zemos as substituições 0, 1,−z e a substituição y = z+1.

Page 90: OBM por assunto: Os primeiros passos olímpicos · tenha muito a acrescentar na sua vida pois o foco do treinamento é justamente para alcançar esse nível de habilidade. ... Esse

90 CAPÍTULO 5. ANÁLISE

Exercícios

1. Mostre que cada uma das funções a seguir é injetivaa) f : R −→ R uma função tal que f(x− f(y)) = 1− x− y para todos osreais x e y.b) f : N −→ N uma função tal que f(x+ f(y)) = f(x) + y para todos osinteiros positivos x e y.

2. (IMO 2002 adaptado) Encontre todas as funções f : R → R tais que∀x, y, u e v ∈ R temos:

(f(x) + f(y))(f(u) + f(v)) = f(xu− yv).

3. Encontre todas as funções f : R→ R tais que :

f(x2 − y2) = (x− y)(f(x) + f(y)).

Para aprender mais sobre equações funcionais, veja a Revista Eureka! nú-mero 37.

OBM

2000. Seja f uma função real que tem as seguintes propriedades:i) Para todos x, y reais, f(x+ y) = x+ f(y);ii)f(0) = 2.Quanto vale f(2000)?

2001. Seja f uma função de Z em Z de�nida como f(x) = x/10 se x édivisível por 10 e f(x) = x + 1 caso contrário. Se a0 = 2001 e an+1 = f(an),qual é o menor valor de n para o qual an = 1?

2002. Seja f uma função real de variável real que satisfaz a condição:

f(x) + f

(2002

x

)= 3x

para x > 0. Qual é o valor de f(2)?

2003. A função f é de�nida para todos os pares ordenados (x; y) de inteirospositivos e tem as seguintes propriedades:

f(x;x) = x, f(x; y) = f(y;x), (x+ y)f(x; y) = (2x+ y)f(x;x+ y).

Qual é o valor de f(21; 12)?

2004. A função real f , de�nida nos inteiros, satisfaz

f(n)− (n+ 1)f(2− n) = (n+ 3)2,

para todo n inteiro. Quanto vale f(0)?

2008. Considere a função f , de�nida no conjunto dos números reais e satis-fazendo f(x) = cx

2x+3 , para todo x 6= −3/2. Determine o número de tais funções

Page 91: OBM por assunto: Os primeiros passos olímpicos · tenha muito a acrescentar na sua vida pois o foco do treinamento é justamente para alcançar esse nível de habilidade. ... Esse

5.5. FUNÇÕES 91

f para as quais f(f(x)) = x, para todo x tal que f(f(x)) está bem de�nida.

2009. Seja f : R → R uma função tal que f(0) = 0, f(1) = 1, f(2) = 2 ef(x+ 12) = f(x+ 21) = f(x) para todo x ∈ R. Quanto vale f(2009)?

2012. Seja N = {0, 1, 2, · · · } e considere a função f : N→ N tal que f(0) = 1,f(1) = 2, f(2) = 0 e, para todo natural n ≥ 1, satisfaz as seguintes condições:

i) f(3n) = 3 · f(n) + 1;ii) f(3n+ 1) = 3 · f(n) + 2;iii) f(3n+ 2) = 3 · f(n);Quanto vale f(2012)?

5.5.3 Mais problemas sobre funções

Inversa

1999. Pedro saiu de casa e fez compras em quatro lojas, cada uma num bairrodiferente. Em cada uma gastou a metade do que possuía e a seguir, ainda pagouR$ 2, 00 de estacionamento. Se no �nal ainda tinha R$8, 00, que quantia tinhaPedro ao sair de casa?

Máximos e mínimos

1998. A função f associa a cada real x o menor elemento do conjunto{x+ 1, 15−x

2

}.

Qual é valor máximo de f(x)?

2015. Considere a função f , de�nida nos inteiros não negativos, tal quef(0) = 0 e f(n) = f

(b n10c

)− 10b n10c. Quantos algarismos tem o menor inteiro

positivo m tal que f(m) = 2015?

2016. Uma função f dos inteiros positivos nos inteiros positivos é tal que sem é múltiplo de n e m > n então f(m) > f(n). Qual é o menor valor possívelde f(2016)?

Iteração

1999. Para todo n natural de�nimos a função f por: f(n) = n2 se n é par,f(n) =

3n+ 1 se n é ímpar. Qual é número de soluções da equação f(f(f(n))) = 16.

2001. Seja f(x) = x2 − 3x + 4. Quantas soluções reais tem a equaçãof(f(f(...f(x)))) = 2 (onde f é aplicada 2001 vezes)?

2013. Seja S = {1, 2, 3, 4, 5, 6, 7, 8, 9}. Considere uma função f : S → Sde�nida pela tabela a seguir Qual é o menor valor inteiro positivo de n para oqual f(f(· · · (f(x))) = x para todo x ∈ S?

Page 92: OBM por assunto: Os primeiros passos olímpicos · tenha muito a acrescentar na sua vida pois o foco do treinamento é justamente para alcançar esse nível de habilidade. ... Esse

92 CAPÍTULO 5. ANÁLISE

Grá�co

2003. O grá�co de y = x2 − 5x+ 9 é rodado 180o em torno da origem. Qual éa equação da nova curva obtida?

2006. Altino está encostado num muro bem alto, durante a noite. A ruaonde Altino está é iluminada por uma lâmpada no topo de um poste de 4 metrosde altura, a 10 metros de distância do muro. Altino, um rapaz de 2 metros dealtura, anda em direção ao muro. Seja f(x) a altura, em metros, da sombra deAltino produzida pela lâmpada no muro quando Altino está a uma distância dex metros do muro. Qual alternativa representa melhor o grá�co de f(x)?

2010. Esmeralda ia desenhar o grá�co de y = 2x+ 6 mas trocou os eixos delugar. Como �ca o desenho dessa relação com os eixos trocados de lugar?

Page 93: OBM por assunto: Os primeiros passos olímpicos · tenha muito a acrescentar na sua vida pois o foco do treinamento é justamente para alcançar esse nível de habilidade. ... Esse

Capítulo 6

Teoria dos Números

Gosseiramente falando, teoria dos números (inteiros) é o estudo das propriedadesaritiméticas dos números inteiros. É talvez a primeira coisa que se começa aaprender em matemática, na vida, mas tem uma profundidade sem medida. Oque veremos aqui, é um tanto fora do cânone do ensino básico brasileiro. Teoriados números é um tópico de grandíssima beleza, tendo sido adjetivada por Gausscomo sendo a rainha da matemática.

"Ela é uma rainha matadora. Gelatina, pólvora, dinamite com raio laser.É garantia de explodir sua mente"

- Freddie Mercury.

6.1 Divisibilidade

Começaremos com um dos conceitos mais básicos e um tanto quanto conhe-cido, mas cuja profundidade das aplicações são surpreendents, o algoritmo deEuclides.

Teorema 13 (Euclides) Seja a um inteiro positivo. Dado qualquer inteiro bpodemos escrever de modo único b = aq + r com r ∈ {0, 1, ..., a− 1}.

Pela familiaridade que se tem com esse algoritmo, a prova será omitida, masa�rmamos com muita convicção, que ela não é difícil, segue por indução.

Como já se sabe, o r no algoritmo anterior é chamado de resto da divisão deb por a.

Vejamos alguns exemplos:Exemplo 1:

112 = 31× 3 + 19

251 = 31× 8 + 8

Qual é resto da divisão de 112 + 251 = 363 por 31?

93

Page 94: OBM por assunto: Os primeiros passos olímpicos · tenha muito a acrescentar na sua vida pois o foco do treinamento é justamente para alcançar esse nível de habilidade. ... Esse

94 CAPÍTULO 6. TEORIA DOS NÚMEROS

Exemplo 2:

85 = 13× 6 + 7

142 = 13× 10 + 12

Qual é o resto da divisão de 142− 85 = 57 por 13?

Exemplo 3:

205 = 29× 7 + 2

349 = 29× 12 + 1

Qual é o resto da divisão de 205× 349 = 71545 por 29?

Estas perguntas estão aí para nos dar uma noção intuitiva do que iremos verem nosso objeto �nal. Começaremos com a seguinte

De�nição 7 Dizemos que b é múltiplo de a, a é divisor de b, ou que a divideb, e escrevemos a|b quando existe um inteiro c tal que b = ac. Caso a não sejadivisor de b dizemos que a não divde b e denotamos por a 6 |b.

Em outros termos, dizer que a divide b signi�ca que ba é inteiro. Ou ainda,

a divide b quando o resto da divisão de b por a é zero.

Exemplos:

• −13|26 pois existe um inteiro c, a saber c = −2, tal que 26 = −13c;

• 1|n para qualquer inteiro n, pois n/1 = n e n que é um inteiro;

• 17|51 pois o resto da divisão de 51 por 17 é igual a 0;

• n|0, qualquer que seja n inteiro (diferente de 0), pois 0/n = 0 que é inteiro;

• 8 6 |4 pois não existe um inteiro c tal que 4 = 8c;

• 19 6 |189 pois 189/19 não é inteiro;

• 14 6 |37 pois o resto da divisão de 37 por 14 é diferente de 0.

De�nição 8 Dizemos que um número inteiro positivo p ≥ 2 é primo quando osseus únicos divisores (positivos) são 1 e o próprio p.

Exemplos: 2, 3, 5, 7, 11, 13, 17, 19, 23, 29, 31, 37, 41, 43, 47, 53 · · ·

Agora daremos algumas propriedades que facilitarão, no futuro, a resoluçãode problemas.

Lema 1 i) Se d|a e d|b, então d|ax± by para quaisquer x, y inteiros;ii) Se d|a, então a = 0 ou |a| ≥ |d|;iii) Se a|b e b|c então a|c.

Page 95: OBM por assunto: Os primeiros passos olímpicos · tenha muito a acrescentar na sua vida pois o foco do treinamento é justamente para alcançar esse nível de habilidade. ... Esse

6.1. DIVISIBILIDADE 95

Exemplo: Encontre todos os pares de inteiros (x, y) tais que 13x2 + 39y =2012.

Solução: Observe que se existisse solução inteira (x, y), teríamos que 13|13x2

pois 13x2

13 = x2 que é inteiro, do mesmo modo 13|39y, logo 13|13x2 +39y = 2012,mas 13 6 |2012 pois 2012

13 não é inteiro, absurdo! Logo não pode existir um parde inteiros (x, y) que seja solução da equação.

Exemplo: Mostre que 7 6 |7k − 1 para nenhum k.Solução: De fato, sabemos que 7|7k, se 7|7k − 1 teríamos que 7|7k − (7k −

1) = 1, mas 7 6 |1, logo 7 6 |7k − 1.

Exemplo: Encontre todos os inteiros positivos n tais que 7n+5913n+20 ∈ Z.

Solução: Como n é inteiro positivo 7n+ 59 6= 0, assim devemos ter obriga-toriamente |7n+59| ≥ |13n+20| ⇒ 7n+59 ≥ 13n+20⇒ 59−20 ≥ 13n−7n⇒39 ≥ 6n⇒ n ≤ 6, 5. Assim, temos seis possibilidades de inteiros positivos paratestar. Ao testarmos veri�camos que a única solução é n = 1.

Exercícios

1. Encontre todos os trios de inteiros (x, y, z) tais que 98xy+343zx = 70007.

2. Encontre o resto da divisão de 1 + 2 + 22 + ...+ 2100 por 4.

3. Mostre que (212 + 1)|(224 − 1).

4. Seja n um natural qualquer. Quais são os posíveis restos da divisão de n2

por 3, por 4, por 5 e por 8?

5. Encontre todos os trios de inteiros (x, y, z) tais que 4x+ y2 + z2 = 3507.

6. Mostre que o produto de dois números consecutivos é sempre par, e que oproduto de três consecutivos é sempre múltiplo de 6.

7. Prove ou dê um contra exemplo: O produto de n números consecutivos émúltiplo de n!.

8. Um inteiro a é dito livre de quadrados quando não existe nenhum in-teiro m 6= ±1 tal que m2|a. Qual é o maior número possível de inteirosconsecutivos livres de quadrados?

9. (IME) Mostre que (a− b)(a− c)(a− d)(b− c)(b− d)(c− d) é múltiplo de12, quaiquer que sejam a, b, c, d inteiros.

10. Se n é composto, então n|(n− 1)!.

11. Encotre todos os inteiros positivos n, tais que

a)(2n− 1)|(n3 + 1);

b)(2n3 + 5)|(n4 + n+ 1).

12. Mostre que

a) se m|(a+ b) e m|a então m|b;b) se m|(a− b) e m|a então m|b.

Page 96: OBM por assunto: Os primeiros passos olímpicos · tenha muito a acrescentar na sua vida pois o foco do treinamento é justamente para alcançar esse nível de habilidade. ... Esse

96 CAPÍTULO 6. TEORIA DOS NÚMEROS

13. Seja r o resto da divisão de a por b. Mostre que

d|a e d|b ⇐⇒ d|b e d|r.

14. (Deserto de Primos) Dado n ∈ N encontre uma sequência com n natu-rais consecutivos tais que nenhum deles é primo.

15. (IMO 1989) Prove que para todo positivo n existem n inteiros positivosconsecutivos nenhum dos quais é potência de primo.

16. (IMO 1984) Encontre dois inteiros positivos a, b tais que nenhum dosnúmeros a, b, a+ b é divisível por 7 e (a+ b)7 − a7 − b7 é divisível por 77.Dica: Veja os exercícios da seção de Binômio de Newton.

6.1.1 Máximo divisor comum

Exercício: Encontre o maior d que é ao mesmo tempo divisor de 10 e de 18.

Antes de continuar lendo, faça o exercício acima.

De�nição 9 O máximo divisor comum entre dois inteiros m,n é o maiorinteiro d tal que d|m e d|n. Denotamos por d = mdc(m,n).

Exemplo: Calcule mdc(8, 12).

Solução: Observe que o conjunto dos divisores de 8 é D(8) = {1, 2, 4, 8}enquanto que o conjunto dos divisores de 12 é D(12) = {1, 2, 3, 4, 6, 12}. Vejaque D(8) ∩D(12) = {1, 2, 4}, logo o maior divisor comum é 4.

Como 8 e 12 são números pequenos, �ca fácil olhar os conjuntos de divisores,mas e se fossem dois números grandes e com muitos divisores? A seguir veremosmétodos de calcular o mdc de números que são bem mais práticos.

De�nição 10 Dizemos quem,n são coprimos, ou primos entre si, semdc(m,n) =1.

Exemplos: Números consecutivos, são sempre coprimos; números ímpa-res consecutivos são sempre coprimos; e números pares consecutivos nunca sãocoprimos.

De�nição 11 Sejam m e n inteiros, a fração mn é dita irredutível quando m

e n são primos entre si.

Uma fração que não é irredutível é dita redutível. Assim, uma fração mn é

dita redutível quando mdc(m,n) ≥ 2.

Exemplos: As frações −94 ,

34 ,

1113 são irredutíveis, enquanto que 12

14 ,13013 ,

1751

são redutíveis.

Teorema 14 (Bachet-Bézout) Sejam m,n ∈ Z e d = mdc(m,n), então exis-tem x, y ∈ Z tais que

mx+ ny = d.

Page 97: OBM por assunto: Os primeiros passos olímpicos · tenha muito a acrescentar na sua vida pois o foco do treinamento é justamente para alcançar esse nível de habilidade. ... Esse

6.1. DIVISIBILIDADE 97

Prova: Será omitida.

Exemplo: (The William Lowell Putnam Competition) Sejam m e n inteiros

positivos, com m ≥ n. Prove que mdc(m,n)

m

(m

n

)é inteiro.

Solução: Sabemos que existem a, b ∈ Z tais que mdc(m,n) = am + bn.Assim

mdc(m,n)

m

(m

n

)=am+ bn

m

(m

n

)= a

(m

n

)+ b

n

m

(m

n

)= a

(m

n

)+ b

(m− 1

n− 1

)∈ Z.

Exercício: Mostre que para a, b, c inteiros, a equação

ax+ by = c

possui solução inteira em x e y se e somente se mdc(a, b)|c.

Proposição 4 Se mdc(a, b) = 1 e a|bc, então a|c.

Prova: Exercício: Use Bachet-Bézout e multiplique os dois lados da equa-ção por c.

Exercícios

1. a)Mostre que se 2|n e 3|n então 6|n.b) Generalize para p, q primos quaisquer.

c) Generalize para a, b com mdc(a, b) = 1.

d) Generalize para uma quantidade maior de números.

Proposição 5 Se a, b, c ∈ Z, temos

1. Se k ∈ Z então mdc(a, b) = mdc(a− kb, b).

2. Se a|c então mdc(a, b)|mdc(a, c).

3. Se mdc(a, b) = 1, então mdc(ac, b) = mdc(c, b).

Prova: Exercício. Use os últimos exercícios da seção 1.

A seguir veremos como usar a propriedade 1 da proposição acima.

Exemplo: Calcule mdc(1055, 107).

Page 98: OBM por assunto: Os primeiros passos olímpicos · tenha muito a acrescentar na sua vida pois o foco do treinamento é justamente para alcançar esse nível de habilidade. ... Esse

98 CAPÍTULO 6. TEORIA DOS NÚMEROS

Solução:

1055 = 9× 107 + 92

107 = 1× 92 + 15

92 = 6× 15 + 2

15 = 7× 2 + 1

2 = 2× 1

Usando a propriedade 1 da proposição acima, com k sendo o número sublinhadoem cada linha das equações acima temos

mdc(1055, 107) = mdc(1055− 9× 107, 107)

= mdc(92, 107)

= mdc(92, 15)

= mdc(2, 15)

= mdc(2, 1)

= 1.

Exercícios

1. Calcule os mdc's abaixo

a) mdc(1001, 109);

b) mdc(2025, 125);

c) mdc(1023, 64);

d) mdc(1023, 127).

2. Sendo n ∈ N, mostre que as frações abaixo são irredutíveis

a)21n+ 4

14n+ 3(Primeiro problema da primeira IMO);

b)n3 + 2n

n4 + 3n2 + 1.

OBM

2007. Qual é o máximo divisor comum entre os números

1221, 2332, 3443, 4554, · · · 8998?

2010. Qual é o máximo divisor comum de todos os números que são o pro-duto de cinco ímpares positivos consecutivos?

2010. Quatro números inteiros positivos a < b < c < d são tais que o mdcentre quaisquer dois deles é maior do que 1, mas mdc(a, b, c, d) = 1. Qual é omenor valor possível para d?

Page 99: OBM por assunto: Os primeiros passos olímpicos · tenha muito a acrescentar na sua vida pois o foco do treinamento é justamente para alcançar esse nível de habilidade. ... Esse

6.1. DIVISIBILIDADE 99

2012. Em 2012 estamos realizando a edição 34 da OBM, emdc(2012, 34) = 2.Supondo que a OBM sempre será realizada todo ano, qual é o maior valor pos-sível para o mdc do ano e da edição da OBM realizada no ano?

2012. Esmeralda desenhou uma tabela com 100 linhas e 100 colunas e escre-veu, na linha i e coluna j da tabela, mdc(i, j) se i < j e mmc(i, j) se i ≥ j. Porexemplo, na linha 4, coluna 6 ela escreveu mdc(4, 6) = 2 e na linha 15, coluna10 ela escreveu mmc(15, 10) = 30. Qual é o produto de todos os 1002 númerosda tabela?

6.1.2 Critérios de divisibilidade

Iremos representar um número N ∈ N como anan−1 · · · a1a0, onde para cada0 ≤ i ≤ n os números ai ∈ {0, · · · , 9} são os seus dígitos. A partir dessarepresentação por dígitos, iremos fazer a análise dos divisores de N . Antes demais nada observe que

anan−1 · · · a1a0 = 10nan + 10n−1an−1 + · · ·+ 10a1 + a0.

Abaixo iremos sempre supor que N = anan−1 · · · a1a0.

Divisibilidade por 2

Observe que anan−1 · · · a1a0 = 10×anan−1 · · · a1 +a0, como 2|10×anan−1 · · · a1

segue que 2|N ⇐⇒ 2|a0.

Divisibilidade por 3

Primeiro veja que 10k − 1 = 99 · · · 9 = 3 × 33 · · · 3, logo 3|(10k − 1) para todok ∈ N

anan−1 · · · a1a0 = 10nan + 10n−1an−1 + · · ·+ 10a1 + a0

= [(10n − 1)an + (10n−1 − 1)an−1 + · · ·+ (10− 1)a1] + an + an−1 + · · ·+ a1 + a0

logo, como 3|[(10n − 1)an + (10n−1 − 1)an−1 + · · · + (10 − 1)a1], 3|N ⇐⇒3|(an + an−1 + · · ·+ a1 + a0).

Divisibilidade por 4

4|N ⇐⇒ 4|a1a0.Exercício: Escreva anan−1 · · · a1 = 100 × anan−1 · · · a2 + a1a0 e faça de

modo análogo ao que �zemos com 2.

Divisibilidade por 5

5|N ⇐⇒ 5|a0.Exercício: Análogo ao 2.

Page 100: OBM por assunto: Os primeiros passos olímpicos · tenha muito a acrescentar na sua vida pois o foco do treinamento é justamente para alcançar esse nível de habilidade. ... Esse

100 CAPÍTULO 6. TEORIA DOS NÚMEROS

Divisibilidade por 6

6|N ⇐⇒ N = 6k ⇐⇒ 2|N e 3|N .

Divisibilidade por 8

8|N ⇐⇒ 8|a2a1a0.Exercício: Use uma ideia parecida com a do 4.

Divisibilidade por 9

9|N ⇐⇒ 9|(an + an−1 + · · ·+ a1 + a0).Exercício: Análogo ao 3.

Divisibilidade por 11

11|N ⇐⇒ 11|(−1)nan + (−1)n−1an−1 + · · · − a1 + a0. Em palavras: se asoma dos dígitos das casas pares menos a soma dos dígitos nas casas ímparesfor múltiplo de 11.

Exercício: Análogo ao 3.

Divisibilidade por 10m

10m|N ⇐⇒ amam−1 · · · a1a0 = 0 · · · 00.

Exercícios

1. Usando os dígitos 1, 2, 3, 4, 5, 6, 7, construímos vários números de setedígitos distintos. Existem dois deles, distintos, tais que um divide o outro?

2. Mostre que 7|an · · · a1a0 ⇐⇒ 7|(an · · · a1 − 2a0).

3. Existe uma potência de 2 cujos dígitos podem ser rearranjados de modoa se obter outra potência de 2 (distinta)?

4. Determine os dígitos a, b na equação abaixo

23! = 2585201ab38884976640000.

OBM

1998. O número 1234a6 é divisível por 7. Quanto vale o o algarismo a?

1999. Quantos são os possíveis valores inteiros de x para quex+ 99

x+ 19seja

um número inteiro?

2001. Qual é o número de soluções inteiras distintas da equação

(−6x2 + 12x− 2)x2−2x+2 = 4?

2002. N = λ539984λ é um número inteiro positivo com oito algarismos,sendo o primeiro e o último desconhecidos. Sabendo que N é um múltiplo de

Page 101: OBM por assunto: Os primeiros passos olímpicos · tenha muito a acrescentar na sua vida pois o foco do treinamento é justamente para alcançar esse nível de habilidade. ... Esse

6.1. DIVISIBILIDADE 101

198, encontre o algarismo das unidades de N/198.

2002. Sejam x, y, z números inteiros tais que x+y+z = 0. Sobre x3 +y3 +z3

quais a�rmativas seguintes são verdadeiras?i) É necessariamente múltiplo de 2.ii) É necessariamente múltiplo de 3.iii) É necessariamente múltiplo de 5.

2004. Para quantos inteiros positivos m o número

2004

m− 2

é um inteiro positivo?

2005. Esmeralda digitou corretamente um múltiplo de 7 muito grande, com4010 algarismos. Da esquerda para a direita, os seus algarismos são 2004 alga-rismos 1, um algarismo n e 2005 algarismos 2. Qual é o valor de n?

2006. Considere os 2161 produtos 0 · 2160, 1 · 2159, 2 · 2158, · · · , 2160 · 0.Quantos deles são múltiplos de 2160?

2006. Qual dos valores abaixo de x é tal que 2x2 + 2x+ 19 não é um númeroprimo?

A) 50 B) 37 C) 9 D) 5 E) 1

2006. O professor Piraldo aplicou uma prova para seus cinco alunos e, apóscorrigi-las, digitou as notas em uma planilha eletrônica que calcula automati-camente a média das notas à medida que elas são digitadas. Piraldo notou queapós digitar cada nota a média calculada pela planilha era um número inteiro.Se as notas dos cinco estudantes são, em ordem crescente, 71, 76, 80, 82 e 91,qual foi a última nota que Piraldo digitou?

2007. Considere a seguinte seqüência:

27 = 3×3×3, 207 = 3×3×23, 2007 = 3×3×223, 20007 = 3×3×2223, · · ·

Qual dos seguintes inteiros é um múltiplo de 81?

A) 200.007.B) 20.000.007.C) 2.000.000.007.D) 200.000.000.007.E) 20.000.000.000.007.

2007. Considere todos os números abc de três algarismos onde b = a2 + c2

e a 6= 0 . A diferença entre o maior e o menor destes números é um número:A) Múltiplo de 3.B) Primo.C) Com último algarismo igual a 7.

Page 102: OBM por assunto: Os primeiros passos olímpicos · tenha muito a acrescentar na sua vida pois o foco do treinamento é justamente para alcançar esse nível de habilidade. ... Esse

102 CAPÍTULO 6. TEORIA DOS NÚMEROS

D) Cuja soma dos algarismos é 10.E) Múltiplo de 7.

2008. Quantos dos números 2, 3, 5, 7, 11 são divisores de 3714 − 414?

2009. Os inteiros positivos m e n satisfazem 15m = 20n. Então é possívela�rmar, com certeza, que mn é múltiplo de:

A) 5. B) 10. C) 12. D) 15. E) 20.

2009. Um dos cinco números a seguir é divisor da soma dos outros quatro.Qual é esse número?

A) 20. B) 24. C) 28. D) 38. E) 42.

2010. Quantos são os pares (x, y) de inteiros positivos tais que x2 − y2 =22010?

2011. Os inteiros positivos 30, 72 e N possuem a propriedade de que o pro-duto de quaisquer dois é divisível pelo terceiro. Qual o menor valor possível deN?

2012. Qual é a maior potência de 2 que divide 20112012 − 1?

2014. O número de cinco dígitos xy26z em que cada uma das letras repre-senta um dígito, é divisível por 8, 9 e 11. Qual o valor de x?

2014. Quantos pares ordenados (a, b) de inteiros positivos existem tais que2014a2+b2 é inteiro?

6.1.3 Teorema Fundamental da Aritmética

Teorema 15 (Teorema Fundamental da Aritmética) Seja n ≥ 2 um nú-mero natural. Podemos escrever n de uma única forma como o produto n =p1p2 · · · pm onde m ≥ 1 é um natural e p1 ≤ p2 ≤ · · · ≤ pm são primos.

Exemplos:360 = 2.2.2.3.3.514 = 2.798 = 2.7.7

Exercício: Mostre que todo n composto possui um fator primo p ≤ b√nc.

Teorema 16 (Euclides) Existem in�nitos primos.

Prova: Suponha que existam �nitos, digamos n, e sejam eles todos p1, p2, · · · , pn.Veja que o número N = p1p2 · · · pn + 1 não é divisível por nenhum dos primosque existem, mas pelo Teorema Fundamentel da Aritmética deve possuir algumfator primo. Absurdo! �

Exercícios

Page 103: OBM por assunto: Os primeiros passos olímpicos · tenha muito a acrescentar na sua vida pois o foco do treinamento é justamente para alcançar esse nível de habilidade. ... Esse

6.1. DIVISIBILIDADE 103

1. Mostre que existem in�nitos primos das formas 4k + 3 e 6k + 5.

OBM

2004. Se n! = 215 · 36 · 53 · 72 · 11 · 13, qual é o valor de n?

2006. Que expressão não pode representar o número 24 para valores inteirospositivos convenientes de a, b e c?

A) ab3. B) a2b3. C) acbc. D) ab2c3. E) abbcca.

2008. O inteiro n é tal que n · 2n possui 2008 divisores a mais que n. En-contre n.

2009. Considere o número inteiro positivo n tal que o número de divisorespositivos do dobro de n é igual ao dobro do número de divisores positivos de n.Podemos concluir que n é

A) um número primo.B) um número par.C) um número ímpar.D) um quadrado perfeito.E) potência inteira de 2.

2010. Qual dos seguintes números é um divisor de 35 · 44 · 53 ?A) 42. B) 45. C) 52. D) 85. E) 105.

2011. Quantos números inteiros positivos menores que 30 têm exatamentequatro divisores positivos?

2011. Qual é o maior inteiro positivo n tal que (2011!)! é divisível por ((n!)!)!?

2014. Qual é o menor inteiro positivo com 2014 divisores positivos?

Exercícios

1. Determine todos os inteiros positivos n tais que 2n−1 é divisível por 7.

2. Encontre todos os pares de inteiros positivos (x, y) tais que x2 − y2 = 71.

3. Encontre todos os pares de inteiros positivos (x, y) tais que x2 − y2 = 55.

4. Encontre todos os pares de inteiros positivos (x, y) tais que x2 − y2 = 96.

5. Mostre que√

2 não é racional, isto é, não existem p, q ∈ Z tais que√

2 = pq .

6. (Rioplatense) Prove que a equação q3 = 2n + 1 não possui solução n e 1nos inteiros positivos.

7. (OBM-2001)Mostre que não existem dois números inteiros a e b tais que(a+ b)(a2 + b2) = 2001.

Page 104: OBM por assunto: Os primeiros passos olímpicos · tenha muito a acrescentar na sua vida pois o foco do treinamento é justamente para alcançar esse nível de habilidade. ... Esse

104 CAPÍTULO 6. TEORIA DOS NÚMEROS

6.2 Congruências

O uso de congruências é bastante comum na resolução de problemas de arit-mética e teoria dos números que surgem na jornada de um olímpico. Com estatécnica muitos problemas se tornam muito mais simples do que parecem. A con-gruêcia em geral, é apenas uma maneira simpli�cada de avaliar divisibilidade.

6.2.1 De�nição e exemplos

De�nição 12 Sejam a, b e n inteiros, dizemos que a é congruente a b módulon, e escrevemos a ≡ b(mod.n), quando n|(b− a).

Exemplos:7 ≡ 3(mod.4) pois 4|(3− 7);−1 ≡ 1(mod.2) pois 2|(1− (−1));0 ≡ 65(mod.− 13) pois −13|(65− 0).

Observações:1) Se a = nq+ r é a decomposição obtida pelo algoritmo de Euclides, temos

que r ≡ a(mod n), ou seja, o dividendo é congruente ao resto módulo o divisor,pois n|nq e a− r = nq.

2)Temos que a|b ⇐⇒ 0 ≡ b(mod.a)

Agora pare! Volte ao início! Pense um pouco nas quatro perguntas que�zemos no início deste texto! Você consegue ver algo com outros olhos? Se vocêpercebeu algo interessante você poderá �car feliz com a veracidade dos fatosque serão observados nas propriedades a seguir, fatos que tornarão o seu mundobem mais simples!

Exercícios

1. Dê os critérios de divisibilidade em termos de congruências.

2. Calcule x2(mod.11) para x = 2, · · · , 9.

3. a) Mostre que se a ≡ 5 (mod.13) e a ≡ 5 (mod.17), então a ≡ 5 (mod.221)

b) O que acontece se trocarmos 5 por 7? Generalize.

c) O que acontece se trocarmos 13 por 17 por um par de primos p, q?Generalize.

d) Generalize para uma quantidade qualquer de primos (digamos n pri-mos).

4. Encontre todos os inteiros x com 0 ≤ x < 147 tais que x2 ≡ 1(mod.147).Generalize para p qualquer primo.

5. Calcule(

7k

)/7, para k = 1, · · · , 6. O que você pode concluir em termos de

congruências?

Page 105: OBM por assunto: Os primeiros passos olímpicos · tenha muito a acrescentar na sua vida pois o foco do treinamento é justamente para alcançar esse nível de habilidade. ... Esse

6.2. CONGRUÊNCIAS 105

6.2.2 Propriedades

Proposição 6 (Relação de equivalência) Congruência é uma relação de equi-valência, ou seja, se a, b, c, n ∈ Z valem

1. a ≡ a (mod.n)

2. a ≡ b (mod.n) ⇒ b ≡ a (mod.n)

3. a ≡ b (mod.n) e b ≡ c (mod.n) ⇒ a ≡ c (mod.n)

Prova:

1. n|0 = a− a⇒ a ≡ a (mod.n).

2. a ≡ b (mod.n)⇒ n|b− a⇒ n|a− b⇒ b ≡ a (mod.n).

3.

a ≡ b (mod.n) e b ≡ c (mod.n)⇒ n|b− a e n|c− b⇒ n|(b− a) + (c− b) = c− a⇒ a ≡ c (mod.n).

Proposição 7 (Propriedades) Se valem a ≡ b (mod.n) e c ≡ d (mod.n)então

1. a+ c ≡ b+ d (mod.n).

2. a− c ≡ b− d (mod.n).

3. a · c ≡ b · d (mod.n).

4. am ≡ bm (mod.n) para todo m ∈ N.

5. Se c|a, c|b e mdc(c, n) = 1, entãoa

c≡ b

c(mod.n)

Prova:

1.

a ≡ b (mod.n) e c ≡ d (mod.n)⇒ (a− b) = kn, (c− d) = ln

⇒ (a+ c)− (b+ d) = (k + l)n

⇒ a+ c ≡ b+ d (mod.n).

2. Exercício.

3. Exercício: ac− bd = ac− bc+ bc− bd.

4. Exercício: Use o item acima e indução.

5. Exercício.

Page 106: OBM por assunto: Os primeiros passos olímpicos · tenha muito a acrescentar na sua vida pois o foco do treinamento é justamente para alcançar esse nível de habilidade. ... Esse

106 CAPÍTULO 6. TEORIA DOS NÚMEROS

Exercícios

1. Sejam p primo e 1 ≤ k ≤ p− 1

a) Calcule(p

k

)(mod.p);

b) Mostre que (a+ b)p ≡ ap + bp (mod.p).[Sonho de todo estudante]

2. Sejam a, b ∈ Z e p(x) = cnxn+· · ·+c1x+c0 um polinômio com coe�cientes

inteiros. Mostre que (b− a)|(p(b)− p(a)).

3. Encontre todos os polinômios com coe�cientes inteiros tais que f(7) = 11e f(11) = 13.

4. Seja p(x) um polinômio não constante com coe�cientes inteiros. Mostreque para todo i inteiro,

a) p(i)|p(2017 · p(i) + i);

b) p(2017)|p(i · p(2017) + 2017).

5. Mostre que

a) Se y é ímpar então 2ky ≡ 2k (mod.2k+1).

b) Se y = 8q + 1 com q ∈ N, e k ∈ N, então existe x tal que

x2 ≡ y (mod.2k).

Dica: Fixe y e faça indução em k.

OBM

1999. O número N = 11111 · · · 11 possui 1999 dígitos, todos iguais a 1. Qual éresto da divisão de N por 7?

2001. Quantos dígitos tem o menor quadrado perfeito cujos quatro últimosdígitos são 2001?

2002. Qual é o resto da divisão por 9 de√

1111111111− 22222?

2001. O número N de três algarismos multiplicado por 7 deu como resultadoum número que termina em 171. Determine N .

6.3 Pequeno Fermat e Grande Euler

O Pequeno Teorema de Fermat é um teorema recorrente em olimpíadas de ma-temática, é um dos teoremas que mais chamam a atenção dos novos olímpicos.Esse teorema foi demonstrado pelo entusiasta da matemática Pierre de Fermat,que entre seus legados está um problema conhecido como "O último teoremade Fermat" (sugiro fortemente que o leitor pesquise sobre esse teorema). Já oteorema de Euler pode ser visto como uma generalização do pequeno teoremade Fermat, o matemático que o provou é visto como um dos matemáticos maisprodutivos de todos os tempos

Page 107: OBM por assunto: Os primeiros passos olímpicos · tenha muito a acrescentar na sua vida pois o foco do treinamento é justamente para alcançar esse nível de habilidade. ... Esse

6.3. PEQUENO FERMAT E GRANDE EULER 107

6.3.1 Pequeno Teorema de Fermat

Teorema 17 (Pequeno Teorema de Fermat) Mostre que se p é primo, en-tão ∀a ∈ N,

ap ≡ a (mod.p).

Prova: Use o indução em a e o "sonho de todo estudante".

Exemplo: (OBM 2008/N2) Mostre que existem in�nitos inteiros positi-vos n tais que n|(5n−2 − 1).

Solução: O pequeno teorema de Fermat resolveria o exercício de imedi-ato se em vez de n − 2 fosse n − 1, bastaria pegar n primo. Uma coisa quepodemos fazer então é fatorar o 2 supondo que n = 2k, ou seja procurar2k|(52k−2 − 1) = (5k−1 − 1)(5k−1 + 1). Agora podemos usar o Pequeno Te-orema de Fermat e usar que 2|(5k−1 + 1). Assim, o resultado é verdade paratodo n = 2p onde p > 2 é um primo.

Exercícios

1. (Brilliant.org) Para quantos valores de p, 41p + 1 é múltiplo de p?

2. Mostre que se p > 17 é primo, então 16320|(p32 − 1).

3. (Hong Kong TST 2017) Encontre in�nitos inteiros positivos m para osquais o número 2m−1−1

8191m é inteiro.

6.3.2 Função φ de Euler

De�nição 13 Dado um número natural n de�nimos φ(n) como sendo a quan-tidade de números k ≤ n que são primos com n, ou seja, tais que mdc(n, k) = 1.

Obviamente φ é uma função de φ : N → N. Esse é o primeiro exemplo defunção artitmética importante.

Exercícios

a) Calcule φ(p) para p número primo.b) Calcule φ(p2) para p número primo.c) Calcule φ(pn) para p número primo e n ∈ N.d) Calcule φ(n) para n ∈ {1, 2, 3, · · · , 15}.

Lema 2 A função φ é aritmética multiplicativa, isto é, se mdc(a, b) = 1, entãoφ(ab) = φ(a)φ(b).

Exercício: Calcule φ de 100, 2000, 30000.

Teorema 18 (Euler) Seja n ∈ N, então ∀a ∈ N com mdc(a, n) = 1, temos

aφ(n) ≡ 1 (mod.n).

Page 108: OBM por assunto: Os primeiros passos olímpicos · tenha muito a acrescentar na sua vida pois o foco do treinamento é justamente para alcançar esse nível de habilidade. ... Esse

108 CAPÍTULO 6. TEORIA DOS NÚMEROS

Observe que o teorema de Euler é uma generalização do Pequeno Teoremade Fermat. Mais precisamente, o teorema de Fermat segue do teorema de Eulerno caso em que n é um número primo.

Exemplo: Encontre o dígito das unidade de 777

.Solução: Encontrar o último dígito de um número n, consiste em encontrar

um número m, com 0 ≤ m ≤ 9 tal que n ≡ m (mod.10). Vamos usar aqui oteorema de Euler. Veja que φ(10) = 4 e que 77, o expoente de 7 no exercício, éigual a 7 · (72)3 ≡ 3 (mod.4), ou seja, 77 = 4k + 3 para algum k ∈ N, logo

777

≡ 74k+3 ≡ (74)k · 73 ≡ 1 · 343 ≡ 343 ≡ 3 (mod.10).

Logo o dígito é 3.

Exercícios

1. Seja n um número par. Mostre que (n2 − 1)|(2n! − 1).

2. Calcule

a) φ(21);

b) o resto da divisão de 417 − 45 por 21.

3. Mostre que se a ≥ 3, então φ(a) é par.

4. Se mdc(a, n) = 1 e x ≡ 3(mod.φ(n)) então ax ≡ a3(mod.n). Generalize:troque 3 por um y ∈ N qualquer.

5. Sejam a tal que mdc(a, 91) = 1 e t ∈ {1, · · · , φ(91)} o menor valor tal queat ≡ 1(mod.91). Mostre que t|φ(91). Generalize.

6. Mostre que existem in�nitos múltiplos de 2009 que são da forma 2000 · · · 009.

7. (Brilliant.org) Para quantos valores inteiros de 1 ≤ i ≤ 1000, existe uminteiro j, 1 ≤ j ≤ 1000 tal que i é divisor de 2j − 1?

8. Mostre que se mdc(a, c) = 1 então existe x ∈ N tal que

ax + x ≡ 1(mod.c).

OBM

1998. Qual é o dígito das unidades do número 31998?

2002. Qual é o dígito das unidades de 777···7

, onde aparecem 2002 setes?

2009. Seja N = 88···8

, em que aparecem 2009 números 8. Agilulfo �cou decastigo: ele deve escrever a soma dos dígitos de N , obtendo um número M ; emseguida, deve calcular a soma dos dígitos de M ; e deve repetir o procedimentoaté obter um número de um único dígito. Determine o dígito.

Page 109: OBM por assunto: Os primeiros passos olímpicos · tenha muito a acrescentar na sua vida pois o foco do treinamento é justamente para alcançar esse nível de habilidade. ... Esse

6.4. TEOREMA CHINÊS DOS RESTOS 109

6.4 Teorema Chinês dos Restos

Quando aprendemos a resolver equações é natural o surgimento dos sistemas.O mesmo ocorre quando se estuda congruências. Aprendemos a resolver ascongruências e aí surgem os sitemas de congruências. O método para resolvertais sistemas é o tópico dessa seção.

Teorema 19 Se mdc(a, n) = 1, então existe x tal que a · x ≡ 1(mod.n).

Prova: Exercício. Basta usar o teorema de Bachet-Bezout.

Exercícios

1. Sejam a, b, c, d ∈ N dois a dois primos entre si

a) Encontre x tal que x ≡ 0(mod.a), x ≡ 0(mod.b) e x ≡ 1(mod.c)

b) Encontre x tal que x ≡ 0(mod.a), x ≡ 0(mod.b) e x ≡ d(mod.c).

2. Vamos encontrar uma solução para o sistema de congruências:

k ≡ 8(mod.17)

k ≡ 3(mod.13)

k ≡ 9(mod.12)

a) Encontre um número x tal que o resto da divisão de x por 12 é 0, por13 é 0 e por 17 é 8.

b) Encontre um número y tal que o resto da divisão de y por 12 é 0, por13 é 3 e por 17 é 0.

c) Encontre um número z tal que o resto da divisão de z por 12 é 9, por13 é 0 e por 17 é 0.

d) Qual é o resto da divisão de x+y+z por cada um dos valores 12, 13, 17?

3. Encontre um k ∈ N tal que

k ≡ −5(mod.17)

k ≡ 1(mod.12)

k ≡ 0(mod.7).

4. Encontre um k ∈ N tal que

k ≡ 11(mod.23)

k ≡ −2(mod.17)

k ≡ 4(mod.13)

k ≡ 6(mod.8).

Page 110: OBM por assunto: Os primeiros passos olímpicos · tenha muito a acrescentar na sua vida pois o foco do treinamento é justamente para alcançar esse nível de habilidade. ... Esse

110 CAPÍTULO 6. TEORIA DOS NÚMEROS

Teorema 20 Sejam m1,m2, ...,mr, inteiros positivos primos entre si, dois adois, e sejam a1, a2, ..., ar; r inteiros quaisquer. Então, o sistema de congruên-cias:

x ≡ a1(mod.m1)

x ≡ a2(mod.m2)

· · ·x ≡ ar(mod.mr)

admite uma solução x. Além disso, as soluções são únicas módulom = m1m2...mr.

Prova: Análoga ao que foi feito acima, apenas com mais variáveis. A uni-cidade da solução será omitida.

Exemplo: Mostre que existe uma sequência de 2017 inteiros positivos con-secutivos tais que todos são múltiplos de algum quadrado perfeito maior que 1.

Solução: Escolha seus 2017 primos favoritos, e considere o conjunto formadopor eles P = {p1, · · · , p2017}. Claramente mdc(p2, q2) = 1 para qualquer par deprimos p, q ∈ P . Pelo Teorema Chinês dos Restos, temos que o sistema

x ≡ −1 (mod.p21)

x ≡ −2 (mod.p22)

· · ·x ≡ −2017 (mod.p2

2017)

possui uma solução x, isto é, existe x tal que cada um dos termos x + 1, x +2, · · · , x+ 2017 é multiplo de algum dos p2 com p ∈ P .

Exercícios

1. (Estônia 2000) Determine todos os restos possíveis da divisão do qua-drado de um número primo com 120 por 120.

2. Para um inteiro positivo d, dizemos que um inteiro positivo n está d-segurose |n−m| > 2, para qualquer múltiplo m de d. Por exemplo, o conjuntodos números 10-seguros é 3, 4, 5, 6, 7, 13, 14, 15, 16, 17, 23, ...

Encontre o número de inteiros positivos menores ou iguais a 10.000 quesão simultaneamente 7-seguro, 11-seguro, and 13-seguro.

3. (AoPS). Mostre que para c ∈ Z e p primo a congruência

xx ≡ c (mod.p)

tem solução.

4. (IMO 1989) Prove que para todo positivo n existem n inteiros positivosconsecutivos nenhum dos quais é potência de primo. [Sim, você já viu esseexercício. Agora faça usando o teorema chinês].

Page 111: OBM por assunto: Os primeiros passos olímpicos · tenha muito a acrescentar na sua vida pois o foco do treinamento é justamente para alcançar esse nível de habilidade. ... Esse

6.5. PROBLEMAS VARIADOS 111

5. (Eureka 25) Sejam p um primo, a, b inteiros positivos. Mostre que existex ∈ N tal que

a) {x ≡ r (mod.p− 1)ax + x ≡ b (mod.p).

b) {x ≡ r (mod.p− 1)ax + x ≡ b (mod.p2).

Observação: Esse exercício foi extraído da solução de Gabriel Bujokas doproblema 6 da OBM de 2005.

6.5 Problemas variados

Agora que você já manja dos paranauê você pode resolver uns exercícios umpouco mais conectados com outras ideias.

6.5.1 Contagem com TN

1998. Um número inteiro n é bom quando 4n+ 1 é um múltiplo de 5. Quantosnúmeros bons há entre 500 e 1000?

2004. Com três algarismos distintos a, b e c é possível formar 6 números dedois algarismos distintos. Quantos conjuntos {a, b, c} são tais que a soma dos 6números formados é 484?

2006. Arnaldo tem vários quadrados azuis 1× 1 , vários quadrados amarelos2 × 2 e vários quadrados verdes 3 × 3 e quer montar um quadrado maior noqual apareçam as três cores. Qual é a menor quantidade de quadrados que elepoderá utilizar ao todo?

2006. Num tabuleiro retangular de 13 linhas e 17 colunas colocamos núme-ros em cada casinha da seguinte maneira: primeiro, numeramos as casinhas daprimeira linha, da esquerda para a direita, com os números 1, 2, 3, · · · , 17 nessaordem; depois numeramos a segunda linha, também da esquerda para a direita,com os números de 18 a 34, e assim por diante. Após preenchermos todo otabuleiro, colocamos em cada casinha um segundo número, numerando as ca-sinhas da primeira coluna, de cima para baixo, com os números 1, 2, 3, · · · , 13,nessa ordem, depois numeramos a segunda coluna, também de cima da baixo,com os números de 14 a 26, e assim por diante. Deste modo, cada casinha temdois números. Quantas casinhas têm dois números iguais?

2008. De quantas maneiras podemos dividir R$10, 00 em moedas de 10 cen-tavos e de 25 centavos, se pelo menos uma moeda de cada valor tem que serusada?

2010. Considere todos os números de três algarismos distintos, cada umigual a 0, 1, 2, 3 ou 5. Quantos desses números são múltiplos de 6?

Page 112: OBM por assunto: Os primeiros passos olímpicos · tenha muito a acrescentar na sua vida pois o foco do treinamento é justamente para alcançar esse nível de habilidade. ... Esse

112 CAPÍTULO 6. TEORIA DOS NÚMEROS

2011. Luca comprou uma revista por R$9, 63 e deu uma nota de R$10, 00para pagar. De quantas maneiras ele pode receber o troco de 37 centavos emmoedas, se as moedas disponíveis no caixa são as de 1, 5, 10 e 25 centavos? Su-ponha que há muitas moedas de cada tipo.

2011. Existem 3 valores inteiros positivos de n > 1 tais que 10 pode serescrito como soma de n inteiros positivos e distintos:

n = 2 : 10 = 3 + 7

n = 3 : 10 = 2 + 3 + 5

n = 4 : 10 = 1 + 2 + 3 + 4

Quantos valores inteiros e positivos de n > 1 existem para os quais é possívelexpressar 2011 como soma de n inteiros positivos e distintos?

2013. Entre os números naturais de 1 até n, pelo menos 11 são divisíveis por5 e no máximo 9 são divisíveis por 6. No máximo, quantos desses números sãodivisíveis por 7?

2013. O Aluno D (usaremos este codinome para proteger a identidade doaluno) não prestou atenção na aula e não aprendeu como veri�car, sem realizara divisão, se um número é múltiplo de 7 ou não. Por isso, D decidiu usar a regrado 3, ou seja, ele vai somar os dígitos e veri�car se o resultado é um múltiplode 7. Para quantos números inteiros positivos menores que 100 esse métodoincorreto indicará que um número é múltiplo de 7, sendo o número realmentemúltiplo de 7?

2013. Num circo, a atração principal é a Corrida de Pulgas. Duas pulgas, P1

e P2, perfeitamente treinadas, saltam ao longo de uma linha reta, com velocida-des constantes, partindo de um mesmo ponto e no mesmo instante. Cada saltoda pulga P1 tem alcance m centímetros e cada salto da pulga P2 tem alcance ncentímetros, com m < n, ambos inteiros. Porém a pulga P1 é mais rápida quea pulga P2, de modo que, independente da velocidade de P2, P1 sempre podealcançá-la após alguns saltos. Supondo que, após a largada, as pulgas estarãojuntas, pela primeira vez, ao �nal de 1 metro, determine o número de pares(m,n) possíveis.

2015. Existem quantos números inteiros positivos n tais que ao dividir 2032por n temos resto 17?

2015. Existem quantos múltiplos de 99 com quatro dígitos distintos?

2016. Juca gosta de brincar com um número e a soma dos seus dígitos. Eledecidiu chamar um número inteiro s de sagaz se existe algum número n tal ques é a diferença entre n e a soma dos dígitos de n. Por exemplo, 18 é sagaz, poisele é igual a 28 - (2 + 8). Existem quantos números sagazes maiores que 1 emenores que 1000?

Page 113: OBM por assunto: Os primeiros passos olímpicos · tenha muito a acrescentar na sua vida pois o foco do treinamento é justamente para alcançar esse nível de habilidade. ... Esse

6.5. PROBLEMAS VARIADOS 113

6.5.2 Equações com inteiros

1998. Um fabricante de brinquedos embala bolas de pingue-pongue em doistipos de caixas. Num dos tipos ele coloca 10 bolas e no outro coloca 24 bolas.Num certo dia foram embaladas 198 bolas e usadas mais de 10 caixas. Quantascaixas foram feitas nesse dia?

2000. Quantos são os números inteiros de 2 algarismos que são iguais aodobro do produto de seus algarismos?

2001. Os pontos P1, P2, P3, · · · estão nesta ordem sobre uma circunferênciae são tais que o arco que une cada ponto ao seguinte mede 35◦. Qual é o menorvalor de n > 1 tal que Pn coincide com P1.

2004. Esmeralda, a digitadora, queria digitar um número N de dois algaris-mos que é quadrado perfeito, mas se enganou, trocando cada algarismo pelo seusucessor (a�nal, as teclas são vizinhas!). Por uma grande coincidência, o nú-mero digitado também é quadrado perfeito! Qual é a soma dos algarismos de N?

2007. Qual é número de pares (x, y) de inteiros positivos que satisfazem aequação

x8 + 3y4 = 4x2y3,

com 1 ≤ y ≤ 2007?

2012. Os algarismos não nulos A,B e C formam os números ABC,BCA eCAB tais que ABC+BCA+CAB = AAA×10. Quantos números ABC dessetipo existem?

2013. Se x e y são inteiros positivos tais que x(x+ 2 + 4 + 6 + · · ·+ 4024) =2013y, qual é o valor de y?

2014. Duas frações ab e c

d , a, b, c, d inteiros positivos, são íntimas quandoad− bc = ±1. Duas frações íntimas de 2014

51 têm denominador menor do que 51.Sendo x

y e zw essas frações, quanto vale y · w?

2014. Qual é o número de soluções inteiras (x, y, z) do sistema

x2 − 6y = 2z − 15

y2 − 6z = 2x− 15

z2 − 6x = 2y − 15?

2015. Os inteiros positivos x e y são tais que 12015 = 1

x −1y . Qual é o menor

valor possível para x+ y?

6.5.3 Outros Problemas

2006. Sejam x e y números racionais. Sabendo que x−5√

20064−y√

2006também é um

número racional, quanto vale o produto xy?

Page 114: OBM por assunto: Os primeiros passos olímpicos · tenha muito a acrescentar na sua vida pois o foco do treinamento é justamente para alcançar esse nível de habilidade. ... Esse

114 CAPÍTULO 6. TEORIA DOS NÚMEROS

2007. Dizemos que um natural X é um repunit quando os seus algarismossão todos iguais a 1, ou seja, quando X é da forma 11 · · · 1.

Sejam p, q e r inteiros, p > 0 , tais que pX2 + qX + r é um repunit sempreque X é um repunit. Encontre um possível valor de q?

2007. Versão curta: Qual é o menor inteiro positivo que pode ser escritocomo soma de dois quadrados perfeitos positivos de duas maneiras diferentes?

Versão longa: Um episódio muito conhecido na Matemática foi quando aovisitar o grande matemático Ramanujam no hospital, o outro grande matemá-tico Hardy disse que o número do táxi que o trouxe, 1729, era um número semgraça; Ramanujam respondeu prontamente: "Não diga isso, Hardy! 1729 é omenor número inteiro positivo que pode ser escrito como soma de dois cubos per-feitos positivos de duas maneiras diferentes!"De fato, 1729 = 103+93 = 123+13.Um outro episódio não muito conhecido na Matemática foi quando o pequenomatemático Muralijam foi visitado pelo outro pequeno matemático Softy, quedisse que o número do lotação que o trouxe era um número sem graça. Mu-ralijam responde imediatamente: "Não, Softy, ele é o menor inteiro positivoque pode ser escrito como soma de dois quadrados perfeitos positivos de duasmaneiras diferentes!"A que número Muralijam e Softy se referem?

2009. A famosa Conjectura de Goldbach diz que todo número inteiro parmaior que 2 pode ser escrito como a soma de dois números primos. Por exemplo,18 pode ser representado por 5 + 13 ou, ainda, por 7 + 11. Considerando todasas possíveis representações de 126, qual a maior diferença entre os dois primosque a formam?

2011. Se a, b e c são inteiros positivos tais que 1a + 1

b + 1c = 1

2011 , qual é omenor valor possível de a?

2011. Qual é o primeiro dígito não nulo após a vírgula na representaçãodecimal da fração 1

52012 ?

2015. Um conjunto �nito A de números reais é parfeito quando tem pelomenos dois elementos e {ab : a, b ∈ A e a 6= b} = A, ou seja, o conjuntoobtido multiplicando-se todos os pares de números distintos de A é o próprioA. Por exemplo, {1, 2, 1

2} é parfeito pois {1 · 2, 1 · 12 , 2 ·

12} = {2, 1

2 , 1}, mas{1, 2, 3} 6= {1 · 2, 1 · 3, 2 · 3} não é parfeito. Quantos elementos pode ter umconjunto parfeito?

A) Somente 3 ou 4 ;B) Qualquer quantidade congruente a 3 ou 4 módulo 4;C) Qualquer quantidade ímpar ;D) Qualquer quantidade prima ímpar;E) Qualquer quantidade maior do que 2.

Page 115: OBM por assunto: Os primeiros passos olímpicos · tenha muito a acrescentar na sua vida pois o foco do treinamento é justamente para alcançar esse nível de habilidade. ... Esse

Capítulo 7

Geometria

Pré-requisito: ângulos raso e reto; casos de congruência de ângulo (alternosinternos e externos), teorema de Tales, critérios de semelhança e de congruênciade triângulos, de�nição das �guras planas usuais, de�nição de seno e cosseno.

7.1 Fatos básicos

7.1.1 Desigualdade triangular

De�nição 14 Uma circunferência com centro O e raio r é o conjunto dospontos que se encontram a uma distância igual a r do ponto O.

Um círculo com centro O e raio r é o conjunto dos pontos que se encontrama uma distância menor ou igual a r do ponto O.

2006. Dois pontos A e B de um plano α estão a 8 unidades de distância.Quantas retas do plano α estão a 2 unidades de A e 3 unidades de B?

Um fato muito importante sobre triângulos é: se a soma de dois dos ladosfor menor que o terceiro o triângulo não fecha. Enunciaremos isso precisamenteabaixo.

Teorema 21 (Desigualdade Triangular) Sejam a, b, c os comprimentos doslados de um triângulo. Então a+ b > c, a+ c > b e b+ c > a.

115

Page 116: OBM por assunto: Os primeiros passos olímpicos · tenha muito a acrescentar na sua vida pois o foco do treinamento é justamente para alcançar esse nível de habilidade. ... Esse

116 CAPÍTULO 7. GEOMETRIA

Como o comprimento do segmento O1O2 é maior que a soma dos compri-mentos dos raios r1 +r2, o triângulo formado por O1O2 e pelos raios dos círculos"não fecha".

OBM

2001. Para cada ponto pertencente ao interior e aos lados de um triânguloacutânguloABC, considere a soma de suas distâncias aos três lados do triângulo.O valor máximo desta soma é igual

A) à média aritmética das 3 alturas do triângulo.B) ao maior lado do triângulo.C) à maior altura do triângulo.D) ao triplo do raio do círculo inscrito no triângulo.E) ao diâmetro do círculo circunscrito ao triângulo.

2006. Sejam a, b e c números reais positivos cuja soma é 1. Se a, b e c são asmedidas dos lados de um triângulo, podemos concluir que

A) 0 < |a− b| < 12 e 0 < |b− c| < 1

2 e 0 < |c− a| < 12 .

B) a < 12 e b < 1

2 e c < 12 .

C) a+ b < 12 e b+ c < 1

2 e c+ a < 12 .

D) a ≤ 13 e b ≤ 1

3 e c ≤ 13 .

E) a ≥ 13 e b ≥ 1

3 e c ≥ 13 .

2007. Qual o menor perímetro inteiro possível de um triângulo que possuium dos lados com medida igual a 5

√3

2 ?

7.1.2 Figuras planas básicas

A seguir de�niremos as �guras planas básicas. Suporemos que o leitor saiba oque são um triângulo e um quadrilátero. Sugerimos que use o Google ou seubuscador favorito para conhecer a aparência dessas �guras.

1. Um triângulo escaleno é um triângulo cujos três lados são diferentes (eportanto os três ângulos são diferentes).

2. Um triângulo isósceles é um triângulo com (pelo menos) dois ladosiguais (e portanto dois ângulos iguais).

3. Um triângulo equilátero é um triângulo com três lados iguais (e por-tanto três ângulos iguais). Em particular todo triângulo equilátero é isós-celes.

4. Um triângulo retângulo é um triângulo que possui um ângulo igual a90o.

5. Um paralelogramo é um quadrilátero tal que cada par de lados opostosé paralelo entre si.

6. Um trapézio é um quadrilátero que possui um par de lados paralelos.Chamamos de base maior e base menor, ao maior e ao menor desses lados,respectivamente. Se os dois lados forem iguais, o quadrilátero será um

Page 117: OBM por assunto: Os primeiros passos olímpicos · tenha muito a acrescentar na sua vida pois o foco do treinamento é justamente para alcançar esse nível de habilidade. ... Esse

7.1. FATOS BÁSICOS 117

paralelogramo. Alguns autores não chamam paralelogramos de trapézios,mas aqui será chamado por conveniência.

7. Um trapézio isósceles é um trapézio cujos lados que não são as basessão iguais. Paralelogramos também serão chamados trapézios isósceles.

8. Um retângulo é um quadrilátero tal que todos os ângulos são iguais.

9. Um losango é um quadrilátero com 4 lados iguais. Todo losando é umparalelogramo.

10. Um quadrado é um losango que é também um retângulo.

11. Um n-ágono é um polígono com n lados. Trocamos n por seu nomegrego quando convém. Por exemplo: um 5-ágono, 6-ágono e 7-ágono sãochamados pentágono, hexágono e heptágono, respectivamente.

12. Um polígono é dito regular se tem todos os lados com o mesmo compri-mento e todos os ângulos iguais. Observe que ter todos os lados iguais nãoimplica em ter todos os ângulos iguais, assim como ter todos os ângulosiguais não implica ter todos os lados iguais, a não ser no caso do triângulo.

7.1.3 A soma dos ângulos internos

Outro fato importante sobre triângulos diz respeito a soma dos seus ângulosinternos.

Teorema 22 A soma dos ângulos internos de um triângulo é igual a 180o.

Prova:

Exemplo:a) Calcule a soma dos ângulos internos de um quadrilátero convexo.

Page 118: OBM por assunto: Os primeiros passos olímpicos · tenha muito a acrescentar na sua vida pois o foco do treinamento é justamente para alcançar esse nível de habilidade. ... Esse

118 CAPÍTULO 7. GEOMETRIA

b) Calcule o ângulo externo do triângulo em termo dos ângulos α e β.

c) Calcule o ângulo externo do quadrilátero não convexo.

d) Calcule o valor dos ângulos internos de um n-ágono regular para n =3, 4, 5, 6, 7, 8, 9. Calcule mesmo, não perceba que sabe calcular e pule o exercício.

OBM

2001. O triângulo CDE pode ser obtidopela rotação do triângulo ABC de 90o

no sentido anti-horário ao redor de C,conforme mostrado no desenho abaixo.Quanto vale o ângulo α?

2010. No triângulo ABC, ∠BAC = 140◦. Sendo M o ponto médio de BC,N o ponto médio de AB e P o ponto sobre o lado AC tal que MP é perpendi-cular a AC, qual é a medida do ângulo ∠NMP ?

2011. Em um triângulo ABC com ∠ABC − ∠BAC ≤ 50o , a bissetriz doângulo ∠ACB intersecta o lado AB em D. Seja E o ponto do lado AC tal que∠CDE = 90◦. Quanto mede o ângulo ∠ADE?

Se temos um polígono convexo, podemos fatiá-lo em pedaços triangulares eusar o teorema acima repetidamente de modo a obter que

Page 119: OBM por assunto: Os primeiros passos olímpicos · tenha muito a acrescentar na sua vida pois o foco do treinamento é justamente para alcançar esse nível de habilidade. ... Esse

7.1. FATOS BÁSICOS 119

Teorema 23 A somas do ângulos internos de um polígono convexo com n ladosé (n− 2)180o.

Prova: Exercício de indução: Considere três vértices consecutivos A,B,Ce trace a diagonal de A a C dividindo o polígono em um polígono com um ladoa menos e um triângulo. �

OBM

1998. Todos os ângulos internos de um polígono convexo são menores que (nãopodendo ser iguais a) 160o. Qual é o número máximo de lados desse polígono?

2000. DEFG é um quadrado no exterior do pentágono regular ABCDE.Quanto mede o ângulo ∠EAF?

2001. ABCDE é um pentágono regular e ABF é um triângulo equiláterointerior. Quanto mede o ângulo ∠FCD?

2004. Constrói-se o quadrado ABXY sobre o lado AB do heptágono regu-lar ABCDEFG, exteriormente ao heptágono. Determine a medida do ângulo∠BXC, em radianos.

2006. Três quadrados são colados pelos seus vértices entre si e a dois bastõesverticais, como mostra a �gura. Quanto mede o ângulo x?

2007. Determine em qual dos horários abaixo o ângulo determinado pelosponteiros de um relógio é o menor.

A) 02h30. B) 06h20. C) 05h40. D) 08h50. E) 09h55.

2007. A �gura mostra dois qua-drados sobrepostos. Qual é ovalor de x+ y, em graus?

2008. Dado o quadrilátero ABCD tal que ∠CAD = 25◦,∠ACD = 45◦ e∠BAC = ∠BCA = 20◦, qual o valor do ângulo ∠DBC?

Page 120: OBM por assunto: Os primeiros passos olímpicos · tenha muito a acrescentar na sua vida pois o foco do treinamento é justamente para alcançar esse nível de habilidade. ... Esse

120 CAPÍTULO 7. GEOMETRIA

2008. No desenho temos AE =BE = CE = CD. Além disso,α e β são medidas de ângulos.Qual é o valor da razão α

β .

2009. O relógio de parede indica inicialmente 12h. Que horas os ponteirosdas horas e dos minutos irão formar um ângulo de 145 graus pela primeira vez?

2009. Na �gura, CD = BC,∠BAD = 72◦, AB é o diâmetro eO o centro do semicírculo. Determine amedida do ângulo ∠DEC.

2009. Na �gura, o quadrado A′B′C ′D′

foi obtido a partir de uma rotação nosentido horário do quadrado ABCD de25 graus em torno do ponto médio deAB. Qual é o ângulo agudo, em graus,entre as retas AC e B′D′?

2010. Os pontos P,Q,R, S e T são vér-tices de um polígono regular. Os ladosPQ e TS são prolongados até se encon-trarem em X, como mostra a �gura, e∠QXS mede 140o. Quantos lados o po-lígono tem?

2013. Na �gura a seguir, AB1 = B1B2 = B2B3 = · · · = Bn−1Bn. Os pontosB1, B3, B5, · · · pertencem a uma reta e os pontos B2, B4, B6, · · · pertencem aoutra reta. Todos os pontos B1, B2, B3, · · · , Bn são distintos. Sabendo que o

ângulo ∠B1AB2 mede 1o, qual é o maior valor possível de n?

Page 121: OBM por assunto: Os primeiros passos olímpicos · tenha muito a acrescentar na sua vida pois o foco do treinamento é justamente para alcançar esse nível de habilidade. ... Esse

7.2. PITÁGORAS E TRIÂNGULO RETÂNGULO 121

2016. Na �gura a seguir sabe-se queABCDEFGH é um octógono regu-lar, ABIJK é um pentágono regular eABLM é um quadrado. Determine amedida em graus do ângulo ∠ICL re-presentado na �gura pela letra x.

7.1.4 Diagonais

Teorema 24 Um polígono convexo com n lados possuin(n− 3)

2diagonais.

Prova: Exercício de contagem.

7.2 Pitágoras e Triângulo Retângulo

7.2.1 Semelhança de triângulos

Exercícios

1. Mostre que cada par de ângulos opostos no paralelogramo é igual. Mostretambém que se os ângulos opostos de um quadriláteros são iguais, entãoo quadrilátero é um paralelogramo. (Use ângulos alternos)

2. Mostre que um losango é um paralelogramo.

3. Mostre que as diagonais do losango são perpendiculares.

4. Mostre que dois ângulos adjacentes a um lado que não é uma base numtrapézio somam 180o.

OBM

2006. Na �gura a seguir, ABC é umtriângulo qualquer e ACD e AEB sãotriângulos eqüiláteros. Se F e G sãoos pontos médios de EA e AC, res-pectivamente. Quanto vale a razão BD

FG ?

2008. No triângulo PQR isósceles, com PQ = PR = 3 e QR = 2, a tangenteà sua circunferência circunscrita no ponto Q encontra o prolongamento do ladoPR em X. Qual é o valor de RX?

Page 122: OBM por assunto: Os primeiros passos olímpicos · tenha muito a acrescentar na sua vida pois o foco do treinamento é justamente para alcançar esse nível de habilidade. ... Esse

122 CAPÍTULO 7. GEOMETRIA

7.2.2 Teorema de Pitágoras

Considere ABC é um triângulo retângulo com ∠A = α,∠B = β = 90o−α,∠C =90o. Além disso H será o pé da altura relativa ao lado AB.

Exercício: Mostre que sin(90− α) = cos(α).

Problema: Faça a �gura e mostre que AHAC = AC

AB e BHBC = BC

BA .

Teorema 25 (Pitágoras) Seja ABC um triângulo com ∠C = 90o, então

AC2 +BC2 = AB2.

Prova: Use o problema acima e prove o teorema de Pitágoras.

Exemplo: Calcule o comprimento da diagonal de um quadrado de lado `.

Pelo teorema de Pitágoras, temos

d2 = `2 + `2 ⇒ d2 = 2`2 ⇒ d = `√

2.

Exercício: Calcule a altura de um triângulo equilátero de lado `.

Page 123: OBM por assunto: Os primeiros passos olímpicos · tenha muito a acrescentar na sua vida pois o foco do treinamento é justamente para alcançar esse nível de habilidade. ... Esse

7.2. PITÁGORAS E TRIÂNGULO RETÂNGULO 123

OBM

1999. Um quadrado ABCD possui lado 40cm. Uma circunferência contém osvértices A e B e é tangente ao lado CD. Calcule o raio desta circunferência.

2000. A �gura ao lado mostra o logotipo de umaempresa, formado por dois círculos concêntricose por quatro círculos de mesmo raio, cada umdeles tangente a dois dos outros e aos dois círcu-los concêntricos. O raio do círculo interno mede1 cm. Então quanto o raio do círculo externodeverá medir, em cm?

2001. Uma mesa retangular, cujos pés têm ro-das, deve ser empurrada por um corredor de lar-gura constante, que forma um ângulo reto. Seas dimensões da mesa são a e b (com 2a < b),qual deve ser a largura mínima do corredor paraque a mesa possa ser empurrada através dele?

2003. A �gura a seguir mostra um quadradoABCD e um triângulo eqüilátero BEF, amboscom lado de medida 1cm. Os pontos A, B e Esão colineares, assim como os pontos A, G e F.Qual é área do triângulo BFG, em cm2?

2003. A �gura abaixo mostra duas retas paralelas r e s. A reta r é tangenteàs circunferências C1 e C3, a reta s é tangente às circunferências C2 e C3 e ascircunferências tocam-se como também mostra a �gura.

As circunferências C1 e C2 têm raios a e b, respectivamente. Qual é o raioda circunferência C3?

2004. Dois espelhos formam um ângulo de 30o no ponto V . Um raio de luz,vindo de uma fonte S, é emitido paralelamente a um dos espelhos e é re�etidopelo outro espelho no ponto A, como mostra a �gura. Depois de uma certa

Page 124: OBM por assunto: Os primeiros passos olímpicos · tenha muito a acrescentar na sua vida pois o foco do treinamento é justamente para alcançar esse nível de habilidade. ... Esse

124 CAPÍTULO 7. GEOMETRIA

quantidade de re�exões, o raio retorna a S. Se AS e AV têm 1 metro decomprimento, qual é a distância percorrida pelo raio de luz?

2007. Uma mesa de bilhar tem dimensões de 3 metros por 6 metros e temcaçapas nos seus quatro cantos P,Q,R e S. Quando uma bola bate na borda damesa, sua trajetória forma um ângulo igual ao que a trajetória anterior formava.Uma bola, inicialmente a 1 metro da caçapa P , é batida do lado SP em direçãoao lado PQ, como mostra a �gura. A quantos metros de P a bola acerta o ladoPQ se a bola cai na caçapa S após duas batidas na borda da mesa?

2010. Um quadrado PQRS tem ladosmedindo x. T é o ponto médio de QRe U é o pé da perpendicular a QS quepassa por T . Qual é a medida de TU?

2013. Seja ABC um triângulo retângulo em A. Seja D o ponto médio deAC. Sabendo que BD = 3DC e que AC = 2, quanto vale a hipotenusa dotriângulo?

2014. Na �gura, ABC e DEF são triângulosretângulos isósceles com hipotenusas BC e EFmedindo 15, D está sobre a reta BC e A estásobre a reta EF . O ângulo agudo entre as retasBC e EF é 30o.Quanto mede AD?

Page 125: OBM por assunto: Os primeiros passos olímpicos · tenha muito a acrescentar na sua vida pois o foco do treinamento é justamente para alcançar esse nível de habilidade. ... Esse

7.2. PITÁGORAS E TRIÂNGULO RETÂNGULO 125

2015. No desenho ao lado, o segmento CF étangente ao semicírculo de diâmetro AB. SeABCD é um quadrado de lado 4, determine ocomprimento de CF .

2015. Em cada ponto do plano cartesiano com ambas as coordenadas in-teiras, construímos círculos de raio r. Qual é o menor valor de r para o qualqualquer circunferência de raio 1 (com centro de coordenadas reais quaisquer)corte algum dos círculos de raio r?

2016. Em um jogo de videogame, um personagem, o OBMario, se desloca emuma tela de medidas 3 e 4; quando ele chega no limite da tela, ele reaparece nolado oposto (se sai pela esquerda, volta à direita, à mesma altura, e vice-versa;se sai por cima, volta por baixo, à mesma distância do lado esquerdo da tela,e vice-versa). Chamamos o menor trajeto de OBMario entre dois pontos dedistância de videogame. Qual é a maior distância de videogame possível entredois pontos da tela?

7.2.3 Potência de Ponto

Lema 3 (Da tangente) Seja s uma reta tangente no ponto P a circunferênciaΓ com centro O. Então PO ⊥ s.

Teorema 26 Seja P um ponto e Γ uma circunferência. Para toda reta quepassa por P intersecta a circunferência nos pontos A e B, o produto PA · PBé o mesmo.

Prova:Sejam O o centro e R o comprimento do raio de Γ e M um ponto sobre PB

tal que ∠OMP = 90o.Use esses objetos ea) Mostre que PA · PB = PO2 −R2.b) Mostre que se uma reta PT é tangente a Γ em T , então PT 2 = PO2−R2.c) Mostre que se P é um ponto no interior do círculo então PA · PB =

R2 − PO2.

Page 126: OBM por assunto: Os primeiros passos olímpicos · tenha muito a acrescentar na sua vida pois o foco do treinamento é justamente para alcançar esse nível de habilidade. ... Esse

126 CAPÍTULO 7. GEOMETRIA

O valor PotPΓ := PO2 −R2 é chamado de potência de P com respeito a Γ.

Uma corda é um segmento cujos extremos pertencem a uma circunferência.

Exercícios

1. Duas cordas AB e CD se intersectam no ponto P :

a) Determine a medida de BP , sabendo que os segmentos CP,DP e ABmedem respectivamente 1cm, 6cm e 5cm.

b) AP ·BP = 25 e CP = DP4 . Determine CD.

2. Sejam AB e AC duas cordas com medidas iguais. Uma corda AD inter-cepta a corda BC no ponto P . Prove que os triângulos ABD e APB sãosemelhantes.

3. Dois círculos Γ1 e Γ2 intersectam-se em P e Q. Uma reta passando por Pintersecta Γ1 e Γ2 novamente em A e B, respectivamente, se X é o pontomédio de AB e a reta que passa por Q e X intersecta Γ1 e Γ2 novamenteem Y e Z, respectivamente. Prove que X é o ponto médio de Y Z.

7.3 Informações básicas de trigonometria

Começamos essa seção com a relação base de trigonometria, sua importância éexpressa pelo seu título.

Teorema 27 (Relação Fundamental da Trigonometria) Seja α um dos ân-gulos que não é reto de um triângulo retângulo, então

sin2 α+ cos2 α = 1.

Prova: Use o teorema de pitágoras para mostrar isso.

Exercício: Calcule o valor máximo de:a) sin(x) · cos(x);b) 4 sinx+ 3 cosx.

Page 127: OBM por assunto: Os primeiros passos olímpicos · tenha muito a acrescentar na sua vida pois o foco do treinamento é justamente para alcançar esse nível de habilidade. ... Esse

7.3. INFORMAÇÕES BÁSICAS DE TRIGONOMETRIA 127

7.3.1 Seno e Cosseno como funções reais

Seno, cosseno e tangente, podem ser vistas como funções reais ou funções dosgraus. A tradução de grau pra número real decorre da tradução de grau pararadianos. Trataremos de graus, se o leitor souber trabalhar com radiano, �cacomo exercício fazer a tradução, se o leitor não souber trabalhar com radiano,sugerimos que não esquente a cabeça, não se preocupe com isso durante a leitura,e vá aprender após a leitura.

A seguir listaremos alguns fatos sobre tais funções (não iremos dar provas).

A função seno1. É uma função ímpar, ou seja, sin(−α) = − sinα;2. É crescente no intervalo [0o, 90o] e decrescente no intervalo [90o, 180o].

A função cosseno1. É uma função par, ou seja, cos(−α) = cosα;2. É decrescente no intervalo [0o, 180o];3. É injetiva no intervalo [0o, 180o] (isso decorre de 2);4. Possui inversa no intervalo [0, 180o] (isso decorre de 3).

7.3.2 Ângulos notáveis

A seguir daremos alguns ângulos para os quais são fáceis calcular os seus senose cossenos. Para realizar esses cálculos, utilize o triângulo isóceles com ângulomaior de 90o e uma das metades do triângulo equiláteros partido em dois poruma das suas alturas/medianas/bissetrizes.

• sin 30o = 12 , cos 30o =

√3

2 ;

• sin 45o =√

22 , cos 45o =

√2

2 ;

• sin 90o = 1;

• cos 180o = −1.

Exercício Calcule sin 60o, cos 60o, cos 90o, sin 180o.

1999. Se 0o < x < 90o e cosx = 14 então x está entre:

A) 0o e 30o. B)30o e 45o. C) 45o e 60o. D) 60o e 75o. E) 75o e 90o.

7.3.3 Fórmulas úteis

Teorema 28 (Seno da soma) Sejam α, β ângulos quaisquer, então valem asseguintes fórmulas

sin(α+ β) = sinα cosβ + sinβ cosα

Prova: Essa demonstração será omitida.

Teorema 29 Sejam α e β ângulos quaisquer, então valem as seguintes fórmulasi) sin(α− β) = sinα cosβ − sinβ cosα;

Page 128: OBM por assunto: Os primeiros passos olímpicos · tenha muito a acrescentar na sua vida pois o foco do treinamento é justamente para alcançar esse nível de habilidade. ... Esse

128 CAPÍTULO 7. GEOMETRIA

ii) cos(α+ β) = cosα cosβ − sinβ sinα;

iii) cos(α− β) = cosα cosβ + sinβ sinα.

Prova: Exercíco. Use a fórmula do seno da soma, e fatos como sin(−α) =− sinα, sin 90o = 1, cos 180o = −1.

Exercícios

1. Encontre fórmulas simples para sin(2α), cos(2α), sin(α2 ), cos(α2 ) em termosde sinα e cosα.

2. Calcule sin(180o − α) e cos(180o − α).

3. Calcule sin e cos dos ângulos: 15o, 75o, 120o.

Teorema 30 Sejam α, β ângulos quaisquer, então valem as seguintes fórmulas

i) sinα+ sinβ = 2 sin(α+β

2

)cos(α−β

2

);

ii) sinα− sinβ = 2 sin(α−β

2

)cos(α+β

2

);

iii) cosα+ cosβ = 2 cos(α+β

2

)cos(α−β

2

);

iv) cosα− cosβ = −2 sin(α+β

2

)sin(α−β

2

).

Prova: Use as fórmulas anteriores para provar isso.

7.3.4 Substituição Trigonométrica

Passaremos brevimente a seguir por uma técnica muito interessante, porém depouca obiquidade no contexto dos nossos estudos. Trata-se da substituição tri-gonométrica. Aplicaremos várias das informações acima no exemplo a seguir.

Exemplo: Sejam a, b, c e d reais tais que a2+b2 = 1, c2+d2 = 1 e ac+bd = 0.Determine ab+ cd.

De início esse problema pode parecer não ter nada a ver com trigonometria,mas se você pensar na equação fundamental da trigonometria as coisas mudam.Sempre que surge uma expressão a2 + b2 = 1 podemos supor a existência dealgum α (não necessariamente único) tal que a = sinα e b = cosα. O mesmoocorre quando se tem alguma restrição do tipo −1 ≤ x ≤ 1, ou até mesmoquando temos algo do tipo −r ≤ x ≤ r, onde podemos tomar x = r sinα oux = r cosα.

Solução:Como a2 + b2 = 1 e c2 + d2 = 1, então existem α e β tais que a = sinα,

b = cosα, c = sinβ e d = cosβ, com 0o ≤ α ≤ 360o e 0o ≤ β ≤ 360o. Nestecaso temos

0 = ac+ bd = sinα · sinβ + cosβ · cosα = cos(α− β),

Page 129: OBM por assunto: Os primeiros passos olímpicos · tenha muito a acrescentar na sua vida pois o foco do treinamento é justamente para alcançar esse nível de habilidade. ... Esse

7.3. INFORMAÇÕES BÁSICAS DE TRIGONOMETRIA 129

e

ab+ cd = sinα · cosα+ cosβ · sinβ

=1

2(sin 2α+ sin 2β)

= sin(α+ β) · cos(α− β)

= 0.

(Armênia-1999) Resolva a equação1

x2+

1

(4−√

3x)2= 1.

A solução a seguir é a que Geraldo Perlino de São Paulo mandou para arevista Eureka 11.

Solução:

Seja θ algum ângulo tal que 1x = cos θ e

1

(4−√

3x)2= sin θ então

√3

cos θ+

1

sin θ= 4

√3

2sin θ +

1

2cos θ = 2

sin(θ + 30o) = 2 sin θ cos θ

sin(θ + 30o) = sin(2θ).

Assim concluímos que os valores possíveis de θ são 30o, 50o, 170o e 290o.

OBM

2007. As cidades Aópolis, Beópolis e Ceópolis são ligadas por estradas retas.Sabe-se que a estrada que liga Aópolis e Beópolis é perpendicular à estrada queliga Aópolis e Ceópolis. Rubens mora em Beópolis e tem um compromisso emCeópolis. Todavia, a estrada que liga Beópolis a Ceópolis está interditada, demodo que Rubens é obrigado a fazer o trajeto Beópolis-Aópolis-Ceópolis. Parachegar ao compromisso na hora certa, Rubens trafega com uma velocidade 24%maior do que trafegaria se utilizasse a estrada interditada. Se α é o menorângulo do triângulo determinado pelas três estradas, então

A) 0 < tgα < 16 .

B) 16 < tgα < 1

5 .C) 1

5 < tgα < 14 .

D) 14 < tgα < 1

3 .E) 1

3 < tgα < 1.

2008. Qual é o número de soluções reais do sistema

a2 = b+ 2

b2 = c+ 2

c2 = a+ 2?

O desa�o é descobrir porque esse problema está na seção de trigonometria!

Page 130: OBM por assunto: Os primeiros passos olímpicos · tenha muito a acrescentar na sua vida pois o foco do treinamento é justamente para alcançar esse nível de habilidade. ... Esse

130 CAPÍTULO 7. GEOMETRIA

7.4 Lei dos Cossenos

Teorema 31 No triângulo ABC sejam a = BC, b = CA e c = AB. Então

a2 = b2 + c2 − 2bc cos(∠BAC).

Prova:O desenho a seguir representa o caso em que o triângulo tem todos os ângulos

internos agudos. Use o teorema de Pitágoras para fazer esse caso.

Faça o desenho para o caso obtuso e use o teorema de Pitágoras para mostrara Lei dos cossenos.�

Claro que, analogamente temos b2 = c2 + a2 − 2ca cos∠CBA e c2 = a2 +b2 − 2ab cos∠ACB.

A lei dos cossenos é uma recíproca generalizada do teorema de Pitágoras,pois ela diz que se o triângulo de lados a > b ≥ c não é retângulo (teremosque cosA 6= 0 e) então não vale a fórmula a2 = b2 + c2 (e diz muito mais, poisexplica como calcular os lados em termos dos outros).

1999. Para quantos valores inteiros de x existe um triângulo acutângulo delados 12, 10 e x?

2003. No triângulo ABC,AB = 20, AC = 21 e BC = 29. Os pontos D e Esobre o lado BC são tais que BD = 8 e EC = 9. Qual é a medida do ânguloDAE?

2005. Os pontos L,M e Nsão pontos médios de arestasdo cubo, como mostra a �gura.Quanto mede o ângulo ∠LMN?

2013. Super Esmeralda e Jade Maravilha estão jogando bilhar. Super Esme-ralda dá uma tacada em uma bola com velocidade de 60km/h, com um ângulo

Page 131: OBM por assunto: Os primeiros passos olímpicos · tenha muito a acrescentar na sua vida pois o foco do treinamento é justamente para alcançar esse nível de habilidade. ... Esse

7.5. PONTOS, RETAS E SEGMENTOS NOTÁVEIS 131

de 30o com uma das tabelas. Jade Maravilha deve acertar a bola de SuperEsmeralda com outra bola. As duas bolas partem da tabela da mesa simul-taneamente, e estão a uma distância de 50cm. Jade Maravilha pode escolherqualquer ângulo para dar a sua tacada.

Qual é a velocidade mínima com que Jade Maravilha pode dar sua tacada?

7.5 Pontos, retas e segmentos notáveis

7.5.1 Bissetrizes

De�nição 15 A bissetriz de um ângulo é a semireta saindo do vértice que divideo ângulo em duas parte iguais.

Teorema 32 Seja AD a bissetriz do ângulo ∠BAC no triâgulo ABC. Então

AB

BD=AC

CB.

Prova: Seja A′ o ponto sobre a bissetriz tal que BA = BA′ e P o ponto deintersecção entre AA′ e BC. Use semelhança de triângulos em BA′P e CAPpara provar o resultado desejado. �

A demonstração acima consiste de escolher o ponto certo, ou seja, o pontoque faz um segmento ser congruente a outro segmento, e o resto é bem simples.Para praticar essa ideia de escolher o ponto esperto faça o seguinte problemada OBM:

2013. No trapézio ABCD, com AB paralelo a CD, o ângulo ∠BAD mede82o e o ângulo ∠ABC mede 74o. Suponha que exista um ponto P sobre o ladoCD tal que AD +DP = PC + CB = AB. Quanto mede o ângulo ∠APB?

Teorema 33 Os pontos da bissetriz são equidistantes dos lados que formam oângulo bissectado. Se P é equidistante dos dois lados de um triângulo então Ppertence a bissetriz do ângulo formado por esses dois lados.

Prova: Exercício.

Page 132: OBM por assunto: Os primeiros passos olímpicos · tenha muito a acrescentar na sua vida pois o foco do treinamento é justamente para alcançar esse nível de habilidade. ... Esse

132 CAPÍTULO 7. GEOMETRIA

Exercício: Encontre todos os possíveis valores de α− θ na �gura abaixo.

Teorema 34 As três bissetrizes dos ângulos de um triângulo se encontram emum único ponto interno do triângulo. Chamamos esse ponto de incentro.

Prova: Use o exercício acima.

Teorema 35 O incentro de um triângulo é equidistante dos seus três lados.

Assim, existe uma circunferência (e portanto um círculo) com centro em Ique tangencia os três lados do triângulo, ou seja, uma circunferência inscrita aotriângulo. Chamamos o círculo de incírculo. O raio do incírculo é chamado deinraio.

2008. Dado um triângulo ABC de lados AB = 3, BC = 4 e AC = 5. SejamR1 e R2, respectivamente, os raios da circunferência inscrita e da circunferênciacom centro sobre o lado BC que passa por B e é tangente ao lado AC. Quantovale a razão R1

R2?

Page 133: OBM por assunto: Os primeiros passos olímpicos · tenha muito a acrescentar na sua vida pois o foco do treinamento é justamente para alcançar esse nível de habilidade. ... Esse

7.5. PONTOS, RETAS E SEGMENTOS NOTÁVEIS 133

7.5.2 Alturas

De�nição 16 Uma altura de um triângulo é um segmento saindo de um vérticedo triângulo que é perpendicular ao lado oposto a esse vértice.

A altura com respeito ao lado BC do triângulo ABC é a que sai de A paraBC, nesta caso o ponto A′ de intersecção da altura com o lado BC é chamadode pé da altura.

Teorema 36 As três alturas de um triângulo se encontram em um único pontoH. Chamamos H de ortocentro do triângulo.

Se o triângulo é acutângulo, esse ponto é interno ao triângulo. Se o triânguloé retângulo, esse ponto é o vértice do ângulo reto. Se o triângulo é obtuso, esseponto é externo ao triângulo.

De�nição 17 Dado um triângulo acutângulo ABC, sejam A′, B′, C ′ os pésdas alturas saindo dos vértices A,B,C, respectivamente. O triângulo A′B′C ′ échamado triângulo órtico de A,B,C.

Exercício: Sejam ABC um triângulo com triângulo órtico A′B′C ′.

a) Calcule a razão∠AA′B′

∠AA′C ′;

b) Encontre o incentro de A′B′C ′.

7.5.3 Medianas

De�nição 18 A mediana de um lado de um triângulo é o segmento que vai dovértice oposto a esse lado até o ponto médio do lado.

Teorema 37 As três medianas dos lados de um triângulo se encontram em umúnico ponto interno do triângulo. Chamamos esse ponto de baricentro.

Teorema 38 No triângulo ABC sejam M e N os pontos médios de AB e AC

respectivamente. Então MN =BC

2. Além disso o triângulo formado pelos

pontos médios dos três lados é semelhante a ABC

Prova: Semelhança de triângulos.

O segmentoMN é chamado base média, e o triângulo cujos vértices são ospontos médios do lado de um triângulo é chamado triângulo medial. às vezesé muito útil levar a base média em consideração, portanto, quando vir pontosmédios e não tiver ideia do que fazer, use a base média.

2009. Os lados de um triângulo formam uma progressão aritmética de razãot. Então qual é a distância entre o incentro e o baricentro deste triângulo?

7.5.4 Mediatrizes

De�nição 19 A mediatriz de um segmento AB é a reta que passa pelo pontomédio de AB perpendicularmente a AB.

Page 134: OBM por assunto: Os primeiros passos olímpicos · tenha muito a acrescentar na sua vida pois o foco do treinamento é justamente para alcançar esse nível de habilidade. ... Esse

134 CAPÍTULO 7. GEOMETRIA

Teorema 39 Os pontos da mediatriz de um segmento são equidistantes dosextremos do segmento.

Prova: Exercício.

As mediatrizes de um triângulo são as mediatrizes dos seus lados.

Teorema 40 As três mediatrizes de um triângulo se encontram em um únicoponto O. Chamamos O de circuncentro do triângulo.

Prova: Exercício.

Se o triângulo é acutângulo, esse ponto é interno ao triângulo. Se o triânguloé retângulo, esse ponto é o ponto médio da hipotenusa. Se o triângulo é obtuso,esse ponto é externo ao triângulo.

Teorema 41 O circuncentro é equidistante dos vértices do triângulo.

Assim, existe uma circunferência (e portanto um círculo) com centro em Oe que possui três vértices do triângulo, ou seja, uma circunferência circunscritaao triângulo. Chamamos o círculo de circuncírculo.

7.6 Ângulos Inscritos e Ângulos Tangentes

7.6.1 Ângulos tangentes

Lema 4 Dada uma circunferência Γ e um ponto P externo a ela, sejam r es as retas tangentes a Γ, em Q e R respectivamente, passando por P . EntãoPQ = PR.

Prova:

Use o lema da tangente e o teorema de pitágoras. �

Page 135: OBM por assunto: Os primeiros passos olímpicos · tenha muito a acrescentar na sua vida pois o foco do treinamento é justamente para alcançar esse nível de habilidade. ... Esse

7.6. ÂNGULOS INSCRITOS E ÂNGULOS TANGENTES 135

Teorema 42 (Identidades de Ravi) Sejam a, b e c os comprimentos dos la-dos de um triângulo. Então existem r, s, t > 0 tais que a = r + s, b = s+ t, c =t+ r.

Prova: Use o incírculo e o lema da tangente. �

Teorema 43 (Pitot) Seja ABCD um quadrilátero convexo. Então ABCDestá circunscrito a uma circunferência, se e somente se AB+CD = AD+BC.

Prova: Use o lema da tangente.�

1999. Os pontos S, T e U são os pontos de tangência do círculo inscrito notriângulo PQR sobre os lados RQ,RP e PQ respectivamente. Sabendo que oscomprimentos dos arcos TU, ST e US estão na razão TU : ST : US = 5 : 8 : 11,qual é a razão ∠TPU : ∠SRT : ∠UQS?

2001. O hexágono ABCDEF é circunscritível. Se AB = 1, BC = 2, CD =3, DE = 4 e EF = 5, quanto mede FA?

2012. Seja ABCD um quadrilátero convexo tal que AB = BC = CD = 1e ∠ABD = ∠ACB. Sabendo que as medidas, em graus, dos ângulos ∠ABD e∠ACD são inteiras, determine quantos quadriláteros ABCD podem ser cons-truídos satisfazendo as condições acima.

Page 136: OBM por assunto: Os primeiros passos olímpicos · tenha muito a acrescentar na sua vida pois o foco do treinamento é justamente para alcançar esse nível de habilidade. ... Esse

136 CAPÍTULO 7. GEOMETRIA

2005. Na �gura, a reta PQ toca em N ocírculo que passa por L,M e N . A retaLM corta a reta PQ em R. Se LM =LN e a medida do ângulo PNL é α, α <60◦, quanto mede o ângulo LRP?

2014. Considere a �gura ao lado, onde os pon-tos de A até I estão sobre uma circunferência.Sabe-se que os triângulos ABC e GHI são isós-celes, que AB, CD, EF e GH são segmentosparalelos e que BC, DE, FG e HI são segmen-tos paralelos. Qual a medida do ângulo x emgraus?

7.6.2 Ângulos Inscritos

De�nição 20 O ângulo inscrito relativo a uma circunferência é um ângulo quetem o vértice na circunferência e os lados são secantes a ela.

Teorema 44 Um ângulo inscrito é metade do ângulo central correspondente.

Prova: O triângulo cujos dois lados são raios é isósceles e você já sabecalcular o ângulo externo de um quadrilátero não convexo.

Page 137: OBM por assunto: Os primeiros passos olímpicos · tenha muito a acrescentar na sua vida pois o foco do treinamento é justamente para alcançar esse nível de habilidade. ... Esse

7.6. ÂNGULOS INSCRITOS E ÂNGULOS TANGENTES 137

Exercícios

1. Calcule o ângulo γ em função dos arcos α, β.

OBM

2002. Na circunferência abaixo, temos que:AB = 4, BC = 2, AC é diâmetro e os ângu-los ABD e CBD são iguais. Qual é o valor deBD?

2011. Três polígonos regulares de 8, 12 e 18 lados respectivamente, estão ins-critos em uma mesma circunferência e têm um vértice em comum. Os vérticesdos três polígonos são marcados na circunferência. Quantos vértices distintosforam marcados?

2012. Na �gura a seguir, o ângulo ∠ABC é reto; a reta r corta os segmentosAB e BC em D e E, respectivamente; as retas CD e AE se cortam em F ; P eQ são as projeções ortogonais de A e C sobre a reta r, respectivamente.

Sendo o ângulo entre as retas CD e AE igual a ∠AFD = 40o , qual é amedida de ∠PBQ?

2015. No triângulo ABC, AB = 40, AC = 42 e BC = 58. As bisse-trizes internas de ∠BAC,∠CBA e ∠ACB cortam novamente a circunferência

Page 138: OBM por assunto: Os primeiros passos olímpicos · tenha muito a acrescentar na sua vida pois o foco do treinamento é justamente para alcançar esse nível de habilidade. ... Esse

138 CAPÍTULO 7. GEOMETRIA

circunscrita de ABC em K, L e M , respectivamente. Qual é o menor ladodo triângulo determinado pelas retas tangentes à circunferência circunscrita deABC que passam por K, L e M?

2016. Em uma circunferência de raio 1 estão inscritos um hexágono regularABCDEF e um quadrado AXDY . O segmento BF intersecta os lados AX eBY do quadrado nos pontos R e S, respectivamente. A reta BD intersecta asretas AX e AY em T e U , respectivamente. Calcule os comprimentos de RS ede TU .

7.7 Áreas de �guras planas

Vamos enunciar como calcular as áreas de algumas �guras planas. Não seráexplicado o que é a área nem como isso é obtido. Deixaremos esse trabalhocomo pesquisa. Reenunciaremos a do triângulo só por completude.

1. Triângulo: b·h2

2. Setor circular de raio r e com α graus: α360oπr

2.

Lema 5 1. A área de uma linha é zero.

2. A área da união de duas �guras disjuntas é a soma das áreas das duas�guras

Exemplo: Calcule a área do retângulo sabendo a área do triângulo.

Solução:

Page 139: OBM por assunto: Os primeiros passos olímpicos · tenha muito a acrescentar na sua vida pois o foco do treinamento é justamente para alcançar esse nível de habilidade. ... Esse

7.7. ÁREAS DE FIGURAS PLANAS 139

Divida o retêngulo pela sua diagonal, como mostra a �gura abaixo, obtendoassim dois triângulos congruêntes T1 e T2, e caso você não queira mostrar quesão congruentes, veja que a área de cada um é b·h

2 , donde a área do retângulo éb · h− `, onde ` é a área da diagonal, ou seja ` = 0.

Quando for resolver problemas simplesmente ignore as linhas, não precisafalar que tem área 0.

Exercícios

1. Calcule em termos de sua base e de sua altura, a área do

a) paralelogramo;

b) trapézio.

2. Calcule a área do losango em termos das suas diagonais.

OBM

1999. A circunferência abaixo tem raio1, o arco AB mede 70o e o arco BC mede40o. Quanto mede área da região limi-tada pelas cordas AB e AC e pelo arcoBC?

1999. No quadrado ABCD o ponto E é médio de BC e o ponto F do ladoCD é tal que o ângulo AEF é reto. Aproximadamente, que porcentagem a áreado triângulo AEF representa da área do quadrado?

A) 28%. B) 31%. C) 34%. D) 36%. E) 39%.

2000. Na �gura, as distâncias entre doispontos horizontais consecutivos e as dis-tâncias entre dois pontos verticais con-secutivos são iguais a 1. Qual é a áreada região comum ao triângulo e ao qua-drado?

Page 140: OBM por assunto: Os primeiros passos olímpicos · tenha muito a acrescentar na sua vida pois o foco do treinamento é justamente para alcançar esse nível de habilidade. ... Esse

140 CAPÍTULO 7. GEOMETRIA

2000. Na �gura temos que os triângulosABC e A′B′C ′ são equiláteros e a re-gião destacada é um hexágono regular.Qual é a razão entre a área da regiãodestacada e a área do triângulo ABC?

2002. Traçando segmentos, podemos dividir um quadrado em dois quadra-dinhos congruentes, quatro trapézios congruentes e dois triângulos congruentes,conforme indica o desenho abaixo, à esquerda. Eliminando algumas dessas par-tes, podemos montar o octógono representado à direita. Que fração da área doquadrado foi eliminada?

2005. Traçando as quatro retas perpendiculares aos lados de um paralelo-gramo não retângulo pelos seus pontos médios, obtém-se uma região do planolimitada por essas quatro retas. Podemos a�rmar que a área dessa região é igualà área do paralelogramo se um dos ângulos do paralelogramo for igual a:

A) 30◦. B) 45◦. C) 60◦. D) 75◦. E) 90◦.

2005. Na �gura, todas as circunferên-cias menores têm o mesmo raio r e oscentros das circunferências que tocam acircunferência maior são vértices de umquadrado. Sejam a e b as áreas cinzasindicadas na �gura. Quanto vale a

b ?

2006. Na �gura temos dois semicírculosde diâmetros PS, de medida 4, eQR, pa-ralelo a PS. Além disso, o semicírculomenor é tangente a PS em O. Qual é aárea destacada?

Page 141: OBM por assunto: Os primeiros passos olímpicos · tenha muito a acrescentar na sua vida pois o foco do treinamento é justamente para alcançar esse nível de habilidade. ... Esse

7.7. ÁREAS DE FIGURAS PLANAS 141

2007. O desenho ao lado mostra um semicírculoe um triângulo isósceles de mesma área. Qual éo valor de tg x◦ ?

2009. Na �gura a seguir, ABCD é um quadradode lado 4, K pertence ao lado AD, L pertenceao lado AB, M pertence ao lado BC e KLMé um triângulo retângulo isósceles, sendo Lo ângulo reto. Então quanto vale a área doquadrilátero CDKM?

2009. Os círculos C1 e C2, de raios 3 e 4, res-pectivamente, são tangentes externamente emT . As tangentes externas comuns tocam C1 emP eQ e C2 em R e S. A tangente interna comumem T corta as tangentes externas nos pontos Me N , como mostra a �gura. Quanto vale a ra-zão entre as áreas dos quadriláteros MNPQ eMNRS?

2009. Uma folha de caderno de Carlos é um retângulo com dois lados (bor-das) amarelos de 24cm e dois lados (bordas) vermelhos de 36cm. Carlos pintacada ponto do retângulo na mesma cor do lado mais próximo desse ponto. Qualé a área da região pintada de amarelo?

2011. Um peso de papel tem a forma de umtriângulo de lados BC = 6cm e AB = AC =5cm e está parcialmente preenchido com água.Quando o peso de papel se apoia sobre o ladoBC, a água tem uma altura de 3cm. Qual é aaltura da água, em cm, quando o peso de papelse apoia sobre o lado AB?

2012. Os lados AB e DC do paralelogramoABCD foram divididos em 4 segmentos iguais.Os lados AD e BC foram divididos em 3 seg-mentos iguais. Os pontos de divisão foram co-nectados como indica a �gura abaixo. Se a áreade ABCD é 84, determine a área sombreada.

Page 142: OBM por assunto: Os primeiros passos olímpicos · tenha muito a acrescentar na sua vida pois o foco do treinamento é justamente para alcançar esse nível de habilidade. ... Esse

142 CAPÍTULO 7. GEOMETRIA

2012. O teorema de Morley diz que, ao traçarmos as retas que dividemcada ângulo interno de um triângulo ABC em três ângulos iguais, obtemos umtriângulo equilátero chamado triângulo de Morley de ABC, como o que estáestacado na �gura a seguir:

Qual é a medida do lado do triângulo de Morley de um triângulo retânguloisósceles cujos catetos medem 2?

2014. Considere um quadrado ABCD de lado 1. Externamente ao qua-drado, são formados os triângulos equiláteros ABE, BCF , CDG e DAH. Quala área do quadrilátero EFGH?

2015. Na �gura, os quadrados ABGHe CDEF têm lados de medidas 4cm e6cm, respectivamente. O ponto P per-tence à reta contendo os pontos B,C,G,e F , sendo C o ponto médio do ladoBG. A semirreta AP divide a �guraformada pelos dois quadrados em duasregiões, uma branca e uma cinza. Paraque essas duas regiões tenham áreasiguais, qual deve ser o valor de x = CP?

2015. No triângulo ABC, AB = 2, BC =√

2. Seja M o ponto médio dolado AB. Se ∠BAC = α e ∠BMC = β e ∠MBC = γ, então:

A) α+ β = γ.B) α+ β = 2γ.C) α+ β + γ = 180o.D) α+ β = 90o.E) α+ β = 45o.

2016. A área de um quadrado é um número inteiro de metros quadrados e éigual à área de um retângulo de lados inteiros cujo perímetro é 58 metros. Qualé a medida do lado do quadrado em metros?

2016. Na �gura abaixo, AEFG e ABCD são quadrados e o ponto E está nareta CD. Além disso, M é o ponto médio do segmento CD e C é o ponto médio

Page 143: OBM por assunto: Os primeiros passos olímpicos · tenha muito a acrescentar na sua vida pois o foco do treinamento é justamente para alcançar esse nível de habilidade. ... Esse

7.7. ÁREAS DE FIGURAS PLANAS 143

do segmento ME. Sabendo que o quadrado ABCD possui lado 6, determine arazão entre as áreas do quadrilátero CHAM e do quadrado AEFG.

7.7.1 Área do triângulo

2001. No triângulo ABC,AB = 5 e BC = 6. Qual é a área do triângulo ABC,sabendo que o ângulo ∠ACB tem a maior medida possível?

1998. O quadrilátero ABCD é um quadrado deárea 4m2. Os pontos M e N estão no meio doslados a que pertencem. Quanto vale a área dotriângulo em destaque em m2?

1999. Dois irmãos herdaram o terreno ABC coma forma de um triângulo retângulo em A, e como cateto AB de 84m de comprimento. Eles re-solveram dividir o terreno em duas partes demesma área, por um muro MN paralelo a ACcomo mostra a �gura abaixo. Assinale a op-ção que contém o valor mais aproximado do seg-mento BM.

A) 55m. B) 57m. C) 59m. D) 61m. E) 63m.

Page 144: OBM por assunto: Os primeiros passos olímpicos · tenha muito a acrescentar na sua vida pois o foco do treinamento é justamente para alcançar esse nível de habilidade. ... Esse

144 CAPÍTULO 7. GEOMETRIA

Notação:Dado um triângulo ∆ABC representaremos sua área por [ABC].Nos teoremas a seguir iremos considerar que o triângulo tem lados com

comprimentos AB = c, AC = b, BC = a e denotaremos o semiperímetro de

ABC por s, ou seja, s =a+ b+ c

2.

Teorema 45 Seja r o raio da circunferência inscrita em ABC.

[ABC] = s · r.

Prova: Some as áreas dos triâgulos ∆ABI, ∆ACI ∆CBI, onde I é o in-centro do triângulo ABC.

Teorema 46 (Fórmula de Herão)

[ABC] =√s(s− a)(s− b)(s− c).

Exercícios

1. Calcule o comprimento do inraio de um triângulo com lados a, b, c.

2. Considere dois triângulos semelhantes T1, T2 com razão de semelhança k.Calcule a razão entre os comprimentos dos inraios desses triângulos.

7.7.2 Lei dos Senos

Teorema 47

[ABC] =a · b · sin(∠ACB)

2.

Prova: Mostre que b · sin(∠ACB) é a medida altura relativa ao lado BC.

Analogamente segue que [ABC] = b·c·sin(∠BAC)2 = c·a·sin(∠CBA)

2 .

Teorema 48 (Lei dos Senos)

a

sin(∠BAC)=

b

sin(∠CBA)=

c

sin(∠ACB).

Page 145: OBM por assunto: Os primeiros passos olímpicos · tenha muito a acrescentar na sua vida pois o foco do treinamento é justamente para alcançar esse nível de habilidade. ... Esse

7.8. QUADRILÁTEROS INSCRITÍVEIS 145

Prova: Use o teorema acima.

Observação: As razões acima são também iguais a 2R, onde R é o tamanhodo circunraio, isto é raio da circunferência circunscrita ao triângulo.

Exercícios

1. Mostre que abc = 4RS, onde S é a área do triângulo de lados a, b, c e R écircunraio do mesmo triângulo.

2. Num triângulo ABC são dados ∠A = 60o, ∠B = 45o e BC = 4cm.Determine a medida de AC.

3. Em um triângulo ABC, ∠BAC = 100o e AB = AC. Seja BD a bissetrizde ∠ABC, com D sobre o lado AC. Prove que AD +BD = BC.

4. Sejam ABC um triangulo e D um ponto sobre AB tal que ∠ACD = 6α e∠DCB = α, sabendo que ∠ABC = 2α e que AD = BC, encontre o valorde α.

OBM

2005. O ponto D pertence ao lado BC do triângulo ABC. Sabendo queAB = AD = 2, BD = 1 e os ângulos BAD e CAD são congruentes, entãoqual é a medida do segmento CD?

2007. No triângulo ABC, AD é a altura relativa ao lado BC. Se AB =DC = 1. Qual é a área máxima do triângulo ABC?

2010. Seja ABC um triângulo e X,Y e Z pontos sobre os lados BC,CA,ABtais que CX

XB = AYY C = BZ

ZA = 2. Qual é a razão entre as áreas do triângulo XY Ze do triângulo cujos lados são congruentes às medianas de ABC?

2014. No triângulo ABC, AC = 5 e AB = 6. Seja P um ponto sobre abissetriz interna do ângulo ∠BAC. Se a área de ABP é 3

2 , qual é a área deAPC?

2015. Um triângulo tem lados inteiros distintos, o maior deles medindo 2015.Quais são as medidas dos dois outros lados se a área do triângulo é a menorpossível?

7.8 Quadriláteros Inscritíveis

De�nição 21 Um qudrilátero convexo é dito cíclico se pode ser inscrito numacircunferência, ou seja, se seus vértices estão sobre uma mesma circunferência.

Teorema 49 Se um quadrilátero é cíclico então a soma de dois ângulos opostosdesse quadrilátero é 180o. Reciprocamente, se as somas de dois ângulos opostosé 180o o quadrilátero é incritível.

Page 146: OBM por assunto: Os primeiros passos olímpicos · tenha muito a acrescentar na sua vida pois o foco do treinamento é justamente para alcançar esse nível de habilidade. ... Esse

146 CAPÍTULO 7. GEOMETRIA

Prova: Exercício.

Teorema 50 (Ptolomeu) Se ABCD é um quadrilátero cíclico de diagonaisAC e BD, então:

AB × CD +AD ×BC = AC ×BD.

Teorema 51 (Brahmagupta) A medida da área de um quadrilátero cíclicode lados a, b, c, d cujo semiperímetro denotado por p é a seguinte:

K =√

(p− a)(p− b)(p− c)(p− d).

Exercícios

1. Prove que um trapézio é inscritível se, e somente se, ele for isósceles (ladosnão paralelos iguais).

2. Seja P um ponto sobre o menor arco AC da circunferência circunscrita aum triângulo equilátero ABC. Prove que PB = PA+ PC.

3. Prove que, se ABCDEFG é um heptágono regular convexo, então:

1

AB=

1

AC+

1

AD.

4. (Gabriel) Prove que, um heptágono regular convexo, tem lado ` e o com-primento da menor diagonal é d então:

(d2 − `2)(d− `) = d`2.

5. Mostre que se um quadrilátero com lados a, b, c, d é inscritível e circuns-critível, então sua área é igual a

√abcd.

OBM

2011. Seja ABC um triângulo retângulo em A. O ponto D pertence ao ladoAC e é tal que BD = CD. Sejam M o ponto médio de BC e N a interseçãode AM e BD. Sendo N o ponto médio de AM , qual a medida, em graus, doângulo ∠BCA?

2011. Seja ABCD um quadrilátero inscritível (ou seja, cujos vértices perten-cem a uma circunferência) com AB = 4, BC = 8

√3, AC = 4

√13 e AD = 2

√13.

Sendo E a interseção das diagonais AC e BD, qual é o comprimento do seg-mento BE?

2016. No quadrilátero convexo ABCD, ∠BAD = ∠BCD = 60o e BC ⊥BD. Qual é a razão entre a distância de C ao circuncentro de ABD e a medidado segmento de reta CD?

Para as demonstrações dos teoremas de Ptolomeu e de Brahmagupta vejaas Revistas Eureka! número 5 e 9, respectivamente.

Page 147: OBM por assunto: Os primeiros passos olímpicos · tenha muito a acrescentar na sua vida pois o foco do treinamento é justamente para alcançar esse nível de habilidade. ... Esse

7.9. GEOMETRIA ESPACIAL 147

7.9 Geometria espacial

2005. Uma das faces de um poliedro é um hexágono regular. Qual é a quanti-dade mínima de arestas que esse poliedro pode ter?

2006. Um cubo de aresta 1 é cortadoem quatro regiões por dois planos: umdeles contém as arestas AB e CD e ooutro contém as arestas AE eDF . Qualé o volume da(s) maior(es) das quatroregiões?

2008. O brinquedo favorito de Cícero é um cone reto de vidro com 5cm dealtura. Cícero encheu o cone com areia até a altura de 3cm, como mostrado na�gura 1. Em seguida, Cícero fechou a base do cone e virou-o de cabeça parabaixo, como indicado na �gura. A que altura da base do cone, em cm, �cou amarca de areia?

2012. Considere uma pirâmide V ABCD debase quadrada. Seja P o centro da base ABCDe X,Y, Z e W pontos sobre as faces lateraistais que PXYWZ é uma pirâmide semelhante aV ABCD, com as diagonais da base XZ e YWparalelos a BC e CD, respectivamente. Qual éa razão de semelhança entre as duas pirâmides?

2013. Dizemos que duas retas ou segmentos de retas são reversas quando nãoexiste um plano que contém ambas as retas ou segmentos de retas. De quantasmaneiras podemos escolher três arestas de um cubo de modo que quaisquer duasdessas arestas são reversas?

2013. Oito dos vértices de um dodecaedro regular de aresta 1 são vérticesde um cubo. Qual é o volume desse cubo?

2014. Uma esfera de raio 1 tem como equador a base de um cone e passapelos pontos médios de suas geratrizes. Qual é a altura do cone?

Page 148: OBM por assunto: Os primeiros passos olímpicos · tenha muito a acrescentar na sua vida pois o foco do treinamento é justamente para alcançar esse nível de habilidade. ... Esse

148 CAPÍTULO 7. GEOMETRIA

2016. Considere uma pirâmide P cuja base é um polígono regular de 2016lados. Apesar de sua base ser um polígono regular, a pirâmide P não é regular,pois a projeção do seu vértice sobre o plano da base não coincide com o centroda base. No mínimo quantas faces laterais não congruentes duas a duas P tem?

Page 149: OBM por assunto: Os primeiros passos olímpicos · tenha muito a acrescentar na sua vida pois o foco do treinamento é justamente para alcançar esse nível de habilidade. ... Esse

Capítulo 8

Enunciados dos Problemas

8.1 Combinatória e Probabilidade

8.1.1 Contagem

1998. Cinco cartões numerados com 3, 4, 5, 6 e 7, respectivamente, são coloca-dos em uma caixa. Os cartões são retirados da caixa, um de cada vez e colocadossobre a mesa. Se o número de um cartão retirado é menor do que o númerodo cartão imediatamente anterior, então este cartão imediatamente anterior écolocado de volta na caixa. O procedimento continua até que todos os cartõesestejam sobre a mesa. Qual é o número máximo de vezes que retiramos cartõesda caixa?

2001. O matemático excêntrico Jones, especialista em Teoria dos Nós, temuma bota com n pares de furos pelos quais o cadarço deve passar. Para não seaborrecer, ele gosta de diversi�car as maneiras de passar o cadarço pelos furos,obedecendo sempre às seguintes regras:

i) o cadarço deve formar um padrão simétrico em relação ao eixo vertical;ii) o cadarço deve passar exatamente uma vez por cada furo, sendo indiferente

se ele o faz por cima ou por baixo;iii) o cadarço deve começar e terminar nos dois furos superiores e deve ligar

diretamente (isto é, sem passar por outros furos) os dois furos inferiores. Porexemplo, para n = 4, representamos a seguir algumas possibilidades.

Determine, em função de n ≥ 2, o número total de maneiras de passar ocadarço pelos furos obedecendo às regras acima. Observação: Maneiras comoas exibidas a seguir devem ser consideradas iguais.

149

Page 150: OBM por assunto: Os primeiros passos olímpicos · tenha muito a acrescentar na sua vida pois o foco do treinamento é justamente para alcançar esse nível de habilidade. ... Esse

150 CAPÍTULO 8. ENUNCIADOS DOS PROBLEMAS

2001. Dizemos que um conjunto A formado por 4 algarismos distintos enão nulos é intercambiável se podemos formar dois pares de números, cada umcom 2 algarismos de A, de modo que o produto dos números de cada par seja omesmo e que, em cada par, todos os dígitos de A sejam utilizados. Por exemplo,o conjunto {1, 2, 3, 6} é intercambiável pois 21× 36 = 12× 63. Determine todosos conjuntos intercambiáveis.

2002. Colocamos vários palitos sobre uma mesa de modo a formar umretângulo m× n, como mostra a �gura.

Devemos pintar cada palito de azul, vermelho ou preto de modo que cadaum dos quadradinhos da �gura seja delimitado por exatamente dois palitos deuma cor e dois de outra cor. De quantas formas podemos realizar esta pintura?

2004. Os doze alunos de uma turma de olimpíada saíam para jogar futeboltodos os dias após a aula de matemática, formando dois times de 6 jogadorescada e jogando entre si. A cada dia eles formavam dois times diferentes dostimes formados em dias anteriores. Ao �nal do ano, eles veri�caram que cada 5alunos haviam jogado juntos num mesmo time exatamente uma vez. Quantostimes diferentes foram formados ao longo do ano?

2005. No campeonato tumboliano de futebol, cada vitória vale três pontos,cada empate vale um ponto e cada derrota vale zero ponto. Um resultado éuma vitória, empate ou derrota. Sabe-se que o Flameiras não sofreu nenhumaderrota e tem 20 pontos, mas não se sabe quantas partidas esse time jogou.Quantas seqüências ordenadas de resultados o Flameiras pode ter obtido? Re-presentando vitória por V , empate por E e derrota por D, duas possibilidades,por exemplo, são (V,E,E, V,E, V, V, V,E,E) e (E, V, V, V, V, V,E, V ).

2006. O par ordenado (83, 89) é chamado de par centenário porque 83 +8 + 9 = 89 + 8 + 3 = 100, isto é, a soma de cada número com os dígitos do outronúmero é 100. Quantos são os pares centenários?

Page 151: OBM por assunto: Os primeiros passos olímpicos · tenha muito a acrescentar na sua vida pois o foco do treinamento é justamente para alcançar esse nível de habilidade. ... Esse

8.1. COMBINATÓRIA E PROBABILIDADE 151

2006. Seja n inteiro positivo. De quantas maneiras podemos distribuir n+1brinquedos distintos para n crianças de modo que toda criança receba pelo me-nos um brinquedo?

2007. Um quadrado 4× 4 é dividido em 16 quadrados unitários. Cada umdos 25 vértices desses quadrados deve ser colorido de vermelho ou azul. Ache onúmero de colorações diferentes tais que cada quadrado unitário possua exata-mente dois vértices vermelhos.

2007. Em um certo país há 21 cidades e o governo pretende construir nestradas (todas de mão dupla), sendo que cada estrada liga exatamente duasdas cidades do país. Qual o menor valor de n para que, independente de comoas estradas sejam construídas, seja possível viajar entre quaisquer duas cidades(passando, possivelmente, por cidades intermediárias)?

2008. Quantas permutações de 1, 2, 3, ..., 9 há com a propriedade de que,para todo 1 ≤ i ≤ 9, os números que aparecem entre i e i + 1 (onde i podeaparecer tanto antes como depois de i + 1) são todos menores do que i? Porexemplo, 976412358 é uma permutação com esta propriedade.

2008. Determine a quantidade de funções f : {1, 2, 3, 4, 5} → {1, 2, 3, 4, 5}tais que f(f(x)) = f(x) para todo x ∈ {1, 2, 3, 4, 5}.

2009. Determine a quantidade de números n = a1a2a3a4a5a6, de seis alga-rismos distintos, que podemos formar utilizando os algarismos 1, 2, 3, 4, 5, 6, 7, 8, 9de modo que as seguintes condições sejam satisfeitas simultaneamente:

i)a1 + a6 = a2 + a5 = a3 + a4;ii)n é divisível por 9.

2010. Cada uma das oito casas de um retângulo de duas linhas e quatrocolunas é pintada de uma entre três cores. Uma coluna é chamada de corte seas suas duas casas são da mesma cor. De quantas maneiras é possível pintar oretângulo de modo que haja exatamente um corte?

2010. Diamantino gosta de jogar futebol, mas se jogar dois dias seguidosele �ca com dores musculares. De quantas maneiras Diamantino pode escolherem quais de dez dias seguidos ele vai jogar bola sem ter dores musculares? Umamaneira é não jogar futebol em nenhum dos dias.

2011. Uma sequência de letras, com ou sem sentido, é dita alternada quandoé formada alternadamente por consoantes e vogais. Por exemplo, EZEQAF,MATEMÁTICA, LEGAL e ANIMADA são palavras alternadas, mas DSOIUF,DINHEIRO e ORDINÁRIO não são. Quantos anagramas da palavra FELICI-DADE (incluindo a palavra FELICIDADE) são sequências alternadas?

2011. Esmeralda tem um círculo de cartolina dividido em n setores circu-lares, numerados de 1 a n, no sentido horário. De quantas maneiras Esmeraldapode pintar a cartolina, pintando cada setor com uma cor, tendo disponíveis k

Page 152: OBM por assunto: Os primeiros passos olímpicos · tenha muito a acrescentar na sua vida pois o foco do treinamento é justamente para alcançar esse nível de habilidade. ... Esse

152 CAPÍTULO 8. ENUNCIADOS DOS PROBLEMAS

cores e de modo que quaisquer dois setores circulares vizinhos (isto é, que têmum segmento em comum como fronteira) tenham cores diferentes?

Note que isso implica que os setores de números 1 e n devem ter cores dife-rentes.

2013. Um bispo é uma peça do jogo de xadrez que só pode fazer movimen-tos diagonais, isto é, ele pode se deslocar quantas casas quiser desde que elasestejam em uma diagonal. Dizemos que dois bispos se atacam quando um delesestá em uma casa do tabuleiro que pode ser alcançada pelo outro bispo. Qual éo maior número de bispos que podemos colocar em um tabuleiro 8× 8 sem quehaja dois bispos se atacando?

2013. Para cobrir um tabuleiro de dimensões 1 × 112, podemos utilizarheptaminós amarelos, de dimensões 1×7, e octaminós vermelhos, de dimensões1× 8. De quantos modos podemos cobrir completamente o tabuleiro?

2014. A mediana de um conjunto {a1, a2, ..., an} com a1 < a2 < ... < an éigual à media dos dois termos centrais (an

2+an

2 +1)/2 se n é par, e ao termo cen-tral an+1

2se n é ímpar. SendoM a quantidade de subconjuntos de {1, 2, ..., 2014}

com mediana igual a 2012 , encontre o resto da divisão de M por 2014.

2015. Um subconjunto de 5 elementos do conjunto {1, 2, 3, ..., 20} é ditolargo se ao colocar os seus elementos em ordem crescente tivermos a proprie-dade de que a diferença do segundo menos o primeiro é maior que 1, do terceiropara o segundo é maior que 2, do quarto para o terceiro é maior que 3 e doquinto para o quarto é maior que 4. Existem quantos subconjuntos largos?

8.1.2 Extremo

2014. Um conjunto é dito completamente divisível se para quaisquer elementosa < b temos que a divide b. Um conjunto de inteiros positivos A é completa-mente divisível e possui 2016 como um de seus elementos. Sabendo que todos oselementos de A são menores que 2 milhões, qual o máximo número de elementosque A pode ter?

8.1.3 Jogos e Invariantes

1998. Existem 20 balas sobre uma mesa e duas crianças começam a comê-las,uma criança de cada vez. Em cada vez, cada criança deve comer pelo menosuma bala e está proibida de comer mais que a metade das balas que existemsobre a mesa. Nesta brincadeira, ganha a criança que deixar apenas uma balasobre a mesa. Qual das duas crianças pode sempre ganhar na brincadeira: aprimeira ou a segunda a jogar? Como deve fazer para ganhar?

1999. Nos extremos de um diâmetro de um círculo, escreve-se o número 1(primeiro passo). A seguir, cada semicírculo é dividido ao meio e em cada umdos seus pontos médios escreve-se a soma dos números que estão nos extremosdo semicírculo (segundo passo). A seguir, cada quarto de círculo é dividido ao

Page 153: OBM por assunto: Os primeiros passos olímpicos · tenha muito a acrescentar na sua vida pois o foco do treinamento é justamente para alcançar esse nível de habilidade. ... Esse

8.1. COMBINATÓRIA E PROBABILIDADE 153

meio e em cada um dos seus pontos médios coloca-se a soma dos números queestão nos extremos de cada arco (terceiro passo). Procede-se, assim, sucessiva-mente: sempre cada arco é dividido ao meio e em seu ponto médio é escrita asoma dos números que estão em seus extremos. Determinar a soma de todos osnúmeros escritos após 1999 passos.

1999. Determine todos os inteiros positivos n para os quais é possível mon-tarmos um retângulo 9× 10 usando peças 1× n.

2000. O campeonato Venusiano de futebol é disputado por 10 times, emdois turnos. Em cada turno cada equipe joga uma vez contra cada uma dasoutras. Suponha que o Vulcano FC vença todas as partidas do 1o. turno. Casonão vença o 2o. turno, o Vulcano FC jogará uma �nal contra o vencedor do 2o

turno, na qual terá vantagem caso faça mais pontos que o adversário durantetodo o campeonato (vitória vale 3 pontos, empate vale 1 ponto e derrota 0 pon-tos).

a) Determine o menor n tal que, se o Vulcano FC �zer exatamente n pontosno segundo turno, garantirá pelo menos a vantagem na �nal (independente decontra quem e com que placares conquiste os n pontos).

b) Determine o menor n tal que, se o Vulcano FC �zer pelo menos n pontosno segundo turno, garantirá pelo menos a vantagem na �nal (independente decontra quem e com que placares conquiste os n pontos).

2000. Para efetuar um sorteio entre os n alunos de uma escola (n > 1)se adota o seguinte procedimento: Os alunos são colocados em roda e inicia-seuma contagem da forma "um, DOIS, um, DOIS,...". Cada vez que se diz DOISo aluno correspondente é eliminado e sai da roda. A contagem prossegue atéque sobre um único aluno, que é o escolhido.

a) Para que valores de n o aluno escolhido é aquele por quem começou osorteio?

b) Se há 192 alunos na roda inicial, qual é a posição na roda do aluno esco-lhido?

2012. Esmeralda e Jade, secretárias da OBM, jogam Destrua os triângu-los. Esse jogo é disputado da seguinte forma: tem-se uma esfera e 2012 pontossobre a esfera. Em princípio todos os pares de pontos estão ligados por umsegmento. Esmeralda e Jade apagam, alternadamente, um segmento. A secre-tária que eliminar o último triângulo da esfera vence o jogo. Note que podemsobrar segmentos no �nal do jogo; eles só não formam triângulo. Se Esmeraldacomeça o jogo, qual das secretárias tem estratégia vencedora, ou seja, vence ojogo não importando como o oponente jogue? Justi�que sua resposta, exibindouma estratégia que funcione sempre.

2015. Considere um tabuleiro 2015 × 37, pintado como um tabuleiro dexadrez. Cada linha e coluna tem um botão que inverte a cor de cada casinha dalinha ou coluna correspondente, num total de 2015+37 = 2052 botões. Quantascolorações diferentes do tabuleiro podem ser obtidas?

Page 154: OBM por assunto: Os primeiros passos olímpicos · tenha muito a acrescentar na sua vida pois o foco do treinamento é justamente para alcançar esse nível de habilidade. ... Esse

154 CAPÍTULO 8. ENUNCIADOS DOS PROBLEMAS

8.1.4 Probabilidade

1999. José tem três pares de óculos, um magenta, um amarelo e um ciano.Todo dia de manhã ele escolhe um ao acaso, tendo apenas o cuidado de nuncausar o mesmo que usou no dia anterior. Se dia primeiro de agosto ele usou omagenta, qual a probabilidade de que dia 31 de agosto ele volte a usar o ma-genta?

2002. Quantos dados devem ser lançados ao mesmo tempo para maximizara probabilidade de se obter exatamente um 2?

2003. Um quadrado de lado 3 é dividido em 9 quadrados de lado unitário,formando um quadriculado. Cada quadrado unitário é pintado de azul ou ver-melho. Cada cor tem probabilidade 1

2 de ser escolhida e a cor de cada quadradoé escolhida independentemente das demais. Qual a probabilidade de obtermos,após colorirmos todos os quadrados unitários, um quadrado de lado 2 pintadointeiramente de uma mesma cor?

2006. Ao jogarmos uma certa quantidade de dados cúbicos com faces nu-meradas de 1 a 6, a probabilidade de obtermos soma dos pontos 2006 é igual àprobabilidade de obtermos soma dos pontos S. Qual é o menor valor possívelde S?

2007. Considere o conjunto A dos pares ordenados (x, y) de reais não ne-gativos tais que x + y = 2. Se a probabilidade de um elemento de A escolhidoaleatoriamente estar a uma distância da origem menor ou igual a 5

3 é p, quantovale 2535p2?

2009. No programa de auditório Toto Bola, o apresentador Ciço Magallanesdispõe de duas caixas idênticas. Um voluntário da platéia é chamado a partici-par da seguinte brincadeira: ele recebe dez bolas verdes e dez bolas vermelhase as distribui nas duas caixas, sem que o apresentador veja, e de modo que emcada caixa haja pelo menos uma bola. Em seguida, o apresentador escolhe umadas caixas e retira uma bola. Se a bola for VERDE, o voluntário ganha umcarro. Se for VERMELHA, ele ganha uma banana. A máxima probabilidadeque o voluntário tem de ganhar um carro é igual a m

n , em que m e n são inteirospositivos primos entre si. Determine o valor de m+ n.

2015. . Três pontos A,B e C são marcados no bordo de um círculo de modoque ∠BAC = 60o, ∠ABC = 80o, ∠ACB = 40o. Escolhemos ao acaso um pontoX no interior do círculo. A probabilidade de que, entre os pontos A,B e C, omais distante deX seja B é p

q , em que p e q são primos entre si. Quanto vale p·q?

8.2 Álgebra

8.2.1 Funcões e Recorrências

1998. Seja f : N→ R uma função tal que f(1) = 999 e f(1)+f(2)+ ...+f(n) =n2 · f(n), para todo inteiro positivo n. Determine o valor de f(1998).

Page 155: OBM por assunto: Os primeiros passos olímpicos · tenha muito a acrescentar na sua vida pois o foco do treinamento é justamente para alcançar esse nível de habilidade. ... Esse

8.2. ÁLGEBRA 155

2001. Determine todas as funções f : R → R tais que f(x) = f(−x) ef(x+ y) = f(x) + f(y) + 8xy + 115 para todos os reais x e y.

2001. Seja f(x) =x2

1 + x2. Calcule

f

(1

1

)+ f

(2

1

)+ f

(3

1

)+ ...+ f

(n1

)f

(1

2

)+ f

(2

2

)+ f

(3

2

)+ ...+ f

(n2

)f

(1

3

)+ f

(2

3

)+ f

(3

3

)+ ...+ f

(n3

)...

f

(1

n

)+ f

(2

n

)+ f

(3

n

)+ ...+ f

(nn

)2002. O primeiro número de uma seqüência é 7. O próximo é obtido da

seguinte maneira: Calculamos o quadrado do número anterior (72 = 49) e aseguir efetuamos a soma de seus algarismos e adicionamos 1, isto é, o segundonúmero é 4 + 9 + 1 = 14. Repetimos este processo, e de 142 = 196, temos queo terceiro número da seqüência é 1 + 9 + 6 + 1 = 17, e assim sucessivamente.Qual o 2002o elemento desta seqüência?

2005. Determine o menor valor possível do maior termo de uma progressãoaritmética com todos os seus sete termos a1, a2, a3, a4, a5, a6, a7 primos positivosdistintos.

2005. A função f : R→ R satisfaz f(x+ f(y)) = x+ f(f(y)) para todos osnúmeros reais x e y. Sabendo que f(2) = 8 , calcule f(2005).

2006. A seqüência Fn é de�nida por F1 = F2 = 1 e Fn = Fn−1 + Fn−2

para n ≥ 3. Encontre todos os pares de inteiros positivos (m,n) tais queFm · Fn = mn.

2011. Seja f uma função dos reais não nulos nos reais não nulos tal que

i)(f(x) + f(y) + f(z))2 = (f(x))2 + (f(y))2 + (f(z))2 para todos x, y, z reaisnão nulos tais que x+ y + z = 0;

ii)f(−x) = −f(x) para todo x real não nulo;iii)f(2011) = 1.

Encontre o inteiro mais próximo de f(33).

2014 A sequência a1, a2, a3, ... satisfaz a1 = 1 e an =√a2n−1 + n . Qual é o

inteiro mais próximo de a2014?

2015. Sejam f e g funções dos inteiros não negativos nos inteiros não ne-gativos tais que f(0) = g(0) = 0, f(2x + 1) = g(x), g(2x) = f(x) e f(2x) =

Page 156: OBM por assunto: Os primeiros passos olímpicos · tenha muito a acrescentar na sua vida pois o foco do treinamento é justamente para alcançar esse nível de habilidade. ... Esse

156 CAPÍTULO 8. ENUNCIADOS DOS PROBLEMAS

g(2x + 1) = x para todo x inteiro não negativo. Quantos valores de n tais que0 ≤ n ≤ 2015 satisfazem f(n) = 0?

8.2.2 Somatórios e Produtórios

2000. O número√

1 + 112 + 1

22 +√

1 + 122 + 1

32 + ... +√

1 + 120002 + 1

20012 é

racional; escreva-o na forma pq , p e q inteiros.

2003. Calcule a soman∑k=0

2k+1

32k + 1.

2004. Cada um dos números x1, x2, ..., x2004 pode ser igual a√

2 − 1 ou a√

2 + 1 . Quantos valores inteiros distintos a soman∑k=0

x2k−1x2k pode assumir?

2010. Calcule(24 + 22 + 1)(44 + 42 + 1)(64 + 62 + 1)...(324 + 322 + 1)

(14 + 12 + 1)(34 + 32 + 1)(54 + 52 + 1)...(314 + 312 + 1).

2013. Observe que 1n(n+1) = 1

n −1

n+1 . Assim podemos calcular a série

∞∑n=1

1

n(n+ 1)=

1

1 · 2+

1

2 · 3+

1

3 · 4+... =

(1− 1

2

)+

(1

2− 1

3

)+

(1

3− 1

4

)+... = 1.

Sabendo que∞∑n=1

1

n2=π2

6, o valor de

∞∑n=1

1

n(n+ 1)2é da forma A− π2

B com A

e B inteiros positivos. Determine o valor de A+B.

8.2.3 Equações do segundo grau e Polinômios

2005. Você tem que determinar o polinômio p(x) de coe�cientes inteiros posi-tivos fazendo perguntas da forma "Qual é o valor numérico de p(k)?", sendo kum inteiro positivo à sua escolha.

Qual é o menor número de perguntas su�ciente para garantir que se descubrao polinômio?

2008. Suponha que a α ∈ R é raiz de algum polinômio não-nulo comcoe�cientes racionais. O polinômio minimal de α é o polinômio de menor graum(x) tal que:

i) m(α) = 0;ii) m(x) é Mônico (isto é, o seu coe�ciente líder é 1) e todos os seus coe�-

cientes são racionais.Por exemplo, o polinômio minimal de

√2 é x2 − 2.

Determine o produto dos coe�cientes não nulos do polinômio minimal de3√−27 + 5

√33− 3

√27 + 5

√33.

2010. Sejam r e s números inteiros. Sabe-se que a equação do segundo graux2 − (r + s)x + rs + 2010 = 0 tem as duas soluções inteiras. Quantos são ospossíveis valores de |r − s|?

Page 157: OBM por assunto: Os primeiros passos olímpicos · tenha muito a acrescentar na sua vida pois o foco do treinamento é justamente para alcançar esse nível de habilidade. ... Esse

8.2. ÁLGEBRA 157

2011. A equação do segundo grau x2− 5x+m = 2011 tem pelo menos umasolução inteira. Qual é o menor valor inteiro positivo possível de m?

2011. Seja P (x) um polinômio de coe�cientes inteiros. Sabe-se que P (x) =2011 tem pelo menos duas raízes inteiras distintas iguais a 1 e t, e que P (x) = 0tem pelo menos uma raiz inteira. Determine todos os possíveis valores de t.

2012. Considere a equação ax2 +bx+c = 0, em que a, b e c são reais e a > 0.Suponha que esta equação tenha duas raízes reais r e s tais que 0 < r < 1 e0 < s < 1. Mostre que b+ c < 0.

2013. Um retângulo, o qual não é um quadrado, tem lados com compri-mentos inteiros, medidos em centímetros. Se o seu perímetro é n centímetros esua área é n centímetros quadrados, determine n.

2015. O professor Piraldo passou para Esmeralda uma equação da formaax = b, sendo a e b reais. Esmeralda se enganou e resolveu a equação bx = a,obtendo uma solução que é igual à correta menos 60. Se a solução correta é daforma m+

√n com m e n inteiros, qual é o valor de m+ n?

8.2.4 Parte Inteira e Aproximação

2007. Qual é a soma dos algarismos do inteiro mais próximo de√

111...111,(1000 uns)?

2008. Calcule o valor da soma

b 4√

1c+ b 4√

2c+ ...+ b 4√

2008c.

2009. Para cada inteiro positivo n, seja An = {x ∈ R+;x · bxc = n}.Determine a quantidade de elementos do conjunto

A1 ∪A2 ∪ ... ∪A2009.

8.2.5 Cotas e desigualdades

2008. Em uma matriz 2008 × 2008 o elemento na linha i e coluna j é o nú-mero i + j (as linhas e colunas são numeradas de 1 a 2008). Escolhem-se 2008elementos desta matriz de modo que não haja dois elementos escolhidos numamesma linha ou coluna. Os elementos são multiplicados. Qual o menor produtoque se pode obter desta forma?

2009. Seja c a maior constante real para a qual

x2 + 3y2 ≥ c(x2 + xy + 4y2),

para todos x, y reais. Determine o inteiro mais próximo de 2009c.

Page 158: OBM por assunto: Os primeiros passos olímpicos · tenha muito a acrescentar na sua vida pois o foco do treinamento é justamente para alcançar esse nível de habilidade. ... Esse

158 CAPÍTULO 8. ENUNCIADOS DOS PROBLEMAS

8.2.6 Várias variáveis

2006. Esmeralda e Jade correm em sentidos opostos em uma pista circular, co-meçando em pontos diametralmente opostos. O primeiro cruzamento entre elasocorre depois de Esmeralda ter percorrido 200 metros. O segundo cruzamentoocorre após Jade ter percorrido 350 metros entre o primeiro e o segundo pontode encontro. As velocidades das moças são constantes. Qual é o tamanho dapista, em metros?

2010. Encontre todas as soluções inteiras não negativas x ≤ y ≤ z dosistema {

x+ y + z = 77xy + yz + zx+ xyz = 946.

2011. Encontre todas as soluções reais (x, y, z) do sistema2y = x+

1

x

2z =1

y+ y

2x = z +1

z.

2012. Sendo a, b, c reais tais que ab(a+ b+ c) = 1001, bc(a+ b+ c) = 2002e ca(a+ b+ c) = 3003, encontre abc.

2013. Determine o número de quádruplas (x, y, z, w) de reais tais quex3 = y + z + wy3 = z + w + xz3 = w + x+ yw3 = x+ y + z.

2015. Os reais a, b e c satisfazem as equações

1

ab= b+ 2c,

1

bc= 2c+ 3a,

1

ca= 3a+ b.

Temos (a+ b+ c)3 = pq , com p e q inteiros primos entre si e q > 0. Calcule p+ q.

8.3 Teoria dos Números

8.3.1 Equações e divisibilidade

1998. O menor múltiplo de 1998 que possui apenas os algarismos 0 e 9 é 9990.Qual é o menor múltiplo de 1998 que possui apenas os algarismos 0 e 3?

1999. Encontre as soluções inteiras de x3 − y3 = 999.

1999. Determine o maior natural n para o qual existe uma reordena-ção (a, b, c, d) de (3, 6, 9, 12) (isto é, a, b, c, d = 3, 6, 9, 12) tal que o número

Page 159: OBM por assunto: Os primeiros passos olímpicos · tenha muito a acrescentar na sua vida pois o foco do treinamento é justamente para alcançar esse nível de habilidade. ... Esse

8.3. TEORIA DOS NÚMEROS 159

n√

3a6b9c12d seja inteiro. Justi�que sua resposta.

2001. Se a n-ésima OBM é realizada em um ano que é divisível por n, dize-mos que esse ano é super-olímpico. Por exemplo, o ano 2001, em que está sendorealizada a 23a OBM, é super-olímpico pois 2001 = 87 × 23 é divisível por 23.Determine todos os anos super-olímpicos, sabendo que a OBM nunca deixoude ser realizada desde sua primeira edição, em 1979, e supondo que continuarásendo realizada todo ano.

2002. Determine o maior natural k para o qual existe um inteiro n tal que3k divide n3 − 3n2 + 22.

2003. Dizemos que um númeroN de quatro algarismos é biquadrado quandoé igual à soma dos quadrados de dois números: um é formado pelos dois pri-meiros algarismos de N , na ordem em que aparecem em N e o outro, pelos doisúltimos algarismos de N , também na ordem em queaparecem em N .

Por exemplo, 1233 é biquadrado pois 1233 = 122 + 332. Encontre um outronúmero biquadrado.

2004. Determine todas as soluções da equação n · 2n−1 = m2 − 1 com n em naturais.

2005. Determine todos os pares de inteiros (x, y) tais que

9xy − x2 − 8y2 = 2005.

2006. Encontre todos os pares de inteiros positivos (a, b) tais que ab + 1divide (a+ 1)(b+ 1).

2007. Ache todos os pares (x, y) de inteiros positivos tais que

2(x+ y) + xy = x2 + y2.

2007. Encontre todos os números n de seis algarismos da forma AAABBB,em que A e B são algarismos diferentes e não nulos e n + 1 é um quadradoperfeito.

2008. Um inteiro positivo n é chamado de auto-replicante se os últimos dígi-tos de n2 formam o número n. Por exemplo, 25 é auto-replicante pois 252 = 625.Determine a soma de todos os números auto-replicantes com exatamente 4 dí-gitos .

2008. Determine todos os inteiros positivos m e n tais que m2 + 161 = 3n.

2010. Seja N o menor número inteiro positivo que multiplicado por 33 re-sulta em um número cujos algarismos são todos iguais a 7. Determine a somados algarismos de N .

2011. Qual é o maior valor possível do mdc de dois números distintos per-tencentes ao conjunto {1, 2, 3, · · · , 2011}?

Page 160: OBM por assunto: Os primeiros passos olímpicos · tenha muito a acrescentar na sua vida pois o foco do treinamento é justamente para alcançar esse nível de habilidade. ... Esse

160 CAPÍTULO 8. ENUNCIADOS DOS PROBLEMAS

2012. Os dois menores números primos da forma n2 + 5 são 62 + 5 = 41 e122 + 5 = 149. Qual é o terceiro menor primo dessa forma?

2013. Seja A = {1, 2, · · · , 20} o conjunto dos 20 primeiros inteiros positi-vos. Para cada subconjunto X de 15 elementos de a , calculamos o produtop(X) de seus elementos. Por exemplo, p({1, 2, · · · , 15}) = 1 · 2 · · · · · 15 = 15!.Qual é o máximo divisor comum dos

(2015

)produtos p(X) obtidos com todos os

subconjuntos de 15 elementos de A?

2013. Escrevemos a soma dos recíprocos dos números de a como a fraçãoirredutível , ou seja,

1 +1

2+

1

3+ · · ·+ 1

2013=A

B, mdc(A,B) = 1.

Qual é o maior valor inteiro n tal que B é múltiplo de 3n.

8.3.2 Teorema Fundamental da Aritimética

2000. Qual é o menor inteiro positivo que é o dobro de um cubo e o quíntuplode um quadrado?

2004. Dizemos que um número inteiro positivo é sinistro quando a somade seus fatores primos é igual à soma dos expoentes de sua decomposição emfatores primos. Encontre todos os números sinistros de quatro algarismos.

2007. Quantos divisores positivos do número 123456 são menores que 2007?

2012. Arnaldo pensou em um número de quatro dígitos e desa�ou Bernardoa descobrir qual era o número. Para tanto, passou as seguintes três dicas paraBernardo, sendo que exatamente uma das dicas é falsa.

i) Dica 1: O número é um cubo perfeito;ii)Dica 2: O número é o menor número de quatro dígitos que possui quatro

divisores positivos;iii) Dica 3: O número é múltiplo de 59.Qual o número pensado por Arnaldo?

8.3.3 Teorema Chinês dos Restos e Sistemas de Congruên-cias

2005. Seja a um número inteiro positivo tal que a é múltiplo de 5, a + 1 émúltiplo de 7, a+2 é múltiplo de 9 e a+3 é múltiplo de 11. Determine o menorvalor que a pode assumir.

2009. Sejamm e n dois inteiros positivos primos entre si. O Teorema Chinêsdos Restos a�rma que, dados inteiros i e j com 0 ≤ i < m e 0 ≤ j < n, existeexatamente um inteiro a, com 0 ≤ a < m · n, tal que o resto da divisão de apor m é igual a i e o resto da divisão de a por n é igual a j. Por exemplo, param = 3 e n = 7, temos que 19 é o único número que deixa restos 1 e 5 quando

Page 161: OBM por assunto: Os primeiros passos olímpicos · tenha muito a acrescentar na sua vida pois o foco do treinamento é justamente para alcançar esse nível de habilidade. ... Esse

8.3. TEORIA DOS NÚMEROS 161

dividido por 3 e 7, respectivamente. Assim, na tabela a seguir, cada número de0 a 20 aparecerá exatamente uma vez.

Qual a soma dos números das casas destacadas?

2009. Determine o maior inteiro n menor que 10000 tal que 2n + n sejadivisível por 5.

2014. O imparial de n é igual ao produto de todos os naturais ímpares me-nores ou iguais a n. Quais são os três últimos algarismos do imparial de 2014?

8.3.4 Fermat e Euler

2015. Qual é o menor inteiro a > 1 para o qual existe n inteiro positivo tal quea2n − 1 é múltiplo de 2015?

2010. Uma mesa de bilhar tem o formato de um quadrado ABCD. Su-perPablo tem uma missão especial: ele deve dar uma tacada em uma bola debilhar, inicialmente colocada no vértice A, de modo que, após bater exatamente2010 vezes nos lados do quadrado, a bola chegue, pela primeira vez, a um vérticedo quadrado. Quantos são os possíveis valores do ângulo formado pelo lado ABcom a trajetória inicial da bola?

Observação: ao bater nos lados do quadrado, a bola sofre re�exão perfeita,ou seja, o ângulo de incidência é igual ao ângulo de re�exão. Suponha tambémque a bola seja um ponto.

Page 162: OBM por assunto: Os primeiros passos olímpicos · tenha muito a acrescentar na sua vida pois o foco do treinamento é justamente para alcançar esse nível de habilidade. ... Esse

162 CAPÍTULO 8. ENUNCIADOS DOS PROBLEMAS

8.4 Geometria

8.4.1 Básico

1998. Uma reta que passa pelos pontos médios de dois lados opostos de umquadrilátero convexo forma ângulos iguais com ambas as diagonais. Mostre queas duas diagonais têm o mesmo comprimento.

2000. O retângulo ao lado está dividido em 9 quadrados, A, B,C,D,E, F,G,He I. O quadrado A tem lado 1. Qual é o lado do quadrado I?

2002. Para quais inteiros positivos n existe um polígono não regular de nlados, inscrito em uma circunferência, e com todos os ângulos internos de mesmamedida?

2005. Na �gura, ABCDE é um pentágono regular e AEF é um triânguloeqüilátero. Seja P um ponto sobre o segmento BF , no interior de ABCDE, etal que o ângulo PEA mede 12o, como mostra a �gura abaixo.

Calcule a medida, em graus, do ângulo PAC.

Page 163: OBM por assunto: Os primeiros passos olímpicos · tenha muito a acrescentar na sua vida pois o foco do treinamento é justamente para alcançar esse nível de habilidade. ... Esse

8.4. GEOMETRIA 163

2003. Entre 15 números reais distintos, o menor deles igual a 1, não há trêsque podem ser lados de um triângulo. Quais valores o maior dos 15 númerospode assumir?

8.4.2 Semelhança de Triângulos e Triângulos Retângulos

1998. Sobre os lados AB e AC de um triângulo acutângulo ABC são construí-dos, exteriormente ao triângulo, semicírculos tendo estes lados como diâmetros.As retas contendo as alturas relativas aos lados AB e AC cortam esses semicír-culos nos pontos P e Q. Prove que AP = AQ.

2004. Seja ABCD um trapézio retângulo de bases AB e CD, com ângulosretos em A e D. Dado que a diagonal menor BD é perpendicular ao lado BC,determine o menor valor possível para a razão CD

AD .

2005. Um prisma é reto e tem como base um triângulo equilátero. Umplano corta o prisma mas não corta nenhuma de suas bases, determinando umasecção triangular de lados a, b e c. Calcule o lado da base do prisma em funçãode a, b e c.

2006. Na �gura a seguir, o pentágono regular ABCDE e o triângulo EFGestão inscritos na circunferência Co, eM é ponto médio de BC. Para qual valorde α , em graus, os triângulos EFG e HIG são semelhantes?

2006. No triângulo ABC tem-se AB = 4, AC = 3 e o ângulo BAC mede60o. Seja D o ponto de intersecção entre a reta perpendicular a AB passandopor B e a reta perpendicular a AC passando por C. Determine a distância entreos ortocentros dos triângulos ABC e BCD.

2007. No quadrilátero convexo ABCD, ∠BAC + ∠CBA = 120◦, AD =BC = 5 e AB = 8. Externamente ao lado CD, construímos o triângulo eqüilá-tero CDE. Calcule a área do triângulo ABE.

2008. Um trapézio isósceles ABCD, com lados paralelos AB e CD, é talque a diagonal BD mede 100m e o ângulo BDC mede 30◦. Seja S a área do

Page 164: OBM por assunto: Os primeiros passos olímpicos · tenha muito a acrescentar na sua vida pois o foco do treinamento é justamente para alcançar esse nível de habilidade. ... Esse

164 CAPÍTULO 8. ENUNCIADOS DOS PROBLEMAS

trapézio em m2. Determine S ·√

3.

2011. O ângulo interno do vértice A de um triângulo acutângulo ABC mede75 graus. A altura relativa ao vértice A toca o lado BC no ponto D. As distân-cias de D ao vértice B e ao ortocentro do triângulo são ambas iguais a 10cm.Qual é a área do triângulo ABC, aproximada para o inteiro mais próximo? Senecessário, use

√3 ' 1, 732 .

2012. Uma tira retangular de papel ABCD é dobrada ao longo das linhasEF e HG de forma tal que os vértices A e B são levados para um mesmo pontoA′ da mediatriz do segmento AB e o ângulo ∠HA′E é reto. Obtém-se assim opentágono A′EFGH.

Sabe-se que as bordas inferiores da tira (segmentos FC ′ e GD′ na �gura)se cortam no ponto médio M do lado AB. O lado menor da tira mede 1 e amedida do lado maior mede a+

√b , com a e b inteiros positivos. Quanto é a+b?

2012. No triângulo ABC, seja AD a altura relativa a BC. Quantos triân-gulos não congruentes satisfazem 1

AB2 + 1AC2 = 1

AD2 com AD = 2012 e BD eCD ambos inteiros? Note que AB e AC não precisam ser inteiros.

2012. Sejam ABCD um quadrado, E o ponto médio do lado BC, F oponto médio do lado CD. Constroem-se os triângulos equiláteros ABG e BEHde forma que G está no interior do quadrado, e H no seu exterior. Determine oângulo agudo entre as retas BF e GH.

2014. Uma caixa de madeira em forma de paralelepípedo retângulo possuidimensões 3×4×6. Ela está sobre o chão com uma de suas faces completamenteapoiada sobre o chão. Uma fonte de luz emite raios paralelos de luz formando45o com o chão. Considerando apenas essa fonte de luz, qual a área da maiorsombra possível da caixa no chão? Não inclua a base da caixa na sombra.

8.4.3 Lei dos Senos e dos Cossenos

2000. O trapézio ABCD tem bases AB e CD. O lado DA mede x e o ladoBC mede 2x. A soma dos ângulos DAB e ABC é 120o. Determine o ânguloDAB.

2001. No triângulo ABC, a mediana e a altura relativas ao vértice A di-videm o ângulo BC em três ângulos de mesma medida. Determine as medidasdos ângulos do triângulo ABC.

Page 165: OBM por assunto: Os primeiros passos olímpicos · tenha muito a acrescentar na sua vida pois o foco do treinamento é justamente para alcançar esse nível de habilidade. ... Esse

8.4. GEOMETRIA 165

2010. As bissetrizes internas dos ângulos A e C do triângulo ABC cortam-se no ponto I. Sabe-se que AI = BC e que m(ICA) = 2m(IAC). Determine amedida do ângulo ABC.

2013. O quadrado ABCD está inscrito em um círculo de raio 30. A cordaAM corta a diagonal BD no ponto P . Se AM = 50, encontre o valor de AP .

2014. Um círculo tangencia os lados do quadrilátero ABCD . Os pontosde tangência são R sobre AB, S sobre BC,T sobre CD e U sobre DA. Sabe-seque AU = 1,DU = 2, BS = 2 e CS = 4 . Calcule o comprimento SU .

8.4.4 Áreas

2005. Um terreno quadrangular foi dividido em quatro lotes menores por duascercas retas unindo os pontos médios dos lados do terreno. As áreas de três doslotes estão indicadas em metros quadrados no mapa ao lado.

Qual é a área do quarto lote, representado pela região destacada no mapa?

2009. No triângulo retângulo ABC, ∠A = 90o, AB = 5cm e BC = 9cm.Se I é o incentro de ABC, determine o comprimento do segmento CI.

2014. No desenho abaixo, o triângulo ABC é equilátero e BD = CE =AB/3 . A razão EG/GD pode ser escrita na forma m/n, mdc(m,n) = 1.Quanto vale m+ n?

Page 166: OBM por assunto: Os primeiros passos olímpicos · tenha muito a acrescentar na sua vida pois o foco do treinamento é justamente para alcançar esse nível de habilidade. ... Esse

166 CAPÍTULO 8. ENUNCIADOS DOS PROBLEMAS

8.4.5 Pontos notáveis e Ângulos na circunferência

1999. Seja ABCD um quadrado. Escolhemos pontos M,N,P,Q respectiva-mente sobre AB,BC,CD e DA, de modo que as circunferências circunscri-tas aos triângulos MBN e PDQ sejam tangentes exteriormente. Mostre queMN + PQ ≥ AC.

2002. Em um quadrilátero convexo ABCD, os lados opostos AD e BC sãocongruentes e os pontos médios das diagonais AC e BD são distintos. Proveque a reta determinada pelos pontos médios das diagonais forma ângulos iguaiscom AD e BC.

2004. Sejam H, I e O o ortocentro, o incentro e o circuncentro do triânguloABC, respectivamente. A reta CI corta o circuncírculo de ABC no ponto L,distinto de C. Sabe-se que AB = IL e AH = OH. Determine os ângulos dotriângulo ABC.

2007. O triângulo ABC é retângulo em B. Sejam I o centro da circunfe-rência inscrita em ABC e O o ponto médio do lado AC. Se AOI = 45o, quantomede, em graus, o ângulo ACB?

2008. Um trapézio ABCD, com lados paralelos AB e CD, está inscrito emuma circunferência de raio 25. Sabe-se que CD é um diâmetro e a altura dessetrapézio é 24. Seja E um ponto no arco menor determinado por A e B e sejamF e G os pontos de interseção de ED e EC com AB, respectivamente. CalculeAF ·BGFG

.

2009. No triângulo ABC, temos ∠BAC = 120◦ e BC = 12cm. A circun-ferência inscrita em ABC tangencia os lados AB e AC, respectivamente, nospontosD e E. SejamK e L os pontos onde a retaDE intersecta a circunferênciade diâmetro BC. Determine a distância entre os pontos médios dos segmentosBC e KL.

2010. Na �gura a seguir, as três circunferências em traço contínuo são tan-gentes às retas r e s e a circunferência tracejada passa pelos pontos A,B,C e D.Além disso, a circunferência menor é tangente também a AD e a circunferênciamaior é também tangente a BC. Se os raios das circunferências externas aoquadrilátero ABCD são 8 e 18, calcule o raio R da circunferência inscrita emABCD.

Page 167: OBM por assunto: Os primeiros passos olímpicos · tenha muito a acrescentar na sua vida pois o foco do treinamento é justamente para alcançar esse nível de habilidade. ... Esse

8.4. GEOMETRIA 167

2011. No triângulo ABC, o ângulo ∠BAC mede 45o. O círculo de diâmetroBC corta os lados AB e AC em D e E, respectivamente. Dado que DE = 10,encontre a distância do ponto médio M de BC à reta DE.

2013. Pode-se provar que num triângulo acutângulo ABC, o triânguloDEFcom D,E e F sobre os lados BC,CA e AB respectivamente com perímetro mí-nimo é obtido quando D,E eF são as interseções das alturas com os lados. Taltriângulo é o triângulo órtico de ABC. Se AB = 13, BC = 14 e CA = 15, operímetro de seu triângulo órtico pode ser escrito na forma a

b , com a e b inteirosprimos entre si. Determine o valor de a+ b.

2015. Duas circunferências C1 e C2 se intersectam nos pontos A e B. Atangente a C1 por A corta C2 novamente no ponto P e a tangente a C2 porA corta C1 novamente no ponto Q. Sabendo que PB = 640 e QB = 1000,determine o comprimento do segmento AB.

8.4.6 Potência de Ponto

2012. Dois círculos se cortam em dois pontos A e B. Seja X um ponto sobreo segmento AB. Dez retas, todas passando por X, cortam os círculos em umtotal de quarenta pontos, quatro para cada reta. Qual é a quantidade mínimade quadriláteros cíclicos cujos quatro vértices estão entre esses quarenta pontos?Obs: um quadrilátero é cíclico se, e somente se, existe um círculo que passa porseus quatro vértices.

2015. Seja ABCD um paralelogramo com AB = 8 e BC = 4. O círculoΓ passa por A, C e pelo ponto médio M de BC, e corta o lado CD no pontoP 6= C. Sabe-se que AD é tangente a Γ. Calcule a medida do segmento MP .

8.4.7 Geometria Espacial

2006. Qual a maior quantidade de lados que pode ter uma secção determinadapor um plano em um octaedro regular?

Page 168: OBM por assunto: Os primeiros passos olímpicos · tenha muito a acrescentar na sua vida pois o foco do treinamento é justamente para alcançar esse nível de habilidade. ... Esse

168 CAPÍTULO 8. ENUNCIADOS DOS PROBLEMAS

Page 169: OBM por assunto: Os primeiros passos olímpicos · tenha muito a acrescentar na sua vida pois o foco do treinamento é justamente para alcançar esse nível de habilidade. ... Esse

Referências Bibliográ�cas

[1] Provas da OBM de 1998 a 2016 http://www.obm.org.br/como-se-preparar/provas-e-gabaritos/

[2] Revista Eureka! números 1, 5, 8, 11, 27 e 37.

[3] Morgado A., Pitombeira J., Carvalho P., Fernandez P. Análise Combina-tória e Probabilidade, 10.ed., Sociedade Brasileira de Matemática, 2016;

[4] Martinez F.B; et al. Teoria dos números: um passeio com primos e outrosnúmeros familiars pelo mundo inteiro, IMPA, 2010

[5] Fernandez A., Oliveira K., Iniciação à Matemática: um curso com proble-mas e soluções, 2. ed., SBM, 2012

[6] Hefez A., Indução Matemática, OBMEP, 2009

[7] Holanda B., et al, Banco de problemas para a XV Olimpíada Rioplatensede Matemática , 2006

[8] Notas de Aula do POTI http://poti.impa.br/

[9] Andrade M.V., Notas de aula.

[10] Dolce O., Pompeo J.N., Fundamentos de matemática elementar - vol. 9:geometria plana, 9.ed., Atual Editora, 2013

[11] L. Larson, Problem Solving Through Problems, Springer-Verlag, New York,1983

[12] E. Lozansky, C. Rousseau, Winning Solutions, Springer-Verlag, New York,1996

[13] Engel A., Problem-Solving Strategies, Springer-Verlag, New York, 1998

[14] Gelca R., Andreescu T. Putnam and Beyond, Springer Science & BusinessMedia, LLC, 2007

[15] Gelca R., Andreescu T. Mathematical Olympiad Challenges, 2.ed., SpringerScience & Business Media, LLC, 2009

[16] Stevens J., Olympiad Number Theory Through Challenging Problems,3.ed.

169